You are on page 1of 508

AKP Exams

Categorized
First edition—Dec 2021
Disclaimer
Dear Doctors, please Don’t take
answers of these questions for
granted , make your best effort to
verify the answers .
also note that, a lot of questions
are incomplete ,
It will help but also may deceive you
So consider them as a guide in your
study.
Best wishes
Dr. Shereen

‫نسألكم الدعاء‬
Index
1. Neurology 4
2. Developmental assessment 61
3. Endocrinology 64
4. DM 86
5. Haematology 103
6. Oncology 122
7. Immunology 133
8. GIT 140
9. Haepatology 154
10. Nutrition,Obesity,Allergy 159
11. Nephrology 170
12. Cardiology 192
13. Respiratory /ENT 224
14. Infections 264
15. Immunization 289
16. Neonatology 296
17. Psychiatry 328
18. Musculoskeletal 345
19. Metabolic medicine 359
20. Emergency 372
21. Pharmacology, poisoning,Patient safety 386
22. Ethics 413
23. Safegaurd 428
24. Palliative 447
25. Dermatology 452
26. Opthalmology 470
27. Genetics 484
28. Statistics 504
;’

Neurology

4
Neurology

Sept 2021
1. EMQ
A. ADEM
B. GBS
C. FRIEDRICH ATAXIA
D. INFANTILE SPASM
E. ALCOHOL INTOXICATION
F. CERBELLAR TUMOR
G. SSPE
H. BACTERIAL MENINGITIS
I. LEAD POSIONING
J. POSTVIRAL CEREBELLITIS
K. POSTERIOR FOSSA TUMOR

A. 12 years old boy, presented with frequent falling and ataxic


gait had URTI 10 days ago, on examination, power and tone NL in both
lower limbs, but DTR difficult to elicit reflexes, cranial nerves exam
was NL.
GBS
B. 18 months old, previously well, came with sudden onset of
ataxic gait and two episodes of vomiting, he was drowsy but
arousable, but GCS 15/15, other nervous examinations was normal.
POSTVIRAL CEREBELLITIS
C. 8 years old Pakistani child presented by abnormal behaviour for
example head lagging and decreased school performance. He never
went to GP clinic and did not follow up in any medical clinic came by
behavioural changes and delayed school performance.
SSPE

2. 3 yrs. old case of girl sleep at 7 pm and wake up at 9 pm start to jump,


kicking and crying. These attacks increase when she is unwell, repeated
episodes. What to do?
a. Reassurance
b. EEG with sleep deprivation
c. Ferritin

5
Neurology

3. Adolescent with one sided facial palsy, left side weakness, tingling, when
he came to hospital his condition improved spontaneously, but after 1 hr.
he developed headache and recurrence of the same condition.
BP: 125/75. What is the diagnosis?
a. Hypertensive encephalopathy
b. Hemiplegic migraine
c. Bell’s palsy

4. Known case of NF type 1, Wt. and Ht. were normal, ophthalmic review
done with normal developmental assessment. What other measures of
assessment other than ophthalmology should be done annually?
a. B/P measuring
b. Cranial MRI
c. Renal U/S

5. 2 years old girl, was normal till 1 year, started walking at 14 months old
and was able to use spoon. Now she is not able to speak, can’t feed
herself with spoon, history of 2 afebrile generalized seizures, asking
about diagnosis
a. Rett syndrome ??
b. Lennox gastaut
c. Landau kleiffner ??

6. Scenario about adolescent girl, 13 years, who have headache and


vomiting, amenorrhea for the last 5 months, she denies any sexual
relation, abdominal US was normal, high HCG in urine, T4 is low, TSH is
normal, urine osmolarity is low 118, high serum osmolarity 301 (not
remember exactly), asking about next investigation
a. Transvaginal US to exclude pregnancy
b. Cranial MRI
c. Serum HCG

6
Neurology

7. 7 years old child has poor academic achievement. EEG: as below.Which


medication will you give?

a. Ethoxemide
b. Lamotrigine
c. Vigabatrin
d. Gapabentin

8. 2 years old child with 2 attacks of epilepsy in the last 9 months, the
second one is the cause of this referral. Patient wakes up at night,
abnormal voice, unable to speak, tingling sensation in part of eye and lip,
recover alone, EEG was shown to you. Patient and his parents are keen to
stop these seizures, what is the next step to do?
a. Reassurance and discharge
b. Prescribe appropriate anti-epileptics
c. Keep diary and review after 4-6 months
d. Repeat EEG

9. 2 years old, delivered preterm, one of twin. He has delayed walking and
he is reluctant to stand but cruise. He can say 2-word sentences, feed
himself, can use his upper limbs freely, the other twin walked by 20
months, asking about the diagnosis?
a. Spastic diplegia
b. DMD
c. CP
d. DDH
e. Global developmental delay

7
Neurology

10. Scenario about known case of DMD, presented with headache, lethargy,
weakness, poor chest expansion, and heart is clear, fundus is normal,
asking about most appropriate investigation for cause of this condition?
a. ABGs
b. CK
c. MRI brain
d. CXR.
11. Scenario regarding child with urine retention and full bladder on
palpation. He developed sensory level at T10, lost her sensation below it.
Asking about diagnosis
a. Transverse Myelitis
b. MS
c. GBS
d. Spinal cord tumour
12. 13 yrs old female who was travelled to Scotland with her family
presented with recurrent early morning vomiting. And nausea during the
day she complains of difficulty at school and recently her mother noticed
that her daughter developed difficulties in watching TV and difficulties in
reading Class board despite she set in the first raw. MRI FINDING
revealed blurring of optic disc. What is the most likely Dx of her
condition?
a. Brain tumor
b. Lyme disease
c. School avoidance

13. Child diagnosed with spastic paraplegia with sever planter flexion
causing pain and interference with walking on left side with difficulty to
wear clothes. On physiotherapy for 3 months and on baclofen. What is
the next step?
a. Intrathecal baclofen
b. Botulinum toxin A injection
c. Selective proximal rhizotomy
d. Oral diazepam
e. Orthopaedic referral

8
Neurology

14. 5 weeks boy has floppy internal rotation of upper limbs & flex wrist, frog
like lower limbs, decrease muscle tone, spontaneous movements, brisk
reflexes in upper and lower limbs. He was normal till 2-3 weeks then
developed weight loss, difficult feeding, drooling, cough when swallowing
and extended neck. What is the diagnosis?
(exam photo is different)
a. Cervical spinal cord compression
b. Arnold chiari
c. Neuronal migration disorder
d. Syringeomyelia

15. Picture of baby (not on nasal oxygen, not attached to monitor, narrow
chest, distended abdomen, alert face), presented with Hypotonia since
birth. What is the Dx?
a. Muscular dystrophy
b. Myontonic dystrophy
c. Myasthenia gravis
d. Cleido cranial dystosis
e. SMA type 1

16. 2 days history of wt loss, right upper and lower facial palsy and
convergent squint?
a. Facial nerve palsy
b. Intracranial tumor
c. Pontine tumor
d. Bell’s palsy
17. Scenario about young child with delayed motor milestones and delayed
speech, try to stand from lying position with help of his hand. What is
the investigation to reach diagnosis?
a. CK level
b. EMG
c. MRI brain
d. Muscle biopsy

9
Neurology

May 2021
1. An-11-month-old infant with VP shunt, came with sunken fontanelle,
bilateral brisk reflexes, irritability, and spasticity. There was no Fever. His
HR: 110/min and RR 36/ min, What is the cause of these abnormalities?
a. Shunt infection
b. Shunt displacement
c. Shunt block
d. Shunt over drainage
e. Shunt leaking

2. EMQ
A. CP
B. Friedreich's ataxia
C. Duchenne muscular dystrophy
D. Becker’s muscular dystrophy
E. SMA
F. Myasthenia graves
G. Emotional neglect
A. A 12-year-old with ataxia, tremor, nystagmus, absent peripheral
reflexes, hypertonia and incoordination.
Friedreich's ataxia

B. A 21-month-old child of young parents came with delayed walking,


when he was put on the floor from baggy, he was not crawling. He
still has no word, can feed himself by bottle and weighs 9 kg.
Duchenne muscular dystrophy
C. A 12-year-old, was recently unable to jump, now has difficulty
walking with Trendelenburg gait. He is tired and his reflexes are
present but slightly decreased.
Becker’s muscular dystrophy

10
Neurology

3. A previously healthy 19-month-old child woke up that day with weakness


of the left side of his body with picture of MRI flare. What is the
diagnosis?

a. Localized encephalitis
b. Ischemic Stroke
c. Meningitis
d. Astrocytoma

4. A teenage girl complains of early morning headache for 1 year that was
not improving with analgesia and associated with occasional nausea but
no other symptoms. There was a family history of brain tumor. What is
the next step for diagnosis and management?

a. MRI brain
b. LP
c. Acetazolamide
d. Mannitol

5. A 13-year-old boy has generalized tonic colonic seizures and recurrent


vacant episodes occurring frequently and affecting his school
performance. His mother mentioned that he was clumsy and dropped
things in the morning. What should be given?
a. Carbamazepine
b. Levetiracetam ??
c. Na Valproate ??
d. Ethosuximide

11
Neurology

6. A teenage girl with recurrent attacks of blank episodes reported both at


home and school occurring when she concentrated on her lessons and
her colleagues noticed that she often closed her eyes that sometimes
rolled up. When she gains consciousness, she is aware that she has
missed her lesson. She used to regain consciousness when her colleagues
call her name or flicker fingers beside her. She feels sleepy all day. She
had a history of leg giving way when she laughs. What is the diagnosis?

a. day dreaming
b. Narcolepsy
c. Absence seizure
d. Pseudo seizure

7. A child met with an RTA 6 month ago. He had subdural hematoma and it
was evacuated but still having convulsion and behavioral change. What
is the diagnosis?

a. Cortical atrophy
b. Frontal lobe infarction
c. Brain edema
d. Hydrocephalus

8. A 5-year-old child with convulsions. What is the diagnosis?


a. TS
b. CMV
c. Toxoplasma
d. TB

12
Neurology

9. What is the diagnosis?

a. Left UMNL
b. Left LMNL
c. Right UMNL
d. Right LMNL
e. Right Horner's syndrome
f. Left Horner's syndrome

10. A patient had URTI and received amoxicillin 10 days ago. Now he is
febrile with Temp 38.6 C’. He has weakness in his legs and cannot reach
out for a toy. There was hypotonia and abnormal neck extension when
lying down.
CBC: high WBCs with Neutrophilia
Blood Glucose: 6.2
CSF analysis: WBS 10
Lymphocyte 10
Neutrophile 0
Protein 2.6
Glucose 4.6 (normal 2.2-4.4)
No organism
Culture bending
What is the diagnosis?
a. TB meningitis
b. Partially treated meningitis ??
c. Guillain barre syndrome
d. Bacterial Meningitis
(Viral meningitis was not in the options)

13
Neurology

Jan 2021
1. Child diagnosed with spastic paraplegia with sever planter flexion causing
pain and interference with walking on left side. On physiotherapy for 3
months, next step?

a. intrathecal baclofen
b. botulinum toxin A injection ??
c. selective proximal rhizotomy
d. oral diazepam
e. orthopaedic referral

2. A boy with Learning difficulties, show a photo with his parents- what is
the syndrome?
The mother has mask face and the child and his father are smiling

a. William syndrome
b. myasthenia gravis
c. fragile X
d. myotonic dystrophy
e. Noonan syndrome

3. Child have dark pigmentation in the axilla (axillary freckling) with 2-3 caffe
au lait patches, what other features confirm diagnosis?

a. Pigmented nodules in the iris


b. Shagreen patch
c. Subungual fibroma (fleshy)
d. Rhabdomyoma (heart)

14
Neurology

4. An MRI of child with cerebellar symptoms, ataxia who had a history of


varicella (4 months ago), diagnosis?

a. Post varicella
cerebellitis
b. cerebellar atrophy
c. Dandy walker
d. Medulloblastoma

5. 16 years old boy from Pakistan with fever for one week, there was history
of Frontal sinusitis diagnosed by GP and received Amoxicillin. The patient
was riding bicycle and developed convulsions and fell on ground. CT head
showing?

a. Brain Abscess
b. Tuberculoma
c. Epidural hematoma (subdural)
d. Glioma

6. Case of lower extremities flaccid paralysis, after upper resp infection, MRI
done was normal choose TWO investigations to confirm the diagnosis?
a. LP
b. Nerve conduction study
c. Stool culture for polio

7. Headache for 6 Months, no Vomiting, frontal and temporal regions,


currently on paracetamol and ibuprofen twice daily. What will you do?
a. Stop Analgesics
b. LP

15
Neurology

8. EMQ :
A. myotonic dystrophy
B. SMA
C. non ketotic hyperglycinemia
D. myasthenia gravis

A. baby has poor feeding, mother has recent onset of diplopia


myasthenia gravis
B. baby with birth wt 2,2 kg has poor feeding and inverted V shape
mouth myotonic dystrophy
C. baby with hiccups and myoclonic convulsions, CSF glycine was high.
Non ketotic hyperglycenemia

9. 8 years old girl came with 2 weeks history of affected vision (6th cranial
nerve palsy) and unilateral optic neuritis, unilateral motor weakness, MRI
show high signal on subcortical area, csf lymphocytes 17, diagnosis?

a. multiple sclerosis
b. ADEM
c. viral meningitis

10. 5 years old with left vesicular preauricular rash, left side facial palsy,
abnormal behavior, GCS 8 , what’s the investigation to reach diagnosis?
a. ENT referral
b. CT scan
c. EEG
d. LP
e. MRI

11. Scenario of infant has frequent colic after feeding, Irritability,


abnormal flexion of limbs. Diagnosis?
Infantile spasm

16
Neurology

12. a photo of boy with eye swelling for 6 mo, few hyperpigmented spot
on trunk- diagnosis?

a. NF 1
b. Tuberous sclerosis
c. Neurofibromatosis type 2

13. 16 years old girl on sodium valproate, fit free 1 yr, patient ask about
contraception and risk of pregnancy, choose TWO best advices.
a. stop antiepileptic
b. give POP
c. change to another antiepileptic ??
d. refer for contraceptive advice and family planning ??
e. refer to epilepsy specialist nurse ??
f. Refer to adult transition service for epilepsy

Sept 2020
1. EMQ: Antiepileptics
Carbamazepine
Ethosuximide
Sodium valproate (Wt. Gain)
Topiramate
Vigabatrin
Lamotrigine
Clozapine
A. Antiepileptic drug in certain ethnic group can cause Steven Johnson
Syndrome Carbamazepine
B. Specific antiepileptic only used for single type of seizure
Ethosuximide
C. Antiepileptic for intractable epilepsy cause rapid weight loss
Topiramate

17
Neurology

2. Child with Cerebral Palsy 12 years on wheelchair, on sodium valproate for


refractory seizures asking what you would do to prevent future low impact
fractures.
a. Active training program ??
b. Hydrotherapy
c. Oral bisphosphonate ??
d. Vibration therapy
e. Support frames

3. Child had chicken pox 1 month ago and now presented with abnormal
gait and shaking of hands, what to do?
a. MRI brain
b. Viral serology
c. LP

4. Child came with parental concern about his vision that he sits near TV.
He has history of bilateral glue ear with some hearing loss and he follows
with ENT surgeon. He develops unilateral decrease in visual acuity; Lt.
6/18, Rt. 6/6 and found brown pigmentation on skin, Dx?
a. Optic glioma
b. Acoustic neuroma
c. Optic neuritis

5. Which of these babies with delayed or abnormal development need MRI


brain? 2 options ‫عليه خالف شديد لم نصل إلجابه نهائيه‬
a. Hand preference before 1 year
b. No social smile at 3 months
c. A boy not walking by 18 months
d. No eye contact to caregivers
e. Cannot build 2 cubes by 2 year
f. Only 3 words by two 2 years

18
Neurology

6.EMQ:
Tension headache
Cluster headache
Migraine
Space occupying lesion
Medication over use headache
IIH
A. Unilateral Lt. Orbital severe pain with lacrimation, not responding to
analgesia, lasts for 3 hours
Cluster headache
B. Bilateral Frontal and occipital headache with pallor and nausea with no
vomiting or photophobia and sometimes responds to paracetamol and
Ibuprofen. His attacks occurs 2 times /week for the last 6 months
Tension headache
C. Frontal headache bilateral with photophobia and nausea and morning
vomiting, headache on awakening, Poor response to analgesics, normal
fundus and neurological examination
Space occupying lesion

7. Scenario about GBS: 3 weeks ago she had URTI then came with lower
limb weakness and absent reflexes, What is investigation to confirm dx?
Need TWO options
a. CSF analysis
b. NERVE CONDUCTIOn
c. EMG
d. MRI spine

19
Neurology

Jan 2020
1. Picture of cervical stenosis, long scenario, weakness in Upper limbs
Spinal compression

2. Picture like this I think, 11- or 13-years boy, what the diagnosis?

a. Adenoma sebaceous.
b. Acne
c. glue sniff
d. discoid lupus
e. anticonvulsant effect

3. child has ptosis, decrease pupillary reaction. Eye out and downward
3rd nerve palsy

4. Picture of neonate of facial palsy, e-asymmetrical facial cry

a. LT UMNL
b. LT LMNL
c. RT UMNL
d. RT LMNL
e. asymmetrical facial cry

20
Neurology

5. Scenario with picture (teenager was found collapsed in the toilet)


a. Cerebral hemorrhage
b. Subarachnoid hemorrhage
c. Subdural hemorrhage
d. Interventricular hemorrhage

6. EEG absent 3spikes and waves ask about treatment?

Ethusoxamide

7. 14 years old, Boy, History of headache, fatigue, 3 syncope, nausea, Abd


pain left side weakness and tingles in hand, drooling of saliva ,slightly high
pressure , improved then after one hour ,headache with same symptoms
again?
Hemiplegic migraine

21
Neurology

8. Epileptic boy improved on valproate, school performance improved, but


aggressive at home What to do?

a. Stop valproate
b. substitution by carbamazepine withdrawal sodium valproate gradually
c. Check sodium valproate level
d. Continue and tell parent this is known side effect and is part of disease
g. Change to carbamazepine

9.EMQ
Motor stereotype
athetosis CP
Gilles de la Tourette syndrome
Opsoclonus myoclonus
Psychogenic tics
Sydenham chorea

A. 13yr abnormal movement heart murmur, Sydenham chorea


B. 2 yr, abnormal movement, repeated fall, history of active resuscitation in
neonatal period athetotic CP
C. Child with nocturnal enuresis after social problem Family history of
divorce and recently abnormal movement in eyelid with abnormal
movement of face , there were blinking she can able to stop
psychogenic tic

10. History of URTI, absent reflexes in lower limbs, (GBS) what to do


Nerve conduction studies

22
Neurology

11. CT picture of Case of convulsions and learning difficulties


a. Tuberous sclerosis
b. CMV
c. Toxoplasmosis

12. Adolescent boy with different type of seizures treat


Valoprate

13. Myotonic dystrophy picture

Sept 2019
1. female with myoclonic seizure, ask about drug of choice
a. Leviteracitam
b. lamotrigen
c. NA valproate
d. Topiramate

2. History of hypotonic since Birth & Picture of


child with narrow chest alert face
a. SMA type 1
b. myotonic dystrophy
c. cleidocranial

23
Neurology

3. CT picture of patient who undergo surgery for post traumatic


haemorrhage show improvement
after procedure then 6-months later
develop focal convulsion and
behaviour abnormality
(this not typical picture there was no
white shadow in ventricle)
a. frontal lobe atrophy
b. brain abscess
c. cerebral oedema
d. cortical infection

4.15-yrs old with seizure refuse to start


treatment, parent agree for
treatment,patient has good school performance,, seizure occur 6time in
previous 7yrs( not sure) last attack occur 4month ago,, EEG not normal
a. persuade him to agree as it parent wish
b. no need now can be given later if more seizure
c. persuade boy & Council him about risk of sudden death from seizure
d. repeat EEG
e. Do MRI and give treatment if abnormal

5. question about transverse myelitis scenario of sensory level,, inability to


walk urine incontinence and other motor manifestation can't
remember,,
a. transverse myelitis
b. GBS

24
Neurology

6. picture of CT brain of patient take aggressive antibiotic for febrile


neutropenia ,,not improve ,,he known case of ALL ,,with focal
convulsion CT show lesion rounded in rt lobe(like cyst) ,
need 2 option

a. anti fungal
b. refer to neurosurgery
c. steroid
d. change antibiotic
e. CT chest
f. repeat cbc
g. bone marrow
h. echo

Fungal abcess

7. question about patient with nystagmus and limited up gaze eye ,,,absent
pupillary reflex
a. oculomotor
lesion
b. parinaud
syndrome
c. trochlear nerve

25
Neurology

8. patient with headache for 2-months and eye picture (fundoscopy) show
papilledema what to do?

a. MRI
b. LP
c. Acetazolamide
d. Steroid

9. known case of Duchene muscular dystrophy with decrease chest


movement, tired, what to do?
a. Ck
b. Echo
c. Polysomnography
d. blood gas

10. girl with abnormal movement in arm for 6month ,change in behavior
heart mummer with history of fever
a. Sydenham chorea

11. Patient has abnormal movement in face increase when teacher talk to
him, ,he is aware of it but cannot control it ,,occur in school and home,
4-months duration ,he has past history of viral meningitis

a. simple tics
b. post anxiety tics
c. focal epilepsy
d. ADHD

26
Neurology

12. girl height decreasing 50 to 25 centile,, weight25 to 50 increased to


75centile, no increase shoes size not change for long time with 2yrs
history of headache,,
a. craniopharyngioma
b. posterior fossa tumor
c. hypothyroidism
d. turner syndrome

13. headache for 1 month mother become increase concern because her
daughter became pale and space out just,, before the attach ,,family
history of migraine ,,what to do?
a. MRI
b. EEG
c. Reassure the family

14. EMQ:
A. baby 10 month presented with ataxia
post viral cerebilitis
B. baby with decrease school performance, behavioral change no history of
gp visit lead//SSPE
C. 18month
Alcohol

15. baby 5wk increasing head circumferance 2wk before 75th centile
now 98centile
a. familial
b. hydrocephalus

27
Neurology

May 2019
1. 20 months with history of IVH RESOLVE SPONTANOUSLY HC -98
PERCENTILE ... weight and height normal. and normal motor development
have social problem and speech.ASKING ABOUT Cause of this?
a. HEMORRHGIC hydrocephalus
b. PVL frontal lobe
c. hearing loss

2. patient with ataxia, diagnosis?

a. chicken pox
b. ADEM
c. brain tumour

3. Picture of lower back (tethered cord) or sacral dimple, done x-ray spine
normal, investigation of choice?

a. MRI spine
b. MRI brain
c. LP

4. Picture of plexiform neuroma ask for


diagnosis?
NF1

28
Neurology

5. EMQ:

A. a boy presented with nystagmus. Ataxia, difficulty in playing games had


pescavus .
Fredrich ataxia
B. 12 yrs old boy weak ness in gym, Trendelenburg gait became tired with
going upstairs
Backer
C. 21 months boy. young parents. he cant stand independantly. he can't talk
but feeding bottle by himself
Duchine

6. 15-years female myoclonic seizure on levetiracetam not controlled on


oral contraceptivee, with which drug we can replace?
a. lamotrigine
b. sodium valproate
c. Clonazepam

7. 13y boy looks backer lower limp weakness want initial investigation?
a. CK
b. EMG
c. NCS
d. LP
e. Genetics

29
Neurology

8. MRI of 2 years patient of seizure

a. NF
b. TS
c. CMV
d. Sturge wiper $

9. 12-yrs boy with (senario of myoclonic epilepsy) clumsy in the morning


drug of choice?
a. Na valbroate
b. Lamotrigine

10. he looks like dad, he takes him self seriously not laughing for jokes
a. myotonic dystrophy

11. EMQ
A. child in beach, playing with mom, step in cold water then become pale
call ambulance and he is well
reflex anoxic siezure
B. child when he wake up cannot speak but can memorized action that
happened at night, when wake up do EEG that is normal.
night mares?
C. collapse during activity, positive family history
long qt

30
Neurology

Jan 2019
1. 12 year old from AFGHANISTAN, seizures, truncal Ataxia, behavioural
changes slurred speech, facial myoclonus, bilateral optic disc pallor
,upward gaze ,measles at 2 years chickenpox 6 years, unknown vaccination
a. SSPE
b. ADEM
c. posterior fossa tumour
d. lead poisoning

2. A girl has headache, BMI 36, no papilloma,MRI normal, best investigation?


a. Catecholamines
b. LP with manometry

3. A picture of CT of infant brought by his grandmother who is taking care


of him, brain oedema, midline shift, Subdural haemorrhage,
What is the most appropriate action?

a. Ophthalmology
b. skeletal survey
c. MRI
d. social services

31
Neurology

4. A 4-year-old child developed a new behaviour recently and is increasing,


the last event was when he was playing with his brother in the garden
and struggled to gain his toy, he screamed and became pale and
collapsed for few minutes before gaining his full consciousness,
he is normal in between the attacks, diagnosis?
Reflex anoxic seizures

5. A Picture of a child who has cyanotic heart disease, he had unilateral


Ptosis and meiosis what is the cause of facial appearance:

a. Horner syndrome
b. DiGoerge
c. Neuroblastoma

6. EMQ:
A. Can't lift legs after URTI, diminished reflexes?
Guillain Barre syndrome
B. Patient have tender calf, CK 600
acute viral myositis
C. Juvenile dermatomyositis

7. 2 days history of weight loss, right upper and lower facial palsy and
convergent squint?
a. pontine tumour
b. intracranial tumour
c. Facial nerve palsy
d. bells palsy

32
Neurology

8. A case of pruritus, had peripheral neuropathy and some


clumsiness, ataxia, jaundice known case of progressive familial
cholestasis diagnosis?
a. vit B12
b. vit E

9. A teenage girl has episode of going blank, close her eyes, sometimes eye
rolling, wake when her friends call her name or flickering their fingers
abnormal movement, sleeping in class:
a. day dreaming
b. Absence seizures
c. Complex partial seizure
d. Narcolepsy

10. antenatal scan showed echogenicity in the heart?


a. NF1
b. NF2
c. Tuberous sclerosis (rhabdomyoma)
d. PHACE

11. An MRI of child with cerebellar symptoms, had a history of varicella,


diagnosis?
a. Post varicella syndrome
b. cerebellar atrophy
c. Dandy walker
d. Medulloblastoma

33
Neurology

12. 2-year-old child was ok till one year then started language
regression, before eating by spoon now cannot?
a. Landu Kleifner syndrome
b. Rett syndrome

13. well controlled epileptic for 1-year A girl is taking sodium valproate
wants OCP:
a. Different antiepileptic
b. progesterone only pills
c. continue sodium valproate
d. refer to ob/gyn to offer contraceptive

Sept 2018
1. 15-yr old girl, has boyfriend, juvenile myoclonic seizers insist to take
valproate after you told her teratogenic side effects
what’s 2 management?
a. check vit d
b. FBC monthly
c. high dose folic
d. make sure she take UpToDate contraceptives
e. EEG
f. liver enzymes

2. Picture of facial palsy


LMNL

34
Neurology

3. 5 weeks boy floppy internal rotation of upper limbs & flex wrist, frog like
lower limbs, decrease muscle tone , spontaneous movements , up & lower
limp reflexes brisk , he was normal till 2-3
weeks then develop weight loss,
difficult feeding, drooling, cough when
swallowing , extended neck , picture
of MRI , diagnosis ?
a. compressed cervical spine
b. neuronal immigration
c. syringomylia
d. cellebellar tumor
e. hydrocephalus
f. Arnold chiari

4. Picture of familythe child and father smiling,the mother isn’t smiling ,


child and mother have mild ptosis , he has learning difficulty ?
a. William
b. fragile x
c. myotonic dystrophy
d. myopathy
e. mythesnia gravis
f. noonan
5. Girl is now 7-yrs-old came with defect speech, incontinence, progressive
muscle weakness, she was normal till 4 years old, she sat independent
at 8 months, walk independent 14 month, draw circle at 2.5 y ,start to join
2 words in sentence in 2 yrs.
A. what was her developmental pattern at 2 .5-yrs?
a. normal
b. delayed motor
B. What’s the diagnosis?
a. tethered cord
b. leukodystrophy
c. cp

35
Neurology

6. Child with ALL on aggressive antibiotics for febrile neutropenia, brain


changes? picture of brain MRI with circular lesion

a. astrocytoma
b. fungal abscess
c. leukmoid deposit

7. 7-yrs girl with deterioration in school

A. what is the EEG showing?


3 hz spikes wave
B. Treatment
Na valoprate

36
Neurology

8. 15-yr-old boy collapsed suddenly at school toilet, bp =160/110, bilateral


upgoing planter, picture of CT brain
with white lesion?
a. intra cerebral haemorrhage
b. subarachnoid he
c. brain abscess
d. astrocytoma
e. ischaemic stroke

9. Picture one eye ptosis, absent pupillary reflex

3rd nerve palsy

10. 14-year-old girl Headache, fatigue for 1 month, 3 syncope, vomiting,


intermittent abdominal pain, nausea, Last menstruation 3 month ago,
deny sexual activity, Bp 107/64, Na 150, K 3.9, Hco3 23.4 , TSH 1.2,
Free t4 low, Osmolality 309, Urine osmolality 118, Urine B-HCG positive
+++, Us abdominal pelvis normal, Investigation????
a. Serum HCG
b. Prolactin
c. Water deprivation test
d. MRI head
e. Transvaginal us
f. U/s thyroid

37
Neurology

May 2018
1. 12-years-old intelligent boy was good academically but recently
deteriorate with symptoms of sleep deprivation in class when the school
doctor came he told him that he had an episode of falling down and
developed rhythmic movement for 2 minutes but no loss of memory.
he went to parties lately, etc ..looks like absence seizure
A. what is your diagnosis?
a. Psychomotor ??
b. Narcolepsy ??
B. What’s investigation to do?
a. sleep deprivation EEG for him

2. 4-years-old with fall down from a chair, the child was drowsy unbalance,
CT brain normal and GCS at hospital was I guess 6, then suddenly he
return to normal what is your diagnosis?
brain concussion

3. a girl develop stiffness of her arm and legs after she came back from the
garden where she came to help her brother, brother cutting his hand,
she has episode like this before what is your diagnosis?
a. Vasovagal attack
b. Syncope
c. Partial complex seizure
d. Generalized seizure

4. a case of GBS and MRI done and they asked 2 next investigation, last
weak has URTI , absent reflexes ?
a. Lumbr puncture
b. Nerve conduction study

38
Neurology

5. picture of of narrow chest of a child with hypotonia since birth

a. SMA
b. myotonic dystrophy
c. muscular dystrophy
d. thanatotrophic dwarphidm

alert face— SMA and follow up with lung


function

6. 15-years-old with headache and obese, BMI= 28, child with normal MRI
brain, but papilledema (blurred margin of optic disc) on eye examination

a. MRV
b. LP
c. mannitol infusion
d. antibiotics
e. refer for neurosurgery

39
Neurology

7. male child with hypotonia with symptoms in mother with history of


difficult labour, polyhydramnios, also he has follow up with
physiotherapy, the baby born distress

congenital myotonic dystrophy

8. CSF data with normal every things Here the patient has fever and neck
stiffness received amoxicillin CSF glucose .6 ,protein =1.7 , lymphocytosis
65 , neutrophils 35
a. Viral mengitis
b. Partial treated meningitis
c. Tuberclus meningitis

9. EMQ
A. Restricted lateral gaze
brain stem tumor or stroke >>abducent nerve injury
B. Bitemporal hemanopia
pituitary tumor>>> optic chiasma
C. Lt eye sudden vision loss
amaurosis fugax, central retinal vein occlusion

10. picture baby 5-months with NG and monitor, developmental delay and
sever hypotonia since birth, since birth,
alert face , has large head what
test to confirm?
genetics testing
SMA, follow up lung function

40
Neurology

Jan 2018
1. Romanian 12-yrs-old had 6 months duration of emotional lability
clumsiness slurred speech, pan systolic murmur radiating to axilla,
abnormal movement in face and hands mainly what is your diagnosis?
a. Wilson
b. Huntington
c. Psychosomatic
d. cerebellar stroke
e. Sydenham’s chorea
f. SLE

2. 15-yrs had absence attack and 2 tonic colonic seizures what to give?
a. sodium valoproate
b. carbimazipines
c. topramate
d. lamotirogen

3. 14-yrs-old intermittent weakness arms and legs for 6 weeks had more
weakness on left arm now unable to left her hand normal in between
episodes labs showed K 2.9 what diagnosis?
a. periodic paralysis
b. cataplexy
c. myasthenia
d. fatigue syndrome

4. CT brain post trauma continued to have


seizure after surgery:
a. infarction
b. haemorrhage
c. ventricular dilatation
d. hydrocephalus
e. cerebral atrophy

41
Neurology

5. CT brain 4-yrs with complex focal seizures what is your diagnosis:


a. Tuberous sclerosis
b. Sturge Webber $
c. toxoplasmosis
d. non accidental injury
e. CMV
f. Herps
g. NF
6. 6-yrs-old with headache unwel vomiting leg pain now can not stand with
neck stiffness no fever now developed more weakness progressive to
proximal muscles more over right had absent tendon reflex

A. where is the lesion?


a. neuromuscular junction
b. muscle cells.
c. ant horn cells
d. peripheral nerves
e. motor cortex

B. What is the diagnosis?


a. Poliomyelitis
b. Guillain barre
c. Herps
d. Tumours
e. bacterial meningitis

c. What are the complications?


a. acute renal failure
b. cardionenic shock
c. respiratory failure
d. icreased intracranial pressure

42
Neurology

7. 12-yrs had 3 weeks history of being ill with 2 days of drowsiness and neck
stiffness CSF protein 1, glucose 1.5, cells are 700 lymphocytes blood
glucose 5.4 what investigation to do?
a. ZN stain
b. C/S listeria
c. cryptococcal C/S
d. PCR simplex virus
e. electron microscopy

8. 9-months unconscious generalized convulsions 1 hour ago on examination


was irritable unwell had inflamed throat and non-bulging tympanic
membranes temp 37.8. RR 18 PR 120. Normal ophthalmic examination
hypotonic with decreased tendon reflexes urine bag showed no glucose or
protein, chest and abdomen were normal with CT brain normal he was
diagnosed before as otitis media 2 doses of amoxicillin were given, Labs
show glucose 4.2, CSF glucose 2.1, polymorphs 45, proteins 0.7,
lymphocytes 20 What is your diagnosis?
a. TB meningitis
b. bacterial meningitis
c. febrile convulsions
d. viral meningitis
e. encephalitis
f. partially treated meningitis

43
Neurology

9. CT brain for 15-yrs old with frontal headache fever received amoxicillin
now febrile, vomiting, tenderness over RT frontal sinus, what is your
diagnosis?
a. Abscess
b. Infarction
c. Tuberculoma
d. Hematoma
e. Glioblastoma

(The lesion was between the cerebrum and the


skull bone, almost half circular with radio dense
borders ,,, I opted Abcess) An image of an epidural
abcess .. this is much bigger than the one in the
exam ... but both are epidural, not cerebral

10. 19-months delayed walking NSVD normal Apgar, birth weight 2.1 kg
immunized no known illness can not walk unsupported or supported he
can be pull him self to stand he has unrecognizable words was normal on
examination what is your diagnosis?
a. global developmental delay
b. familial
c. Fragile X
d. IUGR
e. Emotional
f. Hypothyroidism
g. Autism
h. CP
i. Duchenne muscular dystrophy

11. A girl with microcephaly, developmental delay, loss of acquired skills,


hand movements
Rett $

44
Neurology

12. An image LMNL facial palsy LEFT side

13. 18-month-old refuse to walk had chicken pox 1 month ago, now has
vomiting and not tolerating oral feeds, immunized on examination HR
150, RR 30, abdomen was soft had dry mucus membranes
developmental normal unsteady hands and his hands are shaking
what to do?
a. Ammonia
b. viral serology
c. CT brain
d. LP
e. Nerve conduction

14. boy who is frequently falling no other abnormality except calf muscle
hypertrophy how to diagnose?
a. EMG
b. muscle biopsy
c. CK level

15. picture CT brain had signs of increased intracranial pressure what to do?
a. Neurosurgery
b. Endocrinology
c. Mannitol
d. Dexamethasone
e. Ophthalmology

45
Neurology

16. EMQ:
a. neglect
b. rickets
c. duchenne muscular dystrophy
d. becker
e. Fredrich ataxia
f. myotonic dystrophy
g. CP
h. Brain tumour

A. 21-month-old can not walk, has no words, independently bottle fed


looked normal on examination --- Duchenne

B. 12-yrs increasing clumsiness and fall feels tired to walk and has
Trendelenburg gait and weak Reflexes--- Becker

Sept 2017
1. Girl with wide base gait, +Romberg, absent ankle and knee jerks, normal
vision, mother is on gluten free diet

a. Fredrich ataxia
b. B12 deficiency

2. on sodium valproate with sign of biochemical rickets?


a. change AED
b. add vit D and ca

3. Photo right ptosis


a. rt oculomotor
b. congenital

46
Neurology

4. 11-ys has recurrent headache, awake on headache weakness right face


tongue and rt arm, blood pressure is normal
Hemiplegic migraine

5. CT brain 6 months after RTA but still with seizures & behaviour
disorder
Brain atrophy—mostly frontal lobe

6. infantile spasm Tuberous sclerosis- medicine used?


Vigabatrin

7. Post chicken pox presented with weakness and ataxia➔


varicella cerebllitis

8. Girl obese headache


benign IIH

9. parents education-?
a. buccal midazolam is better than rectal diazepam
b. can have resp depression

47
Neurology

May 2017
1. Picture of child swollen unilateral orbit has also a skin mark
neurofibromatosis type 1

2.child known epilepsy on Na-valproate has investigation showing vit-D


deficiency what to do?
a. stop the drug
b. continue drug and give vit D

3. EMQ new-born with hypotonia scenarios


A. mouth v shape upper lip mother development delay
myotonic dystrophy

48
Neurology

4. case of GBS asking for the causative organism


a. Shigella
b. staph auras
c. Campylobacter

5. 17-years old teenager 3-5min episodes of generalized tonic convulsion


past history of same episode one month and when he was 7 years refuse
treatment (so has only 3 episodes) EEG normal
agree not to give now until more frequent

6. Down syndrome child has fever and symptoms of ICP had corrected
cardiac surgery
a. Brain abscess
b. Encephalitis
7. scenario of GB syndrome investigation, need 2?

a. Electromyogram
b. nerve conduction
c. CSF analysis
8. Adolescent with one sided facial palsy and other neurological signs and
for few hours, resolved but facial palsy still there during examination, BP
125/75 Diagnosis?
a. Hypertensive encephalopathy
b. Hemiplegic migraine

9. Scenario absence seizure asking for the drug?


Ethosuximide- Na valproate – Lamotrigine

10. scenario for severe combined degeneration of spinal cord just like past
test girl with multi neurological signs and has past history of measles
SSPE

49
Neurology

11. NF 1 question asking for criteria to diagnose need 2


criteria of NF Cclinical diagnostic criteria require the presence of two or more of
the following:

1. six or more café-au- lait patches sized >15 mm (or >5 mm before
puberty)
2. a plexiform neurofibroma or two or more cutan- eous
neurofibromas
3. axillary or inguinal freckling
4. sphenoid wing dysplasia or long bone pseud- arthrosis
5. optic nerve glioma
6. two or more Lisch nodules (iris hamartomas)
7. a first-degree relative with NF1.

12. clear infant scenario for crying salam movement ..diagnosis??


infantile spasm

Jan 2017
1. family picture, Mom has ptosis in the picture ,Dad is okay work as actor,
Child has history of delayed walking &learning difficulties
MYOTONIC DYSTROPHY

2. spot diagnosis of Picture of specimen paper. .


(ptosis & miosis ) on one side

Horner syndrome

50
Neurology

3. spot diagnosis of lower motor facial nerve palsy.

The apparently normal side is the


affected side
(don't get confused )
similar question in new format book. .
So in the uploaded picture it's left LMNL

4. case history of headache, Picture of papilledema, Ask about next action?.


brain MRI

5. case of neurofibromatosis. 3 years old. . Developmental delay.. Asks


about 3 most appropriate investigations and referals:

a. Ophthalmology referal
b. Developmental assessment
c. MRIbrain

51
Neurology

May 2016
1. 15-year-old boy with headache (mother gave him paracetamol) went with
his friends and came back night mother when he was watching TV
suddenly collapsed GCS <8 what initial management?
ABC with Intubation Sample paper

Dx is sub arachnid hemorrhage

2. Child picture with Hx of neonatal hypoglycemia needed intubation


a. congenital myotonic dystrophy
b. Prader Willi syndrome
c. CHARGE
the key point fish mouth (inverted V shaped) upper lip appearance

diabetes may
be associated
with adult MD
— diabetic
mother ,may
be

3. Child has bilateral deafness and now developed reduced visual acuity on
left eye, brown spots on his skins what is other associated feature or
cause of deafenss?
a. ash leaf macule (TS)
b. acoustic neuroma (NF2) (bilateral schwannoma of vestibular nerve
CN8)

52
Neurology

4. History of typical case of juvenile myoclonic epilepsy teenage breakfast


tremor and fallings symptom appear when he spent all night with his
friend (sleep deprivation) ask about treatment?
Sodium valproate

5. A case of 1- year girl has absence epilepsy what is treatment?


ethosuximide 1st line or lamotrigine ELS

6. Acase of night seizure while sleeping ask about treatment?


No treatment (benign rolandic epilepsy)
EEG pattern is Centro-temporal spikes

7. Infant with TS has infantile spasm ask about treatment?


vigabatrin

Infentile spasm as general

First line treatment.➡prednisone /tetracosactide(ACTH) and vigabatrin

Second line treatment ➡Na valporate and nitrazepam

Third line treatment ➡vit B6 pyridoxine

8. Picture of NF1 asked about follow up


a. femoral x ray
b. CT brain
c. BP
d. ophthalmology
*optic glioma, lisch nodules
*suspected high blood pressure due to risk of renal artery stenosis and
pheochromocytoma

53
Neurology

9. CT scan shows:
dilated ventricles
Hydrocephalus

10. MRI for lissencephaly


Infant with hypotonia learning difficulties brain CT ask for 2 findings

a. Dilated ventricles
b. abscenet gyruses
brain ‫ممسوح‬

11. Scenario of GBS (Guillain barre syndrome) MRI diagnosis need


2 other investigation
a. CSF ( high protein )
b. EMG
Nerve conduction study
FVC for follow up

54
Neurology

12. A case of Idiopathic intracranial Hypertension (ICP) immediate action?


a. refer to ophthalmologist
(risk of blindness and papilledema not immediately as LP
b. LP manometry (diagnostic and therapeutics)

13. Qs about brain concussion:


24h loss of conscious after head trauma

14. Qs from sample paper: Benign neonatal seizures

15. History of otitis media & signs of increase ICP, then developed
convulsions, Investigations?
CT brain ---- Brain abscess

16. Picture of papilledema, what to do next?


a. Refer to ophthalmology
b. CT brain

55
Neurology

Jan 2016
1. Gail 12 y with weakness in her upper Rt limb since morning can't lift
it with history of previous intermittent episode in the lower limbs,
inv all normal except k + 2.6 what diagnosis :
Periodic paralysis

2. Pic of child with Rt ptosis and big eye lid and history of skin mark
what diagnosis:
a. Neurofibromatosis 1
b. Neurofibromatosis 2
c. Tuberus sclerosis

3. Pic of girl with fever had infected dorsum of the toes bilaterally and
amputated Lt big toe what diagnosis
a. Familial dysautonomia
b. DM
c. Menin. septicemia

4. Extending matching q about headache:


A. one scenario with night vomiting papilledema and uncle died of brain
tumor -- Brain tumor
B. Headache intermittent BP high BMI high normal eye exam
Idiopathic ICP
C. Headache with confusion at the episode with photophobia
Migraine

56
Neurology

May 2015
1. EMQ 4 questions about headache
a. 1 with classic migraine
b. one with cluster headaches
c. tension headache
d. headache with vomiting getting worse but fundoscopy normal..rest
neuro exam IIH/space occupying lesion

2. EMQ 3 questions about seizures:


a. Rolandic
b. reflex anoxic
c. AS.

3. 7 YEAR OLD WITH ABSENCE SEIZURES..Rx..?


a. Valproate
b. lamotrigine
First line: valproate/ethosuximide

Second line: lamotrigine

4. an MRI which showed cervical spine constriction. Child was floppy


a. arnold Chiari
b. cervical spine constriction
arnold Chiari herniation of cerebellar tonsils

5. baby with facial palsy…picture. Can’t remember if it was UMN or LMN


♦️ LMNL: deviated mouth to other side open eye , loss of nasal fold

♦️ UMNL: upper part of the face not affected as it is double innervated

57
Neurology

6. child with calf muscle pain and slightly delayed deep tendon reflexes
a. GB
b. viral myositis
♦️ viral myoscitis: no deep reflex affection with high cpk

♦️ GBS: there is pain then myocsitis , ascending paralysis

7. EEG -- absence seizures

Sept 2014
1. Picture Rt side Horner Syndrome

58
Neurology

2. EMQ: Antiepileptic drugs:


A. Don’t use in absence seizure --- Carbamazibine
B. Cause oral ulcers --- Carbamazibine
C. only medication used in infantile spasm with tuberous sclerosis
Vigabatrine

3. Ct head post road traffic accident


Atrophy of brain/oedema

4. Girl with wide base gait, +Romberge ,absent ankle and knee jerks,
extensor planter response ,normal vision , + systolic murmur, mother has
coelic disease on gluten free diet
a. Vit b12 def.???
b. Fredrich ataxia. ??

59
Neurology

5. young girl 3 episode of fell down in bathroom unconscious without jerky


movement, what make u think this true seizure from history of the girl ?

• the place : ‫ه بتمثل‬


‫مش هتقع يف الحمام لو ي‬
• closing eyes and don’t heart herself— psychic
• prolactin will be high in true convulsions

6. weakness of left upper limb and lt side of the face which resolve
spontaniosly
a. Hemiplegic migraine.
b. Mutiple sclerosis.
c. Crebro-Vascular Accedant

Jan 2013
1. 3-years old girl came with her foster mother generalized tonic clonic
convulsions. Or vacant episodes, ttt?
a. sodium valproate??
b. topiramate
c. carbamazepine
d. no treatment
e. lamotrigine ??

60
Developmental
Assessment

61
Developmental assessment

Sept 2020
1. 4 years old boy, he speaks 2- word sentences, only parents can
understand him, he has 3 other siblings and living in a small apartment
what to do?

a. Speech and language therapy


b. Refer to paediatric audiology (ENT do not remember) for assessment
of hearing

2. Q asking about the developmental age of child who can draw 1st line and
can’t draw the 2nd line!

a. 4.5 – 5.5
b. 5.5 – 6.5
c. 3.5 – 4.5
d. 6.5 – 7.5
e. 2.5-3.5

May 2019
1. 5-yrs patient had clumpy very bad hand writing his behavioural like his
father when he was child

a. hungtingon chorea
b. dyspraxia
c. dyslexia

62
Developmental assessment

May 2018
1. 4-years-old with delay speech, his parent was smoker, the father
understand what he is telling

a. hearing assessment
b. Refer to ENT surgery
c. Refer to psychotherapy
d. Refer to speech and language therapy

Jan 2017
1. question about deafness in 2 year old child . . Asks for 2 answers

a. speech referal
b. hearing aids

May 2015
1. a child with delayed speech. Smoking parents. What to do next..

a. hearing assessment
b. refer to SALT – speech and language therapist
c. refer to social services
d. refer to ENT surgeons

63
Endocrine

64
Endocrine

Sept 2021
1. 8 years old with height on 98th centile, weight 91st centile, scenario with
pubic and axillary hair, B/P 125/78, but testes size= 3 ML. what is the
diagnosis?
a. Hypothalamic hamartoma
b. Premature adrenarche
c. CAH
d. Ideopathic precocious puberty

2. 12 yr. old girl presented with pigmentation of lips and gum, she has
moderate dehydration, vomiting and headache. K: 5.4, Na: 122,
urine Na: 20, BP: 85/40. Ask about investigation?
a. ACTH stimulation test (Synacthen)
b. Dexamethasone suppression test
c. Aldosterone and rennin
d. Brain MRI

3. 13.5 years old boy complains of asthma and short stature among his
beers. No signs of puberty. Height decreased in last year from 9th to 4th
centile, bone age 11 year, mother reached menarche at 15, father start
shaving at 16 year. Mid parental height is 25th centile. What is the next
action?
a. Reassess after 3 months
b. Growth hormone stimulating test.
c. Karyotype
d. TFT
e. ACTH

4. 3 weeks old baby, FTT, K: 6.6, Na: 128, normal BP, with poor feeding.
Asking about diagnostic investigation for this case?
a. Aldosterone and renin
b. 17 hydroxyprogesterone

65
Endocrine

5. Female Obese BMI 32 has amenorrhea for 3month. O/E: striae in trunk
and thigh, brown discoloration neck and axilla, increasing weight for 2 or
3 years. She has good school performance and she follow with ditetion
with no response with normal B/P. Dx?
a. PCO
b. Cushing
c. Hypothyroidism
d. Hypertension

May 2021
1. A 1-month-old female presents with diarrhoea and failure to gain weight
with Na of 126, K of 6.6 and cortisol of 900(normal 500-700). She has
normal genitalia. What is the diagnosis?
a. DI
b. CAH
c. Pseudo hypoaldosteronism

2. A 3-year-old girl with unilateral breast enlargement. The mother thinks it


was present since birth and that the girl was one of the taller girls in the
nursery. What is the diagnosis?

a. Premature thelarche
b. Premature adrenarche
c. B-HCG secreting tumor
d. CAH
e. Precoscious Puberty

3. A 10-year-old girl with a 2-year history of headache came for short stature.
Her weight increased from the 50th to the 75th centile and her height
decreased from the 75th to the 50th centile. Her foot wear size did not
change for 2 years. What is the diagnosis

a. Cushing syndrome
b. Hypothyroidism
c. Posterior fossa tumor
d. Craniopharyngioma

66
Endocrine

4. A 6-week-old baby of weight of 4.8 kg complained of vomiting and


irritability before feeds.RBS:1.7 (corrected 0.8). He was managed by NGT
feeding then discharged. The next day the mother last fed him at 10 pm
and the next feed was supposed to be at 6:30 am but before that he
developed an attack of convulsions, so he was brought back and was
found to be drowsy but arousable. IV canula was inserted and he was
investigated. His RBS was 0.9 What is the next immediate management?
a. Dextrose oral gel
b. 10 ml of 10% dextrose
c. 10 ml of 20% dextrose
d. I.M glucagon
e. NGT feeding

5. A 10-year-old boy was diagnosed of anuresis, on Desmopressin. He had a


history of lethargy and now presents with convulsions. Investigations:
Na: 121, K:5.9 , Glucose: 3.9 (normal 4-7) , Urine osmolarity 218 (low) ,
Serum osmolarity 275 (low) , What is the diagnosis?
a. SIADH
b. Psychogenic polydipsia
c. Salt wasting
d. Addisonian crisis
e. Nephrogenic DI

6. A neonate with hypoglycaemia, micro penis and prolonged jaundice. What


is the diagnosis?
a. Hypopituitarism
b. Congenital hypothyroidism
c. Prader-Willi syndrome
d.
7.
A. A 9-year-old girl comes with goiter. TSH was high, T4 was normal.
What investigation should be done first?
a. Thyroid antibodies
b. Thyroid us

67
Endocrine

B. Then the patient was diagnosed and started on thyroxine. Which


test is best for follow up?
a. TSH
b. T4
c. Bone age
d. Height and weight
Jan 2021
1. Adolescent referred from dental clinic with photo of gum
hyperpigmentation, asking about the cause.
a. Addison ds
b. phenytoin toxicity
c. carotenemia
d. haemochromatosis

2. A girl on chemotherapy, did pelvic radiation also, LH ↑ , FSH ↑ ,


estradiol ↓ , height normal. Diagnosis?
a. 1ry ovarian failure
b. Turner syndrome

3. EMQ:
A. Hypoglycemia
B. Hyperglycemia
C. Hypokalemia
D. Hyperkalemia
E. Hypocalcemia
F. Hypercalcemia
G. Hypophosphatemia

A. 14 years old girl with anorexia nervosa BMI 13 during ECG become
drowsy and sweaty. Hypoglycemia
B. 14 years old girl with anorexia nervosa BMI 13 her ECG with U wave.
Hypokalemia
C. 14years old girl with anorexia nervosa BMI13 came with Tetany and
prolonged QT.
Hypocalcaemia

68
Endocrine

4. case of Short (height on 2nd centile) and obese girl, her blood pressure
was on 91 centile and has normal intelligence. What would it be?

a. simple obesity
b. Cushing syndrome
c. Hypothyroidism
d. Normal variant

Sept 2020
1. EMQ Hypoglycaemia:
MCAD
CAH
Hyperinsulinemia
GSD
Galactosemia
A. A new born 4.2 kg, very low RBS, no ketones, ammonia 60
hyperinsulinemia
B. A newborn boy, 6 days, come with vomiting, hypoglycaemia and
Hyponatremia 121, ketones trace, RBS 1.1, lactate 0.8 CAH
C. 2-month baby, his mother came with the baby from Morocco 2 weeks
ago,the father was studying in UK. Hypoglycemia 1.8 with coryzal
illness, no ketones
MCAD

2. Child with CAH on good treatment parents asking about his growth and
final height?
a. Normal growth but final HT lower than his parents
b. Normal growth but final HT higher than his parents
c. Accelerated growth and final HT more than his parents
d. Delayed growth with final height was higher than his parents

69
Endocrine

3. EMQ
A. CAH
B. MRI
C. Karyotyping
D. Testosterone
E. LH and FSH
F. Urinary catecholamine
G. Brain MRI
H. Karyotyping
I. Testosterone
J. LH and FSH
K. Urinary steroid
What investigations to reach diagnosis?
A. 7 yr. Neurofibromatosis 1 had testicular volume 12 ml, Pubic hair
Brain MRI
B. Tall 5 yr. child with testicular volume 3 ml, Na 122, Public
Urinary Steroids
C. Tall 15 y old with testes 6ml, excluded from school with moderate
gynecomastia
Karyotyping

4. 2 years old Pakistani boy, faltering growth, developmental delay, he is


cruising and stands with support. PTH 41, Alkp 1089, Ca. 2.18 and
Phosphate level was normal dx?
a. Primary hyperparathyroidism
b. Rickets??
c. Gaucher

70
Endocrine

JAN 2020
1. 13.5 years old boy complain asthma and short stature among his beers.
No signs of puberty. Height decrease in last year from 9th to 4th centile
bone age 11-year, mother reached menarche at 15, father start shaving at
16 year...mid parental height at 25th centile next action
a. reassesses after 3 months
b. growth hormone stimulating test.
c. Karyotype
d. TFT
e. ACTH
2. Female Obese BMI 32, amenorrhea for 3month +striae in trunk and thigh,
brown discoloration neck and axilla, Increasing weight for 2 or 3 years,
good school performance follow with dietician with no response with
normal BP
a. PCO
b. Cushing
c. Hypothyroidism
d. hypertension

3. Boy 10-y, post appendectomy high k (6.2), low Na (128), urea 56, creatine
38 normal BB, low glucose (2.9), diagnosis?
a. Sepsis
b. CAH
c. SIAD
d. Cushing
e. Hypoadrenalism

4. 3-weeks, birth weight 3.6kg now 3.05, feeding good, doesn’t look
dehydrated, investigations Na 128, K 6.6 ,urea 7.6 urine Na 129 what the
next investigation?
a. 17 hydroxyprogeston
b. Amino acid in urine
c. Aldosteron
d. Plasma amino acid

71
Endocrine

5. like this question


17-months, investigation hours fasting glucose, insulin ,HCO3
12hr 3.9 3.3 21
15hr 3.7 3.8 19
24 hr 1.6 0.5 11.5
2 possible diagnoses?
a. Neuroblastoma
b. Beckwith
c. ketotic hypoglycemia
d. galactosemia
e. DM
f. RTA

Sept 2019
1. Turner syndrome on growth hormone came with symptom of IIH LP done
pressure high, normal MRI, what to do?
a. discontinues growth hormone
b. LP

2. one question about patient with polyurea & polydipsia and high RBS
ask about investigation for acute management?
a. blood gas
b. lab glucose level
c. hba1c
d. GTT

3. picture of male 8-yrs external genitalia with pubic hair and penile
enlargement high BP ,, but testes size= 3
a. CAH
Small testis – NOT tumor
b. constitutional growth
c. testicular tumour

72
Endocrine

4. A 12-week-old baby girl was referred with poor feeding, poor weight gain
and a history of previous urinary tract infection and hyponatraemia.
On examination there were no abnormal findings. Initial investigations
showed a serum sodium of 125 mmol/L, potassium 6.2 mmol/L,
bicarbonate 17 mmol/L, urea 7.0 mmol/L, osmolality 271 mOsm/kg.
She has a normal urine microscopy and a normal abdominal and pelvic
ultrasound.Additional investigations showed a serum cortisol of
794 nmol/L, a normal 17-OH progesterone (6 nmol/L) and a raised
plasma renin (35 pmol/mL/hour; normal 2.8–4.5 pmol/mL/hour) and
aldosterone (38,000 pmol/L; normal 100–450 pmol/L).
What is the most likely diagnosis?
a. 21-hydroxylase deficient CAH
b. Congenital adrenal hypoplasia
c. Diabetes insipidus
d. Hypopituitarism
e. Pseudo-hypo-aldosteronism
(sop page 518 please read explanation)

5. one case of goitre with TSH high T4 normal


A. ask about investigation to do
a. thyroid scan
b. thyroid antibody
c. THYROID U/S
d. T3

B. she started thyroxin then ask about which test can help in efficiency of
treatment (follow up)?
a. TSH
b. T4
c. bone age
d. Height and weight

73
Endocrine

May 2019
1. neonate with goitre and hypothyroidism. Common cause?
a. dyshermogenesis

2. picture of girl with bilateral breast enlargement, what is the investigation


a. No investigation send home
b. Karyotyping
c. serum oestrogen
d. pelvic u/s

3. EMQ: hypoglycaemia 3 scenarios


a. FOAD
b. Panhypopitutarism
c. Glactoceamia
d. Hypothyroidism
e. Hyperinsulism
f. Non ketotic hypoglycemia with low free fatty acids
g. glycogen storage disease

A. ffa low,serum ketone low, RBS low FaOD


B. ffa high,serum ketone high, RBS low , GH & cortisol both high
ketotic hypoglycemia
C. relatively small penis .GH & cortisol low ...RBS low Panhypopitutrism

4. hyponatremia hypoglycaemia low osmolarity serum and urine , k 5.9


a. addison
b. salt wasting
c. nephrogenic diabetis insipidus
d. SIADH

74
Endocrine

Jan 2019
1. (Table) A boy with short stature had the following results after
administration of clonidine, LHRH stimulation, Growth hormone was
normal, prolactin slightly high, TSH normal & T 4 LOW,
cortisol (not doubled enough) diagnosis?
a. Growth hormone deficiency
b. Hashimoto’s thyroiditis.
c. Central hypothyroidism
d. Panhypopituitarism

2. A girl 6-yrs of precocious puberty, can’t remember the age, no breast


scanty pubic hair, Hypertension, normal Na, decreased cortisol,
investigation?
a. Urinary steroids
b. MRI brain
c. bone age
d. pelvic u/s

3. 12 years old boy, short ,delayed puberty , A1 , P1, 3 ml testis ,3yrs


growing through same sentile (10th) & mid parentral hight bet 25th to 50
no family history of delayed puberty ?
Constitutional delay of growth and puberty

4. A girl on chemotherapy, did pelvic radiation also, LH ↑, FSH ↑ , estradiol


↓ , height normal:
a. 1ry ovarian failure
b. Turner

5. A girl 14- yrs tall and slim, who developed neurological symptoms and
started pumping onto furniture, tanner stages all 1?
a. pituitary tumour
b. Kalman syndrome
c. Klinefelter syndrome

75
Endocrine

Sept 2018
1.X-ray: fractured radius & ulna, cupping,frying,
increase,joint space
a. rickets
b. osteogenesis imperfecta

2. EMQS
A. Boy with moderate gynecomastia, exclude from school , parents worry
about his tall stature karyotype (Klinefelter)
B. Pseudo precocious puberty urinary steroid profile (CAH)
C. True precocious puberty MRI brain ( pituitary tumor)

3. EMQS
a. breast feed failure
b. CAH
c. Galactosemia
d. GSD
e. Hyperinsulinsm
f. Homocysteinuria
g. MCAD
h. organic acidemia
i. prader willi
j. UCD
A. Female ,weight 4.2 kg , 24 hr of age seizer , glucose 0.1, ketone – ve
hyperinsulinmic hypoglycemia
B. 2-month boy , convulsion after viral illness , from Saudi arabia , glucose 1.1
MCAD
C. Male weight 3.1 kg, 6 days poor feed , vomit , glucose 1.8 , NA 125 , ketone
trace CAH

76
Endocrine

4. 13-year-old girl, her mother had type 1 DM, and her grandmother had
thyroid problem. she fainted in school and have abdominal pain, Na 125,
K 5.5, Bp 90/56, Pulse 68
Addison

May 2018
1. 14 years old with type 1 DM increase requirement of insulin with normal
height and weight loss
Thyrotoxicosis (explain both weight loss and increase requirement)

2. 7-years-old recently diagnosed CAH and parent concern about height?


Normal growth & final height below parents

3. 8-weeks old RBS 1 , and small external genitalia with hypotonia , jaundice
difficult feeding
a. prader willi syndrome
b. Hypopituitarism
c. Hypothyroidism
d. CAH

4. picture of external genitalia of 8 years old child boy with hair growth,
testicular volume 3 ml and excessive weight and height increase BP= high?
a. CAH
b. Precocious puberty
c. Hypothalamic tumor
d. Testicular tumor
Could be cushing also

77
Endocrine

5. 13-years-old with delay bone age and short stature his bone age is 9.5
years, with another normal parameter
A. Diagnosis?
constitutional delay
B. What is your action?
monitor for 6 months

6. 13-years-old with thyrotoxicosis has congested throat on carbemazole


treatment
a. EBV serology
b. CBC and differential *>>SE of carbemazole is agranulocytosis

Jan 2018
1. picture of isolated breast development Sample paper
Reassure

2. 7-yrs weight above 98th, height 132, BMI 23.6 product of insemination his
mom and dad and adopted sister all are slim he is dyslexic but in
mainstream school BP 90th centile you are seeing him now after 6 months
having same weight and height bone age is 1 year advanced
what is his diagnosis?
a. GH tumour
b. Laurance moon beidle
c. Prader willi
d. Cushing
e. familial obesity
f. simple obesity

78
Endocrine

3. Repeated Q I think including management of high Bp ,high K


A. possible diagnosis?
Psudohypoaldosteronism
B. treatment?
IV labetalol

4. Diabetic ... uncontrolled last 3 months with low HbA1c ... labs showed
hyponatremia with hyperkalemia.
adrenal insufficiency
5. An image of a girl with Goitre + history of hyperthyroidism symptoms
(fatigue and heat intolerance I think).... most appropriate ttt ?
a. crabimazol ??
b. propranolol

6. EMQ:
a. acquired hypopitutarisim
b. hypothyrodisim
c. less nutrition
d. Turner
e. Prader willi
f. GH def
g. conestitutional

A. child had craniopharyngioma received ttt height on 2nd centile weight on


9th acquird hypo-pitutarism

B. known asthmatc mild only on salbutamol growth velocity 5 cm per year

C. 4-yrs had cleft lip and palate operated when was 1yr lethargic and less
energy weight 50 centle height 0.4

79
Endocrine

7. 17-months investigated for convulsion, Choose 2 options?


a. Neuroblastoma
b. Beckwithwide
c. undetected DM
d. kenotic hypoglycaemia
e. galactosemia?
f. craniopharyngioma
g. RTA
h. Hypopituitarism

8. Asian baby had convulsions, red throat and tympanic membrane was well
on examination labs showed low calcium what to do next?
a. X ray wrist
b. Blood c/s
c. LP
d. EEG

Sept 2017
1. Girl with headache 2-years obesity, short, delayed bone age?
a. Cortisol
DD of delayed bone age:
b. Neuro-imaging??
1. excess cortisol
c. Observe
2. Hypothyroidism
d. thyroid test 3. Gh deficiney

2. 10-days baby with 5-days, thirsty, Na low pot high


a. 21 OH
b. CAH

3. Newborn to mother known graves with positive ab, TFT TSH 11 , T 4 45


admit and repeat TSH after 48hr

80
Endocrine

May 2017
1. mother with diplopia baby poor suck
Myasthenia gravis

2. hiccups … Non-ketotic hyperglycinaemia

3. counselling parent with child has Congenital Adrenal Hyperplasia


2 answers
a. increase in growth?
b. then decrease final adult height

4. repeated question from previous exam hyper thyroid the cause is viral
transient thyroiditis asking the management?
a. Propranolol
b. Carbamizole

Jan 2017
1. Short. . Obese child . . Blood pressure high normal
a. Normal
b. cushing

2. a girl with precious puberty. . Investigation ?


urinary steroid profile

3. TSH. F4. GOITER . . Asks about diagnosis?


Dyshormogenesis

81
Endocrine

May 2016
1. Dm type 1 give recently high insulin requirements 1-1.5 u/kg and history of
weight loss what investigations?
a. IgA transe tissue glutaminase
b. inflammatory bowel disease
c. Thyrotoxicosis

2.EMQ:
A. Boy with gynecomastia & signs of puberty:
Normal variant during puberty --- reassure
B. Tall boy with gynecomastia & no signs of puberty, teased at school
Klinefelter--- Karyotyping

Jan 2016
1. Pt with T4 12, TSH 13
hashimoto (euthyroid or hypo or hyper)
A. what is investigation?
a. Thyroid Ab
b. T3
c. Thyroid US
d. Brain MRI
B. She started on thyroxin, what is best monitoring for ttt:
a. T4
b. TSH
c. Thyroid Ab

2. pt short stature with ph 0.8 normal ca and PTH and ALP :


a. X linked Hypophosphatimic Ricket
b. Nutritional ricke

82
Endocrine

3. Pic of Pt 19 months with bilateral breast


enlargement bone age 2 y
No other abnormality what investigation:
a. Oestrogen level ?
b. LH,FSH
c. Brain imaging
d. TSH
e. pelvic US

4. Neonate with poor feeding low glucose and micropenis, jaundice an


Lethargic with low BP
a. Hypothyroidsm
b. Sepsis
c. Hypopituitarism
d. CAH

5. Child with vomiting and dark skin creases on the palm


Addison disease

83
Endocrine

6. Child known CAH on good treatment, parent ask what his height will be:
a. Decrease growth and final height as parents
b. Rapid growth and final height below parents
c. Decrease growth and final height above parents

May 2015
1. A child with accelerated bone age,high BM and obese..diagnosis?
a. simple obesity
b. Cushing’s

2. A preterm child following 0.4th centile.what would u do if still fails to


catch up growth at 4 years despite of adequate diet input..
a. refer to endocrinologist
b. refer to geneticist If ask for one option – endocrine
if 2 options – endo & genetics
c. refer to social services
d. refer to dietician

3. A girl with subclinical hyperthyroidism [lab T3-T4 normal. TSH low]


What would u do?
a. gives propranolol
b. give carbimazole, iodine ttt
c. if no symptoms no ttt just follow up

Sept 2014
1.Girl with obesity, delayed bone age? Two investigations?
Thyroid tests
Neuro-imaging

84
Endocrine

2.Newborn to mother known graves with positive ab


TFT: TSH—11, T4—45

3.10 days baby with 5 days, thirsty


Na -- low, K --high, RBS—low
21-OH-CAH

Jan 2013
1. 6 years old girl, with family history;tall& obese both father and mother
bring her bcz she get signs of puberty in form of : breast
development : stage 3 pubic hair : stage 2 but no axillary hair she is
otherwise very well , what is the cause for this?
a. premature thelarche
b. premature menarche
c. idiopathic true precocious puberty

85
Diabetes
Mellitus

86
Diabetes Meletus

Sept 2021
1. 10 days old neonate presented with vomiting and clinically dehydrated.
Body weight decreased from 3.7 kg to 3 kg. Labs showed PH 7.2, Bicarb
11, Na 156, and RBS 23.1. What is the Dx?
a. Transient neonatal DM
b. Hypernatremic dehydration
c. Nephrogenic DI
d. Cranial DI
(Neonatal DM not mentioned)

2. Scenario about 12 years old diabetic girl missed 2 clinic came with her
aunt because mother is ill he did not bring her glucometer lost 2 kg and
denied any issues regarding her diabetes and said reading was about 4-9
mg/dl (target 6-10) you did hbA1C came 58 (NL: 42-48) nurses know her
and notice that she is quite anxious and withdrawn. What is the cause of
her condition?
a. Poor adherent
b. Eating disorder
c. Thyrotoxicosis
d. Puberty
e. Depression

3. 10y Child has Glucose in urine, protein+, mother has DM type 2, BMI 75
centile Investigations: HbA1c 7.5, Sugar 7.4. What is the cause for glucose
in urine?
a. DM 1
b. DM2
c. MODY
d. Renal glucosuria

87
Diabetes Meletus

4. 12-year-old Asian girl has DM1 with poor control despite increasing
insulin dose to 1.5 U/kg. She has weight loss and recurrent abdominal
pain and distension. Investigations HB 10.8 (11.5- 14), MCV 66 (low),
MCH normal, HbA1C 88 (high). asking about investigation lead to
diagnosis? (This is the exact scenario)
a. Hb electrophoresis
b. Antitissue transglutaminase
c. ESR
d. Ferritin

5. A 13-year-old obese female, with BMI 38. Her HBA1c of was and she was
included in a weight management program for 2 months without
improvement. What will you prescribe for her?
a. Metformin
b. Sulfonylurea
c. SC insulin
d. No treatment

May 2021
1. You are the senior registrar. A mother of a girl with type 1 Diabetes
mellites on insulin pump called you, she has just changed the insulin pump
cartridge the night before. This morning (Sunday 9 am) blood sugar was 21
and serum ketone 2.1. What should you advise her to do?

a. Call the ambulance to take her daughter to hospital


b. Give a correction dose of short acting insulin subcutaneously
c. Give a correction dose by pump and recheck sugar after 4 hours
d. Give correction of 10% of total daily dose

88
Diabetes Meletus

2. A girl with DKA comes with initial RBS of 20, serum ketone = 6.4 and
urine ketone = 4+. She was managed as per DKA protocol with insulin
0.05unit/kg/hr. After 12h of treatment the RBS was 10, serum ketone
2and urine ketone was still 4+ what is the next best course?

a. A fluid bolus
b. increase insulin to 0.1unit/kg/hr
c. Continue current management
d. Increase infusion drip
e. Insulin bolus

3. A 13-year-old girl with a 3-year history of type I diabetes presents to the


clinic with poor control and a history of weight loss for 3 months. During
this time, her control has gradually deteriorated without obvious reason.
Her total insulin dose was progressively increasing to over 60 units daily
and she has been changed on to a 4 injection per day regimen. Her record
book shows few blood results and most are within the normal range. She
complains of thirst, nocturia and mild intermittent abdominal pain. Her
weight has fallen from 45 kg to 40kg. She is the second of three daughters
of a stable family with no financial worries. She had been successful and
happy at school, but recent events have caused school absences. On
examination she is tall, (height is 162cm between the 75th –90th centile)
and thin, (weight 40 kg <25th centile). General examination is otherwise
unremarkable. Her injection sites are not hypertrophied. She is at puberty
stage 2. What is the most likely because of her recent weight loss?
(Select one answer only) same as sample paper
a. Anorexia nervosa
b. Coeliac disease
c. Hyperthyroidism
d. Occult urinary tract infection
e. Poor compliance with diet
f. poor compliance

89
Diabetes Meletus

4. A 15-year-old patient was diagnosed with type 1 DM one year ago and
urine analysis showed protein 1+. You are going to prescribe Enalapril.
What precautions about investigations would you discuss with the
parents? (Two options)
a. Urea and electrolytes 2 days later
b. Blood pressure measurement

Jan 2021
1. 6 years old diabetic came with his parents to the diabetes clinic, His
parents want continuous glucose monitoring for him What’s the indication
of continuous glucose monitoring from the following?

a. recurrent DKA
b. Recurrent hypoglycaemia
c. needle phobia
d. poor adherence to treatment
e. parents’ preference

2. Picture, Diabetic girl has history of trauma during playing football came
with this lesion.

necrobiosis lipoidica

90
Diabetes Meletus

3. 12 years girl came to ER with her sister. She is Diabetic patient on insulin
pump present with hypoglycemia with tonic clonic seizure, blood glucose
1.2, First action?
a. Stop Insulin pump
b. Give iv glucose 10% bolus
c. Buccal midazolam
d. IM glucagon

4. 4 months old with early morning hypoglycemia who had a history of


Transient neonatal hypoglycemia corrected by feed... investigation?
a. Insulin
b. C peptide
c. White cell enzyme

5. Obese girl with type 2 DM, not responding to lifestyle modifications,


what to do?
a. Insulin
b. Metformin

Sept 2020
1. RBS chart for DM patient on basal bolus regimen, same as AKP case 38,
page 153. What to do? (High readings in the mornings most of days)
a. Increase long-acting evening dose 100 diabetes
b. Decrease short acting dose
c. Increase short acting before breakfast
d. Increase short acting before dinner
e. Check adherence

2. 10 years Cystic Fibrosis patient presented with loss of wt. and


deteriorated lung function. Negative sputum culture what to do?
a. OGTT
b. HA1C

91
Diabetes Meletus

3. 14 years old boy has DM I at follow up visit HA1C 58 increased to 75 and


he told the doctors that he does not want to take his insulin because he
feels different from other students. He was using Lantus and short acting
injections. What to do? 1 option
a. Start him on insulin pump
b. Give 2 doses daily insulin ??
c. Let the school nurse give him insulin
d. Refer to psychologist team??
e. Refer to dietitian

4. Case of girl high blood sugar with positive family history of DM, MODY,
she was diagnosed as glucokinase deficiency. What is her treatment?
a. No treatment
b. Insulin
c. Metformin

5. A child with type 1 DM on insulin pump with diarrhea vomiting, she


checks her glucose reading regularly, no ketones, she tolerates oral fluids
but does not eat, what to do?
a. Continue basal insulin and bolus if hyperglycaemia only
b. Stop pump and give bolus with IV 5% dextrose and 0.9 saline
c. Stop insulin and restart if he takes oral
d. Continue insulin basal and give iv 5% dextrose and 0.9 saline

6. Diabetic patient found by his father unconscious at home, RBS 2 what is


the immediate action to do?
a. Buccal dextrose gel
b. IM glucagon
c. Give juice
d. Dextrose tablets
e. Call ambulance and help

92
Diabetes Meletus

7. 10 years girl type 1 DM, parents mentioned that she started to have
regular menstruation 6 months ago, Tanner stage 2 for pubic hair and
high HA1C, injection sites were good! , she developed 2 DKA and many
times hypoglycaemia. One option
What is the cause of uncontrolled DM? ‫علية إختالف شديد‬
a. Fabricated menstruation
b. Poor adherence to diet
c. Puberty and resistance to insulin 100 % endocrine
d. Eating disorder
e. Unstable diabetes Mellitus
f. Insulin Omission 100% DM

Jan 2020
1. known diabetic 14y Patient on insulin pump previously was well
developed frequent daytime hypoglycemia.
a. Alcohol
b. Increase exercise?
c. improper administration of insulin
d. Addison
e. Celiac

2. Patient known diabetic with low BG 3,2, asymptomatic and alert at night,
her mother called you what to do ?
a. 4 glucose tablet 100%
b. repeat blood sugar after 30 min
c. IM glucagon
d. 200ml milk
3. 10y Child has Glucose in urine, protein+, mother DM type 2 , BMI 75
centile Investigations: HbA1c 7.5 ? Sugar 7.4?
What is the cause for glucose in urine?
a. DM 1
b. DM2
c. MODY
d. renal glucosuria

93
Diabetes Meletus

4. picture of
a. acanthosis nigricans
b. Cushing
c. freckling
d. contact dermatitis
e. Addison

Sept 2019
1. picture of axilla ask about associated
with, need one option
a. neurofibromatosis
b. acanthosis nigricans
c. tuber sclerosis
d. Addison

2. a case of DKA started on protocol initial RBS 34, ketone +6 urine ketone
+4 ,now RBS 10 ketone +1.3 urine ketone +4 ,,baby was well ,what to do?
a. continue same medication
b. work up for sepsis

3. which of the following feature most suggestive of DM type 2


need 2 options
a. elevated c peptide
b. HBA1C (high in both types)
c. Vitiligo (associated with type 1 autoimmune)
acansosis nigricans

94
Diabetes Meletus

4. question about obese (bmi 31) child type 2 DM high HBAIC 58 started
on life style no improvement what to do?
a. Metformin
b. Sulfonylurea
c. Insulin

May 2019
1. 10-years-old girl diagnosis of DM for 3 year .... she had attacks of sever
hypoglycaemia insulin requirement change from 0.8. -- to 0.3. iu/kg/day
family go to new house share room with brother mother have
hypothyroidism taking thyroxin pt had loss of weight
What causes of her changing insulin?
a. unaware about hypoglycaemia ??
b. honeymoon period ??
c. puberty
d. hypothyroidism

2. picture of Necrobiosis lipodica

3. DKA not shocked k with in normal range, Ask about fluid management?
a. 0.9 saline with k 40 mmol/l
b. 0.9 saline k 20 mmol/l
c. 0.9 saline only
d. bollus 10ml/kg n/s
e. bollus 20ml/kg n/s

95
Diabetes Meletus

Jan 2019
1. DKA patient, blood glucose 8, PH 7.24, 3% dehydration, ttt:
a. deficit and maintenance fluids with kcl
b. saline deficit and maintenance
c. Insulin

2. Glucose chart with high readings mainly in the morning, similar to the
one in the AKP cases book, they attend appointments regularly, best
action HBA1c 58?
a. Increase lantus dose
b. Check compliance

3. 14 y/o diabetic since 12 y/o, injection sites are not hypertrophied,


recurrent DkA, lost weight, Hb A1C :
Poor adherence

4. A known diabetic on insulin pump, for elective small surgery, he ran out
of cartridges, his blood glucose is 7.5, it’s 4 am and the diabetic team
won’t be available until 8 am, what is the best action?
a. give SC short acting insulin
b. remove the pump
c. call pharmacy and request cartridges
d. wait for diabetic team 8 am
e. ive long acting
f. iv insulin

Sept 2018
1. Case of DKA , shocked, fluids to give ?
a. NS 20 ml/kg
b. NS 10 ml/kg
c. Ns 10ml/kg with k
d. NS 20ml / kg with k

96
Diabetes Meletus

2. 12-yrs-old Cystic fibrosis, loss weight, lethargy, deteriorate lung function


sputum no organism , no improve on 2 weeks iv antibiotics , next ?
a. OGTT
b. thyroid function
c. MRI chest
d. percutaneous gastrotomy for feed
e. HbA1c
3. Three risk factors for DM type 2 ?
a. acanthosis nigra
b. relative parent with autoimmune thyroid
c. obesity
d. Asian ethnicity
e. Consanguinity

4. obese adolescent with high HA1C = 7.2, acanthosis nigricans not respond
to lifestyle changes, treatment?
a. Metformin
b. Sulfonylurea

May 2018
1. mother of 4 months old child bring the child at 7 pm, the baby is blue, the
main things was RBS 14.4mmol/l ,the asked the reason ?
Stress hyperglycaemia

If he mentioned infection in Qs--- stress hyper glycemia

Criteria to diagnose neonatal hyperglycaemia:

❖ 2 blood sugars are ≥14 on 2 occasions measured ≥2 hr


apart
Or

❖ blood sugars ≥12 on 2 occasions measured ≥2 hr apart


with evidence of significant glycosuria ++ on the urine
dip stick

97
Diabetes Meletus

2. 12-years-old with HNF1 gene mutation (MODY3 ) asymptomatic has


strong family history of type 1DM, his RBS 10.5mmol/l ,BMI=19
asked what to give him treatment?
sulphonylurea
If mention glucokinase deficiency >> no ttt

3. 6-years-old on insulin pump and started vomiting, hyperglycaemia no


ketones he tolerate oral fluid but cannot eat what to do ?
give basal and bolus insulin pump

4. picture of a child with rash on his shins with polyuria,


polydipsia and weight loss
a. necrobiosis lipodica
b. erythema nodosum

5. a child with DKA on treatment insulin infusion everything’s become ok,


but still the patient has 4+ ketone and serum ketone 6 they asked what
to do?
a. continues same treatment ??
b. Check other causes of ketonemia
c. Increase the rate
d. Give bolus normal saline

98
Diabetes Meletus

6. a child with mark in neck


a. acanthosis nigrans
b. Filekering
c. Café aue lait
d. Fungal infection

Jan 2018
1. picture of necrobiosis lipodica

2. Picture of axilla dark, what is the cause?


a. NF
b. Tuberous sclerosiss
c. Addison’s
d. MacCune Albright
e. Insulin resistance

3. EMQ but can not remember all options:


a. long acting insulin
b. rapid acting insulin twice daily
c. Metformin
d. Sulponylurease
e. rapid acting three times daily
f. Life style
g. No treatment
A. BMI 32 HbAIC 44 (normal 42) life style
B. BMI 18 Glucokinase def in the family no treatment
C. BMI 35 HbA1C 59 he is part of life style changing program METFORMIN

99
Diabetes Meletus

4. Diabetic ... uncontrolled last 3 months with low HbA1c ... labs showed
hyponatremia with hyperkalemia.
adrenal insufficiency

5. 14-yrs DM with recent weight loss taking insulin blood sugar 4.6 ketone
1.5 in blood hb 120 wbcs 7.5 Na 126 K7 urea 8 creatnine 98 CRP 4 PH 7.3
PCO 4 Bicarb 15 urine showed 2+ ketones and no glucose what is your
diagnosis?
a. adrenal crisis
b. DKA
c. renal failure
d. coeliac
e. substance misuse

Sept 2017
1. 14-yr with 3rd DKA in 2 months ... Nonadherent
If also with recurrent hypos—brittle diabetes

May 2017
1. DM teenager high A1C losing weight
a. antibodies to insulin
b. poor compliance

2. Girl diabetic, recent hypoglycaemia’s and decreasing insulin, moved to a


new house where she is sharing room with brother
a. Honeymoon period
b. Anxiety
c. Hypoglycaemia unawareness
d. Taking wrong dose

100
Diabetes Meletus

3. patient has DM on insulin pump admitted in word for elective surgery


but the insulin pump finished and blood sugar early morning was ok what
to do Give rapid insulin???
a. Ask for insulin to his pump from hospital pharmacy
b. DM team refer
c. Iv insulin

Jan 2017
1. cystic fibrosis case. . Don't remember the scenario . . Guess was diabetic.
. Asks about most appropriate investigation
Oral glucose tolerance test

2.a child with insulin pump not correctly placed. . High sugar. . Mom calls
you and asks u what to do?
a. call Ambulance and get the child to hospital
b. SC insulin injection.

May 2016
1. Picture of DM type 1 with lesion in her shins
a. necrobiosis lipodica
b. erythema nodosum

2. Dm type 1 give recently high insulin requirements 1-1.5 u/kg and history of
weight loss what investigations?
d. IgA transe tissue glutaminase
e. inflammatory bowel disease
f. Thyrotoxicosis

101
Diabetes Meletus

3. Picture of axilla:
Acanthosis nigricans

Jan 2016
1. Pic of Diabetic leg show ulceration in the chin of tibia:
a. Erythema nodosum
b. Necrobiosis lipoidica

Sept 2014
1.14-yr with 3rd DKA in 2-month
Nonadherent

102
Haematology

103
Hematology

Sept 2021
1. EMQ
A. NAI
B. HEMOPHILIA
C. ITP
D. TTP
E. ALL
F. WISCKOT ALDRICH SYNDROME
G. DIC
H. HSP
A. 4 months old boy had bruises after URTI and during investigation he had
excessive bleeding from puncture site please see lab attached below:
CBC = Hb. 11, WBC 12, PLT 152
coag. Profile = PT 12, APTT 82, INR 1.2
HEMOPHILIA

B. 5 years old boy in foster care presented with purple coloured bruising on
lower limb (bone dependant) multiple in size and features there was
ankle swelling he cannot express the cause of these bruising, normal
neurological and systemic examination, Coag. Profile = PT 14, APTT 26,
INR 1.1
HSP

C. 5 years old boy c/o of fever and arthralgia for 1 month he was treated by
his GP with oral ABx for 3 times but still little improvement and now he
lost much weight and had liver enlarged 3 cm below costal margin with
multiple bruises appeared on his body, he looks pale and unwell
CBC: HB 7, WBC 28, PLT 15, Coag. Profile = PT 13 APTT 28 INR 1.4
Fibrinogen 4.2 (NL 2- 4.1)
ALL

104
Hematology

2. Scenario about child with weight and HC below 0.4 centile,


2 hyperpigmented patch noted in the abdomen (less than 2 cm), and
investigation showed HB: 7, TLC: 3, PLT 22, asking about diagnosis ?
a. Fanconi anemia
b. Shwashmann diamond
c. Spheroctosis

May 2021
1. EMQ
A. Haemophilia A
B. Haemophilia B
C. VWD
D. ALL (3)
E. ITP (2)
F. Ehlers Danlos syndrome
G. NAI

A. A 6 months old boy of 16-year-old single mother presents with a big


bruise in the lumbar region. PTT: prolonged, PT: normal, Factor VIII:
normal and Factor IX showed marked deficiency.
Haemophilia B

B. A 13-year-old girl with hyper extensible joints (like her mother)


presents with Platelet count of 10 and a 2-day history of bruises.
ITP

C. A child, who is a known case of asthma, on steroid treatment, presents


with uncontrolled asthma Investigations: WBCs: 30, HB: 4
Platelet: 6
ALL

105
Hematology

2. A 5-month-old baby presents with offensive diarrhoea and recurrent


respiratory tract infection WBCs: (low) , Neutrophil (very low) ,
Sweat test: 25 What is the diagnosis?
a. CF
b. SCID
c. Schwachman–Diamond syndrome

Jan 2021
1. Skull x ray pic for patient
came from (Libya)
a. thalassemia major
b. sickle cell anemia

2. 6 years old child with pancytopenia, cafe au-lait patch, short stature,
learning difficulties, FTT, high MCV, retic count 0.5 , diagnosis?

a. pure red cell aplasia


b. Fanconi anemia
c. Thalassemia

Sept 2020
1. A child with rash on the legs and buttocks with high blood pressure, his
urine is normal with no protein or blood and doctor prescribed medication.
He came for follow up after 1 week; his joints improved, blood pressure is
normal with normal renal function and no rash. What’s the out-patient
investigation for follow up?
a. Nothing
b. Urine analysis
c. Renal function
d. Complement level

106
Hematology

2. Patient had fever and neutropenia on admission 0.9 and received IV


antibiotics for 2 days and after 48 hours the WBC 13000 and neutrophil
count 0.8. He was discharged home and came after 3 weeks for follow up,
he is not febrile his neutrophil count is 0.8 mother read about neutropenia
and its dangers and treatment and she wants to give child GCSF.
(Scenario of Cyclic Neutropenia) What to tell the MOM?
a. Give GSCF to raise neutrophils to 1.5
b. Come to GP in 4 WEEKS to repeat CBC
c. Admit and start IV antibiotics
d. Prophylactic antibiotics
e. Discharge and reassurance

3. Child with JIA on methotrexate once weekly, he has hypermobile joints.


He came today to take his dose, presented with cough and rhinitis and
fever 39.5. What to do? Two options
a. Admit and give IV antibiotics
b. Stop methotrexate
c. Give IV fluids
d. Oral antibiotics at home with 24 hr. free access to hospital
e. Delay methotrexate dose after result of CBC
f. High dose folic acid

4. Caucasian child 12 years old came with pallor, Spleen 3cm, Known case of
splenomegaly for 3 years, Hb is low, normocytic, retics was initially normal
then low retic 0.2 and Platelet 97,000 with normal WBCS.
A. What is the Diagnosis?
a. HS
b. Sickle cell
c. Leukemia
d. ITP
e. Autoimmune hemolytic anemia

107
Hematology

B. What is the Cause of recent illness?


a. Parvovirus
b. EBV
c. Hemolytic crisis
d. Splenic crisis

Jan 2020
1. picture of HSP in feet What will you check?
a. Blood culture
b. Bone marrow
c. Urine analysis
d. abdominal u/s

2. case 6-month boy take vit k at birth but vomit it injured accidentally
bleeding from puncture site, fever what diagnosis?
a. Hemophilia
b. Vit k deficiency
c. DIC

3. Child has convulsions for more than 25-mins, developed hematuria RBC in
urine
Rhabdomyolyses

4. 2nd couple of a twin, thrombocytopenia, large platelets, atypical


lymphocytes, abnormal movement
uncompleted qs

108
Hematology

5 . EMQ
a. ALL
b. DIC
c. Hemophilia
d. ITP
e. HSP
f. NAI
g. Glanzman Syndrome

A. 13 months boy PT 14, APPT 81 Haemophilia


B. 8yr multiple un explainable bruise with limp Plt 152,PT 15, PTT 25 NAI?
C. A 4yr case with URTI, after 1 week develop petechial rash, platlete
12000, HB 7, wbc 26, next step? All

Sept 2019
1. Baby with short stature and feature of pancreatic insufficiency and
pancytopenia, neutropenia, recurrent chest infection, normal sweat test
a. Schwan man diamond
b. person disease
c. cystic fibrosis
d. HIV

2. Patient with cardiac mummer 2 cafe au-lait patch pancytopenia Hb low


platelet low, with microcephaly diagnosis?
a. Fanconi anaemia
b. neurofibromatosis
c. schwasman syndrome

3. case of HSP what is accurate for follow up?


a. if no renal failure follow at least for 6-months
b. no protein urea and haematuria no need to follow up
c. if persistent microscopic haematuria do biopsy

109
Hematology

May 2019
1. picture of HSP ... WHAT is the most serious complication?

a. acute renal failure


b. GIT haemorrhage
c. CNS complication

2. 6-months failure to thrive with diarrhea loose stool with malodorous


WBC with N 0.5 L:2 low platelet sweat test negative, recurrent chest
infection
a. Shwachman diamond Q
b. SCID
c. CF

3. patient went to FRANCE develop profuse diahrea before 2 days of


admission (didn’t tell bloody) skin rash low platelets. high urea &
creatinine, ASKED about investigation :
blood film for fragmented cells

4. diagnosis patient came from Libya


a. thalassemia
b. sickle cell aneamia

5. 10 YRS with epistaxis... PT 16 normal & APTT 70 Prolonged platelets 149


diagnosis?
a. Von-Willebrand disease
b. Haemophiliaa
c. factor 7 deficiency

110
Hematology

Jan 2019
1. A child with short stature, skeletal abnormalities, pancytopenia no blast
cell and high HB F,
A. Diagnosis?
Fanconi anaemia
B. How to diagnose (2answers):
a. chromosomal fragility test
b. bone marrow

Sept 2018
1. 6-month-girl, chronic diarrhoea, fail to thrive , greasy foul stool , breast fed
till 4 months , eczema on face & elbow , distended abdomen , liver 2 cm ,
macrocytic anemia HB = 10.4 , wcc = 3.9, neutrophils = 0.5, plt = 105 , high
urea & creat , normal protein , stool C&S -ve , diagnosis ?
a. schwashman diamond
b. intinstinal lymphangectasia
c. WAS
d. CF
e. Coliac

2. 9-yrs old boy Caucasian, splemnomegly, pale , fever , cough , haemolysis ,


normal retics , low plts , dad had splenectomy
A. diagnosis?
hertedory sphyrocytosis
B. cause of current illness?
Parvovirus

111
Hematology

3. Child with large hemangioma face and neck & thrombocytobenia


kaseilbach merit $

4. Child, pancytopenia, increase HF


a. Fanconi
b. black fan diamond
c. hypersplenism

5. 6 year-old boy., 24 hr after appendectomy develop rash shown in pic sole


of foot, history of abdominal pain, healthy appendix, platelets =140
wbc6 = normal
A. most appropriate investigation?
a. blood culture
b. BM aspiration
c. Renal Biopsy
d. clotting study
e. urine analysis and microscopy

B. Immediate management??
a. pain relief
b. iv fluids
c. anti-spasmodic
d. anti-viral
e. antibiotics
6. 2-yrs-old Girl Asian, parents vegan, 4-weeks ago develop
weakness, malaise, anorexia, pale, hyporeflexia, unsteady gait ,anaemia.
HB =60 , MCV =99, WBC= 6.0, PLT=70, LDH =2010, ALP=290,
large segmented neutrophiles, diagnosis ?
a. vitamin b12 deficiency
b. leukemia

112
Hematology

7. After apply bp cuff child develop petechiae, normal PT , PTT , platelet count
A. investigation to do ?
a. platelet function
b. factor 8 level
B. Treatment?
a. no ttt
b. give plasma

May 2018
1. 15-years-old child with excessive bleeding with prolong APTT, and factor
VIII was 40 (low), Father also has history of bleeding disorder
a. VWD
b. Haemophilia A
c. Haemophilia B

2. HSP case with normal everything, urine dip stick clear what to do?
Monitor blood pr for 6 months

Jan 2018
1. 3-years-old lethargy headache sore throat and fever 24 hours developed
rash over chest and abdomen, temp 38.5, RR 35, HR 135, has
maculopapular rash chest and abdomen labs showed neutrophils 0.4,
what to do next, need to choose 2 ?
a. IV immunoglobulins
b. antifungal
c. blood C/S
d. salicylate
e. IV antibiotics

113
Hematology

2. bloody urine diarrhea for few days then developed oliguria and petechial
rash Hepatomegaly tip spleen labs showed low platelets increased RFT
what to do?
a. stool c/s
b. blood film
c. ASO
d. bone marrow

3. 15-yrs-old presented with pallor spleen was 3 cm below costal margin


blood film showed spherocytes and polychromasia labs hb 95 mcv 95
mch 29 platelets 175 retculcytes 5.3, What to do to confirm?
a. cold annlutnin
b. haptoglobin
c. bone marrow
d. DCT
e. Bilurubin
f. EMA binding
g. Hb electrophoresis

4. 5-yrs Asian born in UK complained of yellowish eye discoloration visited


India developed fever and fatigability took antibiotic had history of
neonatal
jaundice required NICU admission labs showed HB 106, LFTs normal
what is your diagnosis?
a. Hepatitis A
b. Typhoid
c. Malaria
d. Gilbert
e. G6PD

114
Hematology

5. 14 months with recurrent chest infections and greasy stools neutrophils


0.7 IGs normal Cl at sweat test 19 platelets 130 what is your diagnosis?
a. Schwasman
b. HIV
c. CF
d. Coeliac

Sept 2017
1. joint pain then abdominal pain then rash on the back of LL+ Blood results
anaemia low platelet and normal WBC
a. HSP
b. SLE

2. 8-months Hb 3.2, Fever, vomiting, diarrhea, MCV 80, Retics 0.2 , HbF
raised
a. B-thalassemia
b. Diamond black fan

3. Patient with VWD and epistaxis for 40 min


DDAP and tranexamic acid

4. Africo-Caribbean boy staying with grand parents 2wks h/o of fever joint
pains HB 5-gm cause of anaemia?
a. Sickle
b. aplastic anaemia

5. Male baby admitted for elective hernia repair, PT , Apttt normal,


platelets number normal
platelet function

115
Hematology

May 2017
1. Sickle cell anaemic child has anaemia with normal reticulocyte and wbc
Parvo-virus serology ??

2. HSP how to follow up


you need to know each follow up by weeks like BP and proteinuria to
answer this question

3. picture for neonate has hematoma in eye lid and chest petechial rash
asking how to investigate need 2?
a. Platelet antibodies?
b. sepsis screen

Jan 2017
1. 2 cases of haemolytic uremic syndrome . . One was atypical with
neurological signs.askes about investigation and answer was blood film

2. case with HB electrophoresis. . HbF 9%


a. thalassemia trait
b. thalassemia major
3. a case of female pancytopenia
a. shwashman diamond syndrome
b. WAS

4. fanconi anemia
A. diagnosis
B. 2 investigations
a. bm biopsy
b. chromosomal fragility test

116
Hematology

5. a picture with rash on feet. .Acute abdominal pain went for operation . .
Then developed a rash on sole of leg
HSP

May 2016
1. Girl developedd nausea vomiting and profuse Diarrhea
investigation anaemia, thrombocytopenia, high urea and creatinine
Q1. Dx?
HUS (haemolytic uraemic syndrome)

Q2. Diagnostic investigations 2 option


a. blood culture
b. stool culture (EColi_ 0157)
c. urine culture
d. blood film
(schistocytes due to
microangiopathic heamolysis)

2. Question aplastic crisis in sickle cell anaemia with low Hb normal


reticulocytes and normal WBCs what is cause?
Parvo virus B-19

3. Case of huge hemangioma & low platelets (kasabach), ask about ttt?
a. Embolization
b. Sclerotherapy
c. Propranolol

4. Father has bleeding disorder, child has prolonged aPTT, normal PT


a. vWD
b. Hemophilia

117
Hematology

Jan 2016
1. patient with Kawasaki treated with aspirin and immunoglobulin blood test
reveal Hb 9 , ferittin 300 (? -300 ) , Hb f 9 %, What investigation for cause
of anaemia :
a. Haemoglobinopathy
b. Bone marrow biopsy

2. Pt 9 y with diarrhea bloody and vomiting urine blood and protein


investigation: Hb 9 , platelet low , urea 40 what 3 investigations to
confirm diagnosis :
a. Peripheral blood film
b. Creatinine clearance
c. Reticulocyte count
d. Stool culture

3. EMQ jaundice
A. Neonate with bilirubin 570 (Cilger najar)
B. G6PD
C. Pt with lethargy intermittent jaundice (Gilbert)

4. Pt with history compatible with HSP ask what investigation for follow up

118
Hematology

5. Sickler pt will undergo surgery Hb 8 what parameter willheamoglobin


or dec be given t s
prevent post-operative crises: <30%

a. Increase fluids
b. Top up blood transfusion to increase the Hb to 10
c. prophylactic antibiotic
e. maintain oxygenation

6. child shown with rash on buttocks asking about possible serious


complication associated.
a. GI bleed
b. nephritis
HSP—and most serious comp is nephritis

May 2015
1. child with learning difficulties low hb and platelet.& hbf..
A. Diagnosis?
a. Fanconi
b. thalesemia
B. investigations?

Sept 2014
1. EMQ: about thrombocytopenia Carbamezabine

Carbamizole
A. HUS
hydroxyurea
B. drug induced

119
Hematology

2.Fever, vomiting, diarrhea


Hb:3.2
MCV:80
Retics :0.2
Cong hypoplasiatic anaemia. (Diamond black-fan)

3. Male baby admitted for elective hernia repair, only there is petechia on
forearm when inflating cuff of sphingomanometer,
Pt, apt---normal
Platelet number--- normal

platelet dysfunction
no treatment

4.5-year old African boy, staying with grandmother with 2 weeks hx of fever,
joint pain, Hb 5 g/l , retics 0.4%, wbcs and platelets normal. Tip of spleen
palbable.
Q1:
What is corrlete with above senior?
a. Sequestration crises
b. Aplastic crises
c. Painfull crises
d. Acute haemolysis.

Q2:
What investigation needed? Need two
a. Osmotic fragility test.
b. Parvovirus work up.
c. Hb electrophorasis.

120
Hematology

5. Management of cyclic neutropenia

6. child with joint pain with non-thrombocytopenic purpura

a. HSP
b. EBV
c. SLE (cause thrombocytopenia)

7. Pt with VWD and epistaxis for 40-min which stopped, hb 9.


a. Refer to ENT
b. DDVP and tranxmic acid
c. Factor-8 concentrate
d. Whole blood transfusion

121
Oncology

122
Oncology

Sept 2021
1. 13 yrs old female who was travelled to Scotland with her family
presented with recurrent early morning vomiting. And nausea during the
day she complains of difficulty at school and recently her mother noticed
that her daughter developed difficulties in watching TV and difficulties in
reading Class board despite she set in the first raw. MRI FINDING
revealed blurring of optic disc. What is the most likely Dx of her
condition?
a. Brain tumor
b. Lyme disease
c. School avoidance

2. 2 days history of wt loss, right upper and lower facial palsy and
convergent squint?
a. Facial nerve palsy
b. Intracranial tumor
c. Pontine tumor
d. Bell’s palsy

May 2021
1. A 10-year-old girl with a 2-year history of headache came for short stature.
Her weight increased from the 50th to the 75th centile and her height
decreased from the 75th to the 50th centile. Her foot wear size did not
change for 2 years. What is the diagnosis
a. Cushing syndrome
b. Hypothyroidism
c. Posterior fossa tumor
d. Craniopharyngioma

123
Oncology

2. This is an X-ray of a 14-year-old male with cough and wheezes for 3


months What is the diagnosis?

a. Teratoma
b. Lymphoma
c. Bilateral pneumothorax

3. A 4-week-old miserable neonate with a liver enlargement of 6 cm, pedal


edema and 2 blue non tender subcutaneous nodules in the chin. What is
the diagnosis?

a. Hepatoblastoma
b. TORCH
c. Neuroblastoma

4. A12 year old boy presented with recurrent episodes of headache,


irritability and sweating. His blood pressure was found to be 140/100
What investigation will be needed?

a. Urine catecholamines
b. abdominal ultrasound
c. bone marrow

124
Oncology

Jan 2021
1. boy with left knee pain for 5 days, no swelling or sign of inflammation-
x-ray shows osteolytic lesion over distal femur. 100% oncology

a. Ewing sarcoma
b. Osteosarcoma
c. Osteomyelitis

Sept 2020
1. Chest X-ray of Sudanese boy 13 years came to visit UK, with 2 weeks
cough and x-ray showed massive Lt pleural effusion and mediastinum
shift to right. What is the Diagnosis?
(CXR exactly like sample paper) Sample paper

a. Lymphoma
b. Foreign body
c. Pulmonary tuberculosis
d. Streptococcus pneumonia

125
Oncology

Jan 2020
1. Boy 12yr, with 2 w hx of fever, received antibiotic with no improvement,
liver 3 cm, macular rash, and knee swelling with leukocytosis
investigations: CRP 100, ESR 150, EBV IG AB negative, diagnosis?
a. ALL
b. SLE
c. EBV
d. JIA
e. Septic arthritis

2. X-ray of 10 years old boy from Bangladesh complain of pain in his


knee and Swelling

a. Osteogenic sarcoma
b. Osgood
c. chronic Osteomyelitis
d. Juvenile arthritis
e. leukemia
f. vit d deficiency

3. Girl has supraclavicular Lymph Node complain of sore throat negative EB


Antibody, Liver 1 cm no splenomegaly what diagnostic investigation?

a. Bone marrow
b. LN biopsy
c. Blood culture
d. Ct chest
e. LP

4. Part of EMQ:
C. A 4yr case with URTI, after 1 week develop petechial rash, platlete
12000, HB 7, WBCs 26, next step?
All

126
Oncology

5. 14 y, girl, sexually active have headache, amenorrhea 3 months, nausea,


vomiting, high HCG in urine, low urine osmolality, low T4, normal
abdominal u/s what to confirm diagnosis?
a. MRI
b. Transvaginal u/s to detect
c. serum HCG

Sept 2019
1. picture of x ray of humorous with lesion (I think tumour) , parent notice
swelling which was firm & tender but no change in skin ask what to do?
a. discus with oncology
b. check vitamin D
c. refer to social worker
d. aspirate

2. high ferritin high LDH loss of weight, fever ,and palor ,, CBC picture
low platelet low HB ,,need investigation?

a. urinary catecholamine
b. bone marrow
c. abdominal u/s
this may be HLH or ALL, in both we do bone marrow

May 2019
1. x-ray 9-years old boy URTI and increase breathlessness

a. lymphoma
b. teratoma
c. normal thymus

127
Oncology

2. baby presented with abdominal mass right side 7.cm BCM and left side
4 cm alfafetoproten 29050.diagnosis?

a. Hepatoplastoma
b. Neuroblastoma

3. neonate pulse 180, tachypneic and firm 6cm Hepatomegaly, pedal


oedema with high BP & 2 blue non tender nodules, diagnosis?

a. Myocarditis with heart failure


b. Coarctation
c. Neuroblastoma
d. Nephroplastoma

Sept 2018
1. 2 yrs old with swelling tender arm, normal overlying skin , stable ,
pic x-ray(lytic lesion, periosteal reaction , soft tissue swelling,
osteopenia)

a. refer to oncology
b. aspiration biopsy
c. give antibiotics
d. do bone profile

May 2018
1. x-ray knee (11 years with sun burst appearance, bone density decreased

a. Osteosarcoma
b. Rickets
c. Chronic osteomyelitis
d. Osged schlater

128
Oncology

Jan 2018
1. 18-months-male had 3 weeks history of fever weight loss rapid abdominal
swelling on examination was cachectic with large abdomen 8 cm mass
below costal margin and 4 cm below level costal margin labs showed LDH
960(hinh) alfafetoprotein 29050 (v.high) what is diagnosis?
a. Neuroblastoma
b. Hepatoblastoma
c. ALL
d. Lymphoma

2. Q about tumour lysis syndrome bulky tumour was given high dose of
chemotherapy then they gave you changes happened after receiving the
drugs and you need to choose 2 from the options (repeated Q)
a. Uric acid
b. High K

3. x-ray:
a. Lymphoma
b. TAPVD
c. Teratoma

129
Oncology

4. Medulloblastoma case MRI scan ... best immediate next action


(two answers needed) ?
a. antiemetics
b. refer to neurosurgery
c. refer to oncologist

Sept 2017
1. child with multiple lymph nodes on neck, inguinal, thigh
chest x-ray wide mediastinal

A. diagnosis?
a. Lymphoma
b. IMN

B. investigation?
a. needle biopsy
b. excisional biopsy

2. Africo-Caribbean Child with joint pain and fever spleen tipped hb 5-gm
whats diagnosis?
ALL

3. Lt and rt mass and high AFP


Hepatoblastoma

130
Oncology

May 2017
1. CXR of mediastina mass
a. Lymphoma
b. Teratoma
c. Thymus shadow

2. scenario for 14 girl, weight loss, fever, multiple masses, pale


a. Neuroblastoma
b. ovarian cyst
c. Appendicular abscess

3. child with history of 2yrs headache, short stature, weight on 75th


centile, diagnosis:
a. Hypothyroidism
b. Tumour

Jan 2017
1. history of back pain. . X-ray picture Asks for the process?

a. osteolytic lesions
b. oseteoprosis

2. question about leukemia


A. diagnosis . >>ALL
B. confirmative investigation >>bone marrow biopsy

131
Oncology

Jan 2016
1.pic of child with gingival hypertrophy, tiredness 6 m duration, pallor:
a. Phenytoin toxicity --- in epileptic pt
b. AML – very long duration, 6 months, pt unwell
c. Scurvy

2. 2 y with pallor and irregular abdominal mass Hb 9 and ferritin 650


what 2 inv:
Neuroblastoma
a. Urine catecholamine
b. Bone marrow biopsy
c. Abdominal US
d. HPO OR HPMO

May 2015
1. A child with ALL on treatment..2 things u will worry about..
a. high K
b. high phosphate
c. high uric acid

Sept 2014
1.child with multiple lymph nodes on neck inguinal thigh chest x-ray wide
mediastinum, diagnosis and investigations?

lymphoma
excisional biopsy

2.Lt and rt mass and high AFP


Hepatoblastoma

132
Immunology

133
Immunology

Sept 2021
1. 5 years old boy has recurrent skin infections, pneumonia and meningitis,
uncle died during childhood from meningitis. What is the investigation?
a. Immunoglobulin level
b. NBT
c. Bone marrow aspiration

May 2021
1. An-11-month-old patient with low platelets, dermatitis not responding to
emollients and hydrocortisone. He presented with a history of 2 lower
respiratory tract infections. All investigations were of normal range except
for platelets (39) What is the most likely diagnosis?
a. Winscott-Aldrich syndrome
b. ITP
c. Cow’s milk protein allergy

2. Data about immunoglobulins, IgM and IgG were low, What’s the definitive
treatment for this case?
a. Parental immunoglobulins
b. Bone marrow transplant
c. Systemic antibiotics

Jan 2021
1. 2 years old boy, recurrent Otitis media and previous meningococcal
meningitis at 4 months old, physical examination no lymphadenopathy or
splenomegaly what is the diagnosis?
a. Ig A deficiency
b. SCID
c. X linked agammaglobulinemia
d. VID
e. Wescott Aldrich syndrome

134
Immunology

2. Child with recurrent infections, severe eczema not responding to


treatment, nail candidiasis, multiple fractures, special facies, broad nose.
Father also has same facies. What’s the diagnosis?
a. Hyper IgE syndrome
b. Chronic granulomatous disease
c. Hypogammaglobinaemia

Sept 2020
1. 10month -old boy presented with bruises and two episodes of
respiratory tract infection and eczema which is resistant to treatment
with emollients and topical steroids, CBC normal except platelet 20.
What is the diagnosis?

a. Wiskott- Aldrich syndrome


b. ITP

2. 5 years old boy has recurrent skin infections, pneumonia and meningitis,
uncle died during childhood from meningitis. What is the dx?

a. X linked hypogammaglobulinemia
b. SCID
c. CGD
d. Schwachman diamond

3. 4 months girl with failure to thrive treated for RSV bronchiolitis and then
came after a month with same symptoms, wheezes not improving, still
positive for RSV and have eczema x-ray show bilateral interstitial
shadows. What is the responsible organism?

a. Mycoplasma
b. Pneumocystis carinii
c. Streptococcus

135
Immunology

Jan 2020
1. Boy recurrent otitis media, pneumococcal pneumonia, uncle died from
meningitis what inv to do?
a. NBT
b. Immunoglobulin level

May 2019
1. 10 months had 2 episodes URTI had eczema resistant to treatment low
platelat,, clear scenario of

wiskcott syndrome

2. 5-yrs old hyper IGE SENARIO patient came with recurrent chest infection
have abscess also and his uncle have same condition, father and both
have broad nose (disrubted facial features) high arch palate
history of fracture humours with minor trauma, severe eczema not
response to medical treatment ,, nail candiasis

a. Hyper IGE
b. SCID
c. Wischott
d. atopic dermatitis
e. zinc deficiency

Jan 2019
1. 4 months, FTT, RSV a month ago, persistent cough, +ve RSV now ,
bilateral diffuse infiltrates :
a. Mycoplasma
b. Streptococcus
c. pneumocystis carinii

136
Immunology

Sept 2018
1. 4-month-old , poor weight gain , diarrhoea , after start formula , recurrent
oral and perianal candidiasis , diagnosis ?
pic of facial eczema
a. HIV
b. cows milk intolerance
c. acrodermatitis enteropatheca

2. 2 yrs old male with 3 episodes of otitis media , recover from meningococcal
meningitis, not dysmorphic, normal growth, no lympodenapathy, no
splenomegaly , diagnosis ?
a. IgA deficiency
b. HIV
c. SCID
d. X-linked agamaglobinemia
e. common variable

May 2018
1. male preterm 32-weeks, mother started to breast feed, she developed VZ
after 5 days what to do?
Immunoglobulin to the baby only

2. 14-month-old with 2 month history of cough, and yellow sputum, poor


appetite and weight loss , recurrent otitis media ,maternal uncle died from
recurrent pneumonia ?

a. immunoglobulime level brutons x linked hypogammaglobinemia


b. CT chest
c. Nasal brushing suction

137
Immunology

3. Girl with decreased immunoglobulins, -ve PHT test,


a. Pneumocystis
b. HIV
c. SCID

4. eye photo? ataxia telangiectasia

May 2017
1. EMQ:
A. low platelet and eczema…Wiskott–Aldrich syndrome
B. recurrent cough abscess lymphnodes …….. CGD
C. cardiac problem and low ca cleft lip…….digorge

2. Recurrent infection associated with lymph node enlargement diagnoses?


HIV

Jan 2017
1. case of Low platelets, Male, Repeated infections, Lab given
wiskot Aldrich syndrome

2. Case history going with Cleft palate, Heart Murmur,


DiGeorge syndrome , Asks about best investigation?
Ca level

138
Immunology

3.a case with IgG low


a. SCID
b. HIV

May 2016
1. Child with recurrent sinopulmonary infection and skin abscess uncle died
from meningitis what is diagnostic investigation?
a. NPT
b. T Lymphocyte subclass (nitrobule tetrazolium test used for chronic
c. Ig levels granulomatous disease which is Linked (uncle
d. immunoglobulin assay died)

Sept 2014
1. 4-month-old Girl with decreased immunoglobulins... PHA TEST (--‐VE).
a. Hiv
b. scid

Jan 2013
1. child AROUND 3Y with recurrent abscess formation, no failure to thrive,
his investigations showed: wbc= 7 half of them lymphocytes, IG: normal
igG, igA, high Ig M
what’s best to reach diagnosis?
a. lymphocyte subset
b. IG subclass
c. neutrophil functions ??

139
GIT

140
GIT

Sept 2021
1. Scenario of mother had autoimmune disease and child had wt loss, low
HB: 89 (MCV not mentioned), Rt illiac fossa pain but no palpable masses
and no blood in stool, ESR: 40, CRP: 30, NL WBCS and PLT with mild joints
pain, what is the Dx?
a. Celiac disease
b. Crohns disease
c. Ulcerative colitis

2. X-ray with contrast (typical x-ray)


a. Malrotation
b. Duodenal atresia

3. 4 years old has sudden onset of vomiting, abdominal pain. Then pt.
presented with temp 37.7, CRT was 5. On examination there was
abdominal distension, tenderness and sluggish bowel sounds. Abdominal
x-ray shows dilated bowel loops. Investigation: metabolic acidosis,
lactate 6, RBS 9 (N= 4.5-6) (‫خالف‬
‫ي‬ ‫)سؤال‬
a. Volvulus
b. Perforated appendix??
c. DKA
d. Sepsis??
e. Intussusception

May 2021
1. A 6-week-old infant presented with recurrent vomiting. He was hungry
and jaundiced. Urine analysis from a urine bag shows, WBCs 10 X109/L.
What is the diagnosis?
a. UTI
b. Pyloric stenosis
c. GORD

141
GIT

2. A 13-year-old girl with a 4-month history of recurrent colicky lower


abdominal pain came to ER with severe abdominal pain and vomiting for
18 hours. On examination: There was perioral pigmentation and no
abdominal mass and Investigations showed: HB: 83 , WBCs: 7
(normal up to 5) , CRP: 7 (normal up to 5) ,ESR: normal, Electrolytes: normal
Glucose: 8 (normal up to 7) , What is the diagnosis?
a. DKA
b. Appendicitis
c. Chorns disease
d. Puetz Jegeher’s syndrome
e. Addisonian crisis
3. 18 months old boy presented with vomiting and watery diarrhea, diffuse
lower abdominal tenderness and temp 37.8. Then vomiting stopped but
still has diarrhoea that becomes bloody. Mother described that the
patient withdraws his legs toward his abdomen as if he is in pain.
a. Meckle's diverticulum
b. Campylobacter gastroenteritis
c. Giardiasis
d. Rectal polyps
e. Rectal prolapse

4. A 7-year-old child, a known case of autistic spectrum disorder presents


with a long history of vomiting and his mother tried different types of
food modalities without improvement. He used to put everything in his
mouth and presented to ER several times with ingested foreign body. For
the last 6 month he has been refusing feeds and loosed weight. Bowel
habits were normal. What is the diagnosis?
a. Crohn’s disease
b. Psychogenic ??
c. FB ??
d. Achalasia
e. Reflux esophagitis
f. Eosinophilic esophagitis

142
GIT

Jan 2021
1. First line Drug of choice for induction remission of sever Crohn’s disease in
12 years child

a. Prednisone
b. Elemental diet
c. Azathioprine
d. Methotrexate
e. Aminosalicylic acid

2. A child with symptoms of celiac found to have anti tissue


transglutaminase antibodies 128, (normal < 7), what is the next best
investigation?

a. Diagnose coeliac and start gluten free diet


b. jejunal biopsy
c. gluten free diet and reintroduce challenge
d. Anti endomysal antibodies and HLA

3. 9 months old patient has constipation, he took laxative but not improved,
he has history of delayed passing meconium after 72hrs, what’s the
investigation help to reach diagnosis?

a. X-ray abdomen
b. Rectal biopsy
c. Lower GI contrast study

143
GIT

Sept 2020
1. 22-months Down syndrome has weight loss and constipation alternating
with loose stool and on thyroxine for hypothyroidism, presented by
abdominal pain, low iron. What is the Diagnosis?

a. Celiac disease
b. Thyroxine toxicity
c. Constipation with over flow incontinence
d. Hypothyroidism

2. A child came with fever and rhinitis; during examination you notice his
tongue like the picture. What is the dx?

a. Geographic tongue as sample paper


b. Candida
c. Scarlet fever
d. Leukoplakia
e. Lichen planus

JAN 2020
1. 10 years old losing weight, Abd pain, Stool 3 _ 4 Stool, pale, hypochromic
microcytic anaemia not improved on iron therapy, low albumin, low IgA,
high IgE , distended abdomen
a. chrons disease
b. celiac disease
c. Giardiasis
d. Hypothyroidism
e. Cystic fibrosis

144
GIT

2. 3-year child in nursery has watery offensive diarrhea 3 weeks. .stool


analysis done. cyst discovered by special stain +ve, abd.cramps, all children
in nursery have also diarrhea. What is the causative organism?
a. E.histolytica
b. Giardia
c. cryptosporidium
d. Rota virus

3. picture of geographical tongue


a. Reassure
b. start oral nystatin
c. acyclovir

4. 24 months Painless bleeding per rectum, abdomen soft


a. rectal polyp
b. meckle diverticulum

5. 14 yr girl History of fever, abdominal pain, weight loss of 5kg in two month
now have diarrhoea &vomiting, tender right iliac fossa mass, high ESR,
high CRP , difficult to move lower limb
a. Chrons disease
b. Ulcerative colitis?
c. Appendicular abscess?
d. Volvulus
e. Ectopic pregnancy
f. Twisted ovary

145
GIT

6. 22-months Down syndrome has weight loss and constipation alternating


with loose stool and on thyroxine for hypothyroidism, presented by
abdominal pain, low iron. What is the Diagnosis?
e. Celiac disease
f. Thyroxine toxicity
g. Constipation with over flow incontinence
h. Hypothyroidism

Sept 2019
1. x-ray neonate on day 3 with bilious vomiting, (x-ray not clear but there
was dextro-cardia),ask what is
diagnosis?
a. malrotation
b. intussusption
c. NEC
d. small bowel obstruction

2. case of CP child with recurrent


respiratory infections, arching back
a. recurrent aspiration
b. GOR

3. ex-preterm neonate was intubated for 5-weeks now 5-month, treated for
recurrent pneumonia, high arch palate x-ray show patchy change
what is cause?
a. recurrent aspiration
b. GOR
c. TOF

146
GIT

4. patient with abdominal pain, oedema, biopsy show dilation of lymph


vessel of sub mucosa & mucosa,, lymphocyte low ,,protein low

A. what diagnosis?
intestinal lymphgectesia

B. ask about the diet type?


a. medium chain fatty acid
b. 50% MCT & 50% LDL

5. long senario about pancreatitis need investigation?


a. u/s abd
b. fecal elastase

6. pt with wt loss abd pain


inflamtery bowel disease

May 2019
1. change bowel habits, loss wt , investigations 3 options:
a. Thyroid function test
b. Fecal calprotectin
c. iga tissue transglutaminase

2. out age of intussusception patient have intermittent abdominal pain


, distension, absent bowel sounds, rt iliac fossa mass, bilious vomiting,
blood in napkin
a. Mackles
b. Intussusception

3. Patient with scenario of functional abdominal pain

147
GIT

Jan 2019
1. A Case of bloody diarrhoea for 6 weeks & weight loss after coming from
Pakistan, high inflammatory markers, mouth ulcer, LFT HIGH, CRP high ?
a. Salmonella
b. Shigella
c. amoebic dysentery
d. Ulcerative colitis

2. A child with symptoms of celiac found to have anti tissue


transglutaminase antibodies 128 , (normal < 7 )what is the next best
investigation?
a. Diagnose coeliac and start gluten free diet
b. jejunal biopsy
c. gluten free diet and reintroduce challenge
d. Anti endomyseal antibodies and HLA

3. 2-month-old at birth hypotonic, AXR, FTT, vomiting, normal bowel sound


irregular non tender mass and abdominal distention, no air in the distal
intestine?
a. Hirschsprung
b. Nephroblastoma
c. Neuroblastoma

4. girl taking chemotherapy on morphine did not passed urine since 1:00 pm
now it is 10 a.m & not passed stool 2 days, management?
a. Warm bath
b. Catheterisation
c. Movicol and encourage oral fluid intake

148
GIT

5. EMQ:
A. DM, poor weight gain, ↓ albumin coeliac
B. Bloody diarrhea, weight loss, mouth ulcers crohn's
C. Noonan with faltering growth, loose stools, low albumin, low
lymphocytes:
intestinal lymphangiectasia

sept 2018
1. 14-yrs-old Cystic fibrosis , RT side abdominal pain, intermittent vomit ,
constipation, RT side mass , BL pr =115/75, normal FBC, diagnosis ?
a. distal intestinal obstruction syndrome
b. fibrosing colonopathy
c. appendicitis
d. crohns
e. intussusption

May 2018
1. 22 months old with down syndrome with mix symptoms of lose motion
and constipation, and iron deficiency anaemia
a. coeliac disease
b. Chron’s disease

2. EMQ:
a. Lipoprotein lipase deficiency
b. Familial hypercholesteremia
c. Anorexia nervosa
d. Lipodystrophy
e. Intestinal lymphangiectasis
f. Abetalipoproteinemia

149
GIT

A. Patient has high cholesterol his LDL low and HDL and had node on the
elbow . Familial hyperchlostolemia
B. Patient present with pale stool ,lower limb swelling
Intestinal lymphagectasia
C. scenario about girl with anorexia nervosa
N.B
❖ Lipoprotein lipase deficiency: high triglycerides, high cholesterol,
milky serum, abdominal pain
❖ Abetalipoproteinemia: steatorrhea, ataxia, retinitis pigmentosa,
acanthocytosis, low LDL, low cholesterol low triglyceride
❖ Anorexia nervosa: all hormones dec except prolactin and cortisol
Jan 2018
1. 9 years crampy abdominal pain, 3 months diarrhea loss of weight
(9th centile for weight), muscle wasting, labs showed low albumin
leukopenia 0.8, trace protein in urine what is possible diagnosis?
a. Lymphangectasia
b. nephrotc syndrome
c. crhons
d. coeliac

2. X_RAY 10-months-old Asian had 4 hours unsteady, drowsy was feeding


well with no vomitng term non consanguineous marriage had TOF with
oesophageal atresia operated when he was younger but had delayed
taking solid food and no post operative complications on examination he
was febrile mottled capillary refill 5 sec PR 160 grunting soft abdomen
blood sugar 17mmol, PH7.1, pCO2 3.4, bicarb 9 , ammonia 57 , lactate 10
ketones 0.4 base def -14 , what is diagnosis?
a. Salisylate
b. DKA ‫السيناريو ماشي مع‬
sepsis
c. intusscipton ‫لكن قالوا انه كان فيه اشعه مع السؤال‬
d. volvulus?? ‫فيها‬
e. inborn errors of metabolism volvulus
f. sepsis ??

150
GIT

3. child diagnosed with severe Crhons what to do?


a. azathioprine
b. metronidazole
c. elemental diet ?
d. aminosalicylic acid
e. methotrexate
f. Prednisolone?

Sept 2017
1. 14-yr girl with right abdominal pain recurrent last 3 months, loss of
weight, with tender mass, fever?
a. Appendiclar mass
b. torsion ovaries
c. crohns

2. 6-weeks female presented with vomiting post each breast feed with
faltering of growth
pyloric stenosis

3. mental retarded child, faecal mass impaction, loose stool


constipation

4. Lactose intolerance following rota?


a. low lactose
b. Normal diet

5. 10-months with painless per rectum bleeding


a. Meckles
b. bleeding disorders
May 2017
1. GE SENARIO for Yersinia colitis
after blood fracture — bloody diarrhea—come with high conc in iron

151
GIT

2. clear scenario for CELIAC DISEASE


Jan 2017
1. EMQ question about Gastroenteritis:
A. child handled a turtle >>salmonella
B. child visited veterinary farm>>cryptospiroidosis
C. child with. . History included introduction of formula after he was
breastfeeds--- cow's milk ptn allergy

2. classic history of crohn's disease


A. diagnosis? Crohn's disease
B. diagnostic investigations: ileoscopy &biopsy

3. a case of cystic fibrosis. . History of intestinal obstruction. . Swelling at


right hypochindrium.
DIOS

May 2016
1. A case of Failure to Thrive with offensive Diarrhea (steatorrhea)
investigations with low faecal elastase
A. Ask about Dx??
a. Abeta lipoproteinemia
b. Cystic fibrosis
c. Schwachman diamond
B. ask peripheral blood picture?
a. Acanthocytosis
C. ask about causes of disease?
a. Vit E deficiency
b. Haemolysis
c. fat mal absorption
d. pancreatitis

152
GIT

Jan 2016
1. Extending matching q
Intusseption
campelobacter gastroenteritis
cows milk protein intolerance

May 201
1. A question about oedematous child..low albumin..think was fitting with
intestinal lymphangiectasia

2. a child with bloody stools from Pakistan


a. UC
b. salmonella

Sept 2014
1. 14 yr girl with left abdominal pain with tender mass?
Crhon disese ????

2. mental Retarded child, faecal mass impaction, loose stool?


Constipation

3. following rota virus GE?


Lactose Free milk ??

4. 10-month with painless bleeding per rectum


Meckles scan

5. 6-week female presented with vomiting post each breast feed with
faltering of growth
Pyloric stenosis

153
Hepatology

154
Hepatology

Sept 2021
1. Scenario about 14 yrs. old girl, obese BMI: 32, presented with right
hypochondrial pain, right shoulder pain and epigastric pain of one month
duration. Investigations: conjugated hyperbilirubinemia, normal LFT.
Next step investigation?
a. Abdominal U/S
b. ERCP
c. HIDA scan
d. faecal elastase

2. A 14-year-old boy presents with recurrent jaundice and lethargy. There


was no urobilinogen in the urine. What is the most likely diagnosis?

a. Gilbert syndrome
b. Cigler’s-Naggar syndrome
c. Autoimmune hepatitis

3. A 13-year-old male presenting with knee pain and hyperglycaemia. On


examination: The weight is 99.6kg, Height is in the 50th centile. He has
acanthosis nigricans Investigations: Protein: 8 , Albumin: slightly low ,
ALT/AST: slightly high , GGT: 40 (40-60) What the diagnosis?

a. Non-alcoholic steatohepatitis
b. Hemochromatosis
c. Autoimmune chronic active hepatitis
d. Hepatic cholestasis

Jan 2021
1. Case boy with normal protein IGg = 20 & low albumin, deranged LFT,
diagnosis?
Autoimmune hepatitis

155
Hepatology

Jan 2020
1. case about 15 years old patient with recurrent jaundice, urine clear
with lethargy and fatigue for this was a active in sports however recent
months feeling lethargy she only feels lethargy episodic first
unconjugated bilirubin was 41 rest labs were ok,, repeated bilirubin
unconjugated 57

a. Gilbert syndrome
b. crigger najar
c. alpha one anti
d. chronic fatigue syndrome
e. Wilson syndrome

Jan 2019
1. 12-yrs General malaise, recurrent jaundice, no urinary bilinogen or
bilirubin, increased s.bilirubin (50) , increased ALT , normal AST,
phototherapy at neonatal period GG normal:
a. Gilbert
b. crigler Najjar
c. Hepatitis C

sept 2018
1. Case with normal protein, IG g = 20 & low albumin, deranged LFT
diagnosis?
Autoimmune hepatitis

2. Child obese, knee pain, normal joint , high liver enzymes


a. non alcoholic fatty liver
b. intrahepatic cholestasis
c. Haemochromatosis

156
Hepatology

3. Child from Uk , Pakistani, mom and dad , travel to wedding in Pakistan,


came with abominal pain.. nausea, vomiting, fever, loose stool, pallor no
jundice clinically, 2/6 systolic murmur, abdominal tenderness
no-splenomegaly liver 2 cm palpable, ALT 1200, ALP 60 ,WBC 14 , albumin
26 , Bilirubin 30, Which ivestigation to do?
a. Anti smooth muscle +liver and kidney microsomal antibody
b. blood culture
c. Abd us
d. Clotting
e. viral hepatitis serology
f. liver biobsy
g. copper ceruloplasmin

May 2018
1. 14-years-old with pain in right hypochondrium and they asked
investigation?
ultrasound abdomen direct question about gallstones

2. Increased cholesterol triglycerides, hyperglycaemia and


Jaundice
a. GSD liver biopsy
b. non alcoholic steatohepatitis

3. child with low Glasgow coma scale high liver enzymes low RBS-?
a. Reys
Sudden deterioration,
b. Paracetamol Without jaundice

Low GCS due to hepatic


encephalopathy

4. 5yr old Child with acute liver failure feature presenting with all
parameter’s defective with reversal of ag ratio
Autoimmune-hepatitis

157
Hepatology

May 2017
1. Child with lethargy, jaundice, weight loss, came back from France.
USS: small contracted GB . After admission, became very pale ,Hb down
to 60 from 90 . Increase conjugated bilirubin to 150, High protein ,normal
albumin, Hepatitis screen -ve ?
a. acute viral hepatitis
b. primary biliary cirrhosis
c. sclerosing cholangitis

2. 12-yrs child with prolonged jaundice, cyst on US at Rt kidney, high total


protein, inverted albumin/globulin ratio, high liver enzymes, high
bilirubin, hepatitis screening negative
Auto immune chronic active hepatitis

May 2016
1. A case of History and Examination with excluding the infectious cause and
look for hypoalbuminemia and inverted of albumin/globulin ratio and
also high protein
Autoimmune Hepatitis

May 2015

1. A child who had unconj hyperbilirubinemia as a baby and needed


phototherapy later on in childhood

criggler Najjar?

2. a boy with hereditary spherocytosis. With fever and rt hypochondrial


pain..?
cholecystitis
chronic h.anemia — gall stone

158
Nutrition
Obesity
Allergy

159
Nutrition & obesity

Sept 2021
1. Scenario regarding child with anorexia nervosa. Investigation parameters
(phosphorus was 0.4, K was 3.3, Ca 1.8 HR: 50, B/P: 90/50) ask about
which one to correct first?
a. IV Po4
b. K NGT
c. IV calcium
d. Normal saline

2. Photo of lesion around mouth and tip of fingers (well demarcated).


Scenario of preterm, who developed NEC, did bowel resection and
received TPN in neonatal period. C/O: FTT and dermatitis. What is the
investigation?

a. Zinc level
b. Iron level
c. Lead level
d. B12 level
e. Vit C level

3. 3 months old infant developed facial eczema from 2 months and


received topical steroid and emollient regime which showed little
improvement then deteriorated again and developed loose stool and
weight become less than 2nd centile.
Baby is on bottle feed.
What you will do:
a. Hydrolyzed formula
b. Soy milk
c. Tacrolimus
d. Oral high dose prednisolone

160
Nutrition & obesity

3. Scenario of anorexia nervosa, BMI 13. Which one is the most clinical
concern?
a. 10% dehydration ??
b. HR = 40/m ??
c. B/P < 0.4 centile ??
d. Temp 35.7c
e. HR = 90/m

4. X-ray wrist fracture both radius and ulna with cupping and large articular
space. What is the diagnosis?

a. Osteoprosis
b. Rickets
c. Osteogenesis
imperfecta
d. NAI

5. Baby aged 8 months was exclusive breast feeding till 6 months who was
recently moved to new house started weaning on solid food and formula
for 2 months. Worsen eczema that was initially controlled by
emollient. Pt. Developed mucous loose stool and weight loss. What is
the cause of recent condition?
a. Dietary allergen
b. Change environment
c. Infection

161
Nutrition & obesity

May 2021
1. A 5-month-old baby was started on formula milk one month ago then he
developed eczema resistant to treatment. What is the management?

a. Extensively hydrolyzed formula


b. Skin brick test for cow’s milk protein
c. Soya-based formula

2. A 5-year-old fuzzy eater with vegetarian family presents with ataxic gait,
nystagmus and decreased lower limb reflexes. MCV: 95 (normal up to
91). What investigation should be requested?

a. Serum B12
b. Serum Folate
c. LP
d. MRI spine
e. Nerve conduction velocity
f. Urine toxicology
g. Hair for trace elements

Sept 2020
1. 4-month case of extreme preterm have Nec with intestinal recession on
TPN 3 months (Zinc def. picture)

* What is investigation?
a- Zinc level
b- CMPA

162
Nutrition & obesity

2. Picture +scenario of child has


eczema with no response to
treatment
emollient, hydrocortisone
what mange? (cow milk allergy)
a. hydrolyzed formula

3. A case of anorexia nervosa BMI 15kg, started NG feeding after 24 hrs.


investigations show po4 very low (0,4), hyponatremic, hypotensive ,
hypokalemic 3.2, what imp management?

a. IV phosphate
b. Potassium
c. Refer to ditetion
d. IV normal saline bolus
e. NGT

4. 10 yrs. Girl with history of familial intrahepatic cholestasis and presented


with increased clumsiness, 2 months ago with pruritus and scratch marks,
on exam: Jaundice, decrease tendon reflex, ataxic. What’s dx?

a. Vitamin E deficiency
b. Vitamin b12 deficiency
c. Vitamin d
d. Folic acid
e. Vit k
f. Vit A

163
Nutrition & obesity

5. EMQ Formulas
Soya based
Carbohydrate based
Amino acid formula
Peptide based with 50 %MCT
Whole protein with 90% MCT
Whole cow milk
Goat milk
Standard milk formula

Which milk formula would you choose for the following patients?
A. Infant with Eczema and diarrhea
Amino acid formula
B. Infant with biliary atresia, post-operative, he is breast fed, admitted not
gaining weight. Still Jaundice
Peptide based with 50 %MCT Or 90%
C. Infant with thoracic duct injury after cardiac surgery with milky drain from
chest tube, Albumin 27
Whole protein with 90% MCT or %50

6. 2 years old Pakistani boy, faltering growth, developmental delay, he is


cruising and stands with support. PTH 41, Alkp 1089, Ca. 2.18 and
Phosphate level was normal dx?

a. Primary hyperparathyroidism
b. Rickets??
c. Gaucher

Jan 2020
1. patient in breast feeding for 8-month not remember, came with ca 1.9
twisting of face
vit D

164
Nutrition & obesity

May 2019
1. gum picture with history of
fatigability convulsion, on
antiepileptic, rash on
lower limb fuzzy eater

a. vit c deficiency
b. AML
c. Phenytoin toxicityzinc def

2. 20 months baby boy presented with GE on examination you noticed that


he is not walking. 2 months later he came to clinic still he is not walking
serum ca low. what 2 investigations to do?

a. hip x ray
b. both kness x ray
c. 25 hydroxycolecalciferol
d. parathyroid hormone level

3. Obese girl c/o tiredness for 2 months BMI is 32, HbA1c 48.normal
(42 -46). what to do?

a. oral metformin
b. change life style
c. sulphonyl urea
d. insulin

Jan 2019
1. A case of pruritus, had peripheral neuropathy and some
clumsiness, ataxia, jaundice known case of progressive familial
cholestasis diagnosis?
c. vit B12
d. vit E

165
Nutrition & obesity

2. An obese long boy, Bone age of advanced one year, assessed after one
year and still on the same centile, blood pressure on the 91 centile?
a. simple obesity
b. Cushing’s syndrome

sept 2018
1. X ray: fractured radius & ulna,
cupping,frying,increase,joint
space

c. rickets
d. osteogenesis imperfecta

2. 14-yrs-old girl collapsed, BMI = 15, brought to ER , HR=50, bp = 80/40 and


put her on high energy feed , she received bolus fluid , low k =3 , low po4=
0.5, low Ca = 1.9 , low mg = 0.6, cl = 101 ,next step in management ?
a. NG k
b. IV ca
c. refer to dietitian
d. IV mg
e. IV po4
f. fluid bolus

166
Nutrition & obesity

May 2018
1. picture of a child bottle fed with fluttering growth, rash on face and has
eczema, received emollient, topical steroid but no improvement.
what to next?

a. give hydrolysed formula feed


b. Tacrolimus
c. Soya based formula
d. Oral steroid

2. 5-years-old girl with height more than 99.6 centile and obese, mid
parenteral height was 25-50 centile

a. Check BMI *>> mostly simple obesity


b. insulin like growth factor

Jan 2018
1. 7-months-old with vomitng, irritability & intermittent diarrhea started on
formula milk at 4-months-age and now weight dropped what is diagnosis

a. cow's milk protein intolerance


b. lactose intolerance
c. UTI
d. GERD

Sept 2017
1. mass screening for obesity?
a. BMI more than 98 SD
b. weight >99 S
c. skin thickness >99SD
d. weight for height
2. Was their diarrhea with vesicles in hand and foot
Zinc deficiency

167
Nutrition & obesity

3. Bangladeshi girl ca low phos alk high - 2 investigations?


a. knee x-ray?
b. vit d
4. Baby with constipation, Hyper IG E
COW MILK FREE DIET

5. 3-ys 0ld with constipation, failure of laxative, has high IgE


against caws protines, What is the next step?

a. increase laxative
b. totally hydrolyzed milk
c. partially hydrolyzed milk

May 2017
1. obese girl with morbid obesity brother and normal sister and brother
investigation?
Leptin deficiency

Jan 2017
1. specimen paper x-ray picture of rickets & ulnar and fibula fracture
(2 options needed)

2. obesity assessment definition


BMI > 98th

168
Nutrition & obesity

3. case of cow's milk ptn allergy. . Asks about management


hydrolized formula

May 2015
1. A question with a child non thriving and eczema..whats next step of
management

give hydrolysed formula CMPA

2. Asian chid with data of rickets

Sept 2014
1.Hyper IG E
COW MILK FREE DIET

2. diarrhea with vesicles in hand and foot?


Zinc deficiency

3. Bangladeshi girl
Ca-- low
Po4 -- high
Alkaline--- high
Need Two investigations?
a. Knee-x-ray
b. Vit-D

4.mass screening for obesity


a. BMI more than 98%
b. Wt more than 90%.
c. Skin thickness more 75 centile

169
Nephrology

170
Nephrology

Sept 2021
1. Scenario regarding child with pallor, history of travel to France, recurrent
diarrhoea then become bloody diarrhoea, elevated RFT,
thrombocytopenia, drowsiness, asking about investigation to reach
diagnosis:
a. Blood film for fragmented RBCS
b. bone marrow aspiration
c. Renal us

2. Scenario about 7 yrs. child with nocturnal enuresis with day symptoms.
Ask about next step?
a. Offer oxybutynin
b. Encourage fluid intake in daytime with reward system
c. Wake her at night two times to go bath
d. Offer alarm system

3. A child 3 years girl, with vomiting, fever, WBC in urine 10, mixed
organism in urine. What best next step?
a. Abd. U/S
b. Suprapubic specimen
c. Catheter
d. Clean catch

4. pt. presented with vomiting, polyurea, polydipsea and clinically


dehydrated. CBC: normal, Na 132, K 3.2, Cl 122, RBS: normal. Urine
analysis: glucose +++, protein ++. What is diagnosis?
a. MODY
b. Fanconi syndrome
c. DI

171
Nephrology

5. 28 wks gestation neonate with Hx of PROM for 30 hours. Pt. is 12 hours


old UAC and UVC were inserted. Now presented with oliguria and high
B/P. Invest: CBC: HB 8, Plt 15, Pt&Ptt normal. Creatinine: 300, Urea: 15.
Urine analysis: RBCS 4+, protein 2+. What is the diagnosis?
a. Renal vein thrombosis??
b. DIC
c. Sepsis

6. Scenario about 4 years old boy with past history of non-specific rash, and
arthritis, and investigations showed high KFT, proteinuria, C3 C4
consumed, normal platelet and normal TLC, mild anemia, hypertension,
asking about diagnosis?
a. MPGN
b. HSP nephritis
c. Lupus nephritis
d. Acute glomerulonephritis

7. Scenario of male pt with haematouria and FHx of deafness in second


degree relative, history of viral infection in last 3 days. Urea&Creatinine:
normal, B/P normal. Ask about diagnose?
a. Alport syndrome
b. IgA nephropathy

May 2021
1. A 9-year-old girl with fever was found to have glycosuria but no
history of polyuria or polydipsia. Her fasting blood glucose level was 4
(normal 3 - 6). Fever subsided but glycosuria (3 +) persisted after the
infection had resolved. What is next step to clarify the cause?
a. Hb a1c
b. C peptide level
c. C peptide-creatine ratio
d. Paired fasting glucose with insulin
e. 75-oral glucose tolerance test

172
Nephrology

2. A 4-month-old presented with failure to thrive and vomiting,


Investigations: Serum Na 135, K: 2.2 , CL: 121
PH: acidotic
Serum bicarbonate: low
Base excess: Increased
Urine glucose 1+
Protein 1+
What is the diagnosis?
a. Fanconi syndrome
b. Diabetes insipidus
c. Distal RTA
d. DKA

3. A patient has abdominal distension. A CT abdomen was done. From which


structure the mass is arising?

a. Left Kidney
b. Left Suprarenal gland
c. Spleen
d. Pancreas

4. A 5-year-old boy came with hypertension. He had had a history of


recurrent UTI. Ultrasound showed upper pole changes of the kidney more
in the left side. What is the most appropriate investigation to assess the
cause of his hypertension?
a. DMSA
b. MCUG
c. MAG3
d. Ultrasound
e. Doppler ultrasound

173
Nephrology

5. A 5-year-old boy came with loin pain. There was no family history of
similar condition. He was born at 32 week and was admitted to nursery for
7 days when he was 6 days old. Ultrasound showed bilateral renal
calcinosis and calcium-creatinine ratio = 1.3 (0.6-0.8).
What is the diagnosis?

a. William syndrome
b. Idiopathic hypercalciuria??
c. Bartter’s syndrome??
d. Cystinosis
e. Fanconi syndrome

6. A 9-month-old previously well child presented with puffy eyes, pedal


edema and ascites.HR 110/min, RR 54/min Chest: clear Serum albumin: 18
Urine protein-creatine ratio: High Creatinine: 50 (30-50) What is the
management?
a. US abdomen
b. 24 urine protein
c. Urgent discussion with consultant nephrologist
d. Prednisolone 60 mg/M2/day OD for 28 days
e. Albumin 20 % I.V over 4-5 hours

Jan 2021
1. 10 years old girl with Pic of purpuric rash over extensor surface of
lower limbs asking about Most serious complication in the first week?

a. GIT haemorrhage
b. Acute renal failure
c. Blindness

174
Nephrology

2. pt has non bloody diarrhoea for 3 days then developed oliguria and
petechial rash Hepatomegaly tip spleen labs showed low platelets
increased RFT, What further test for diagnosis ??
a. Urine dip stick
b. stool c/s
c. blood film
d. blood C/S
e. bone marrow

3. Child known CF presented with vomiting labs showed Na 108, K 2.3.


Bicarb 37, What is the cause of these lab abnormalities?
a. Pseudo barter syndrome
b. Nephrogenic DI
c. Pyloric stenosis
d. CAH
e. Barter syndrome

4. MCUG showing grade five reflux asking next step?

a. MAG 3
b. DMSA
c. Kub
d. Cystoscopy

5. Neonate was admitted in NICU and came back after 1 week with
hematuria umbilical venous catheter was connected. Which
investigation to confirm the diagnosis?
a. Renal vein Doppler
b. US
c. RFT

175
Nephrology

6. Mag 3 photo showing left kidney not excreting after Lasix Next step?
There was another curve for bladder

a. refer to urology
b. MCUG
c. DMSA

7. 10 months old infant came with constipation, inx there was metabolic
acidosis, urine glucose +, urine protein + , diagnosis?

a. Barter
b. Distal tubular acidosis
c. Cystinosis

Sept 2020
1. A child with rash on the legs and buttocks with high blood pressure, his
urine is normal with no protein or blood and doctor prescribed medication.
He came for follow up after 1 week; his joints improved, blood pressure is
normal with normal renal function and no rash.
What’s the out-patient investigation for follow up?

a. Nothing
b. Urine analysis
c. Renal function
d. Complement level

176
Nephrology

2. 8-weeks infant presented by non-bilious vomiting, scaphoid abdomen


and constipation, CRT 3 sec NA 154, K 3.8, urine osmolarity (18 ‫) كانت كده‬
what is dx?‫عش فقط‬‫!!!!!! ما كان ف صفر ثمانية ر‬
‫ي‬
a. DI nephrogenic (sample paper question)
b. Salt poisoning

3. 9 months boy, failure to thrive, at 4 month mom started solid food, on


5 month decreased weight and constipated changed formula milk and
also took laxatives that did not help, urine glycosuria and proteinuria,
bicarb 11, ca 2.7, phosphate 1.0 , PTH 40 dx?
a. Cystinosis
b. William syndrome
c. Primary hyperparathyroidism
d. Hypophosphatemic rickets
e. Celiac

JAN 2020
1. 6-months with 3 days vomiting & dehydrated, inv NA 129, K 1.8 ph 7.49,
Hco3 39, diagnosis?
a. Bartter
b. pyloric stenosis
c. CAH
d. CF
e. renal stenosis
f. 1ry hyperaldosteronism

2. 3y UTI, nitrate, leucocytes esterase positive, colony count more than


10,000 mixed organisms on culture, what best next step?
a. Abd U/S
b. suprapubic specimen
c. catheter
d. clean catch

177
Nephrology

3. 15 years old with shortness of breath, fever and cough previously


diagnosed as SLE on steroids, enalapril and mycophenolate mofitl, poor
compliance to treatment She has mild learning problems.
Now: pale, alert, fever, tachypnea, tachycardia, bilateral lower zone
crepitations, decrease air entry bilaterally. Bp 160/100, with high renal
function, good urine output, creatinine 555, thrombocytopenia
K 3.2mmol, low Na, metabolic acidosis, bicarb 10, BE -6, hepatomegaly
6 cm, bilateral pedal odema

What is the most important next two actions?


a. increase dose of enalapril.
b. mechanical Ventilation
c. Oral nifedipine
d. iv furosemide.
e. platelet transfusion
f. Hemodialysis
g. Na bicarbonate iv
h. iv nitroglyceride
i. Iv K

178
Nephrology

4. scenario of UTI:
A. 4 month with atypical UTI elder sister having reflux
US in acute stage
B. Simple UTI 6y properly given antibiotic, elder sister having reflux
Nothing
C. 4 days male antenatal us postnatal us shows bilateral dilated
pelvicalaceal system, diameter 25
MCUG

5. 14 years girl polyuria 6 liters per day, polydipsia, dehydration, normal


glucose, urine osmolarity 180, Na 150, normal: urea, creatinine, k
a. central DI
b. Nephrogenic DI
c. DM
d. Compulsive water intake

6. Upper respiratory tract 3 days, hematuria 3+, Proteinuria 1+


IgA nephropathy

7. Case of hypernatremia and low osmolarity in urine, urine output high,


Holoprosencephaly
A. Diagnosis?
Central Diabetes insipidus
B. What to do?
Desmopressin

179
Nephrology

Sept 2019
1. Male patient with haematuria and family history of deafness in second
degree relative ask about diagnose,, history of viral infection last 3 days
a. Alport syndrome
b. IgA nephropathy

2. EMQ
a. HUS
b. pre renal failure
c. HSP
d. Neuroblatoma
e. Wilms
f. Pre- renal failure
g. drug induced nephritis
A. 5yrs old live in farm, develop non bloody diarrhoea with oligurea then
and renal failure blood film normal urine show trace of protein and blood
pre renal failure
B. Frank haematuria 2-month history of intermittent fever and irritability,
parent feel increase abdominal growth
Wilms
C. macroscopic haematuria painless,3 attacks associated with infection
2 occasions
???? if Ig A nephropathy in choices

4. haematourea HTN c3 and c4 normal, history of second degree relative


with defness
a. Alport syndrome
b. IgA nephropathy
c. PSCGN

180
Nephrology

3. CT scan wilms tumor

5. patient with U&E result with urine electrolyte glucose urea


(was clear scenario of Fanconi syndrome

May 2019
1. Patent with history of recent URTI came with heamaturia, high creatinine
C3 C4 normal , ASOT 200, BP NORMAL.. DIAGNOSIS

a. IGA nephropathy
b. acute GN

2. DMSA scan
not like this exactly
function 60:40 , lt is lower position than rt
(lt less uptake)
a. Scarring
b. Malposition and reduced uptake

181
Nephrology

3. patient with low Na, low k, cl normal, serum metabolic acidosis,


Urine (glu +,amino acid +, ph 5)
a. fanconi syndrome
b. barter
c. distal tubular acidosis
d. cystinosis

4. recurrent UTI with hypertensio (renal artery stenosis history)


investigation next step?
a. DMSA
b. Renogram
c. mag 3
d. doppler u/s

Jan 2019
1. A child with low sodium, low k , metabolic acidosis,urine glucose + ,
protein + :
a. Cystinosis
b. Galactosemia

2. A picture of MCUG of 5 months showing reflux and hydronephrosis,


have UTI, febrile, what to do next
( it was similar to this picture but
unilateral):

a. DMSA
b. MAG3
c. Cystoscopy

182
Nephrology

2. A case with puffy eyelids, history of upper respiratory tract infection FH of


dialysis can't close jeans, protein ++++, blood +++, hypertensive, high RFT:
3 questions:
A. Next step?
a. fluid restriction and follow up UOP
b. nifedipine
c. referral for urgent nephrology outpatient
B. What aids your diagnosis?
a. Complement level
b. u/s abdomen
C. We gave her nifedipine, BP got higher180/100, subconjunctival hge , ttt?
a. labetalol IV
b. increase nifedipine

3. A baby born at 32 weeks, nephrocalcinosis, ↑ calcium/ creatinine ratio,


no family history of similar condition?
a. idiopathic hypercalciuria
b. Barter
c. wiliam syndrome

Sept 2018
1. 10-days-old neonate, 3+ haematuria, removed recently UAC, UVC, what
investigation to confirm diagnosis?
a. doppler renal vein
b. urine C&M
c. serum urea creat electrolytes
d. coagulation
e. urine / creat ratio

183
Nephrology

2. Painless haematuria, continuing viral illness few days ago, no oedema ,


urine BL +++, protein +
a. nephrotic
b. IGA nephropathy
c. HSP
d. Fabricated
e. UTI
f. alport $

3. Neonate on ventilator, birth asphyxia ph= 6.8, weight = 1.7 kg , one of


twins . emergency-c-section, on day 3 develop abdominal distension
treated Abs ,blood culture -ve abdomen soft ,on day 7 develop bilateral
lower limb oedema,long saphenouse line , low serum albumin = 32 , low
globulines = 19 , low total protein
a. congenital nephrotic finish type
b. inf vena cava thrombosis
c. Fluid
d. overload line extravasation

4. EMQS
a. FSGN
b. Ig A nephropathy
c. Minimal change
d. Post strepto GN
e. Post infectious GN
f. HSP
g. SLE
h. Alport

A. 8-years with pain in knee,ankles, abdominal pain, throat infection,


non blanching rash ,+ protein and blood in urine HSP
B. visual defect, deafness, repeated haematuria, protein in urine
alport

184
Nephrology

C. 14 year with high Bp, low albumin, proteinuria ,microscopic haematuria ,


ECG show LVH, -ve PANCA..C ANCA, normal complement and
Immunoglobins FSGNs

5. Neonate female 6 month old vomiting, tetany , bl pr 82/55 low NA , low K


a. barter
b. CF
c. pyloric stenosis
d. renal artery stenosis
e. primary hypoaldosteronism

6.

Nephrogenic DI

May 2018
1. term baby with mass and UAC and UVC inserted urine shows blood
+3, protein plus?
renal vein thrombosis

185
Nephrology

2. CT abdomen asking from were the mass is coming?


Kidney Wilms tumor

3. a child 3 years girl, with vomiting, fever, with UTI, WBC in urine 10, mixed
organism in urine they asked what to do next?

a. clean catch specimen


b. bag of urine
c. suprapubic aspiration
d. start trimethoprim

4. 2 years old with long history of constipation, vomiting, and height less
than 9th centile, weight less than 2nd centile . PH-(acidosis )
a. renal tubular acidosis
b. Pyloric stenosis
c. Barter syndrome
d. Chloride losing diarrhea

5. child newly diagnosed with nephrotic syndrome and oedema and ascites
what to do?
Prednisolone 60 mg + penicillin v for pneumococcal prophylaxis
(1st presentation only)

Jan 2018
1. High serum CL + low Na, K + urine glucose (+) and a.a (+)
Mostly Fanconi

186
Nephrology

2. Child known CF presented with vomitng labs showed Na 108 K 2.3. Bicarb
37, what is your diagnosis?
a. Pseudo-barter
b. Nephronenic DI
c. Pyloric stenosis
d. CAH
e. SIADH

Sept 2017
1. haematuria +1 brother sister had renal failure and the mother
adult polycystic kidney disease AD

2.
A. why urine output decreased in first 2 hr of Water deprivation test?
a. increased DDAP secretion
b. sodium absorption
c. decrease aldosterone
B. condition was?
Central DI

3. renal failure with hearing loss


alport syndrome

4. Fever, after exercise and proteinuria+


Stationary protnuria

May 2018
1. child had an electric shock then developed haematuria
Rhabdomyolysis

187
Nephrology

2. 3-months infant with morning eye puffiness, loose stool anaemia Hgb 9 ,
WBCs 5-6 no available differential, Urine analysis : proteinuria,
Hypoalbuminemia
a. Congenital nephrotic syndrome
b. Schwachman diamond syndrome

3. child with meningococcal sepsis and low urine output, low Na, high urine
Na, urine and serum osmolality were given, what of the findings support
renal than pre renal causes of low Urine out put?
a. Serum Na
b. Urine Na
c. Urine/plasma creatinine ratio
d. Urine plasma urea

Jan 2017
1. water deprivation test. . . With values.
A. diagnosis >>central DI.
B. mechanism. . Increase of ADH in 1st 2 hours

2. case history with lab values all going with Fanconi syndrome

3. a case of PUV. . Most appropriate urgent action?


a. MCUG
b. Electrolytes

4. a case of haematuria. . F/h of deafness in uncle. . IgA level normal


a. alport syndrome
b. berger disease

188
Nephrology

5. Case of proteinuria. . Ask about investigations


Spot for ptn/creatine ratio

6. long case scenario with lab.. Met alkalosis.


barter syndrome

May 2016
1. Diabetic mother, her child has trace of glucose & ketone in urine, blood
glucose is in normal range
Renal glycosuria

2. Case of IgA nephropathy?


A. Investigations?
B. When to refer to nephrologist?

Jan 2016
1. Anuria , high urea& creatinin
a. DIC
b. HUS
c. Renal vein thrombosis

May 2015
1.There was data of water deprivation test showing
psychogenic polydipsia

2. a data question asked diff between pre and post renal failure
pre : NA in urine low<20
renal : NA in urine is high

189
Nephrology

3. there was a child with hematuria and HTC. Management?


a. labetalol
post GBS nephritis — refer
b. fluid restriction
inv : c3 and c4
for HTN we give labetalol

4. EMQ: about proteinuria


A. Alport syndrome Transient proteinuria is the temporary excretion of
protein and can be caused by strenuous exercise, a high
B. Transient proteinuria
fever, exposure to cold, stress and other conditions.
C. AD polycystic kidney disease Pregnant women may also excrete more protein in their
urine. Transient proteinuria does not involve underlying
kidney disease and requires no treatment.

Sept 2014
1. Data (suggested central DI) for 8years girl presented with polydipsia and
polyurea Water depriva5on test performed
Q1:
What cause of decrease urine volume in first 2 hour of the test?
a. Increased DDAP secretion
b. Increase sodium absorption.
c. Decrease DDAVP
d. Increase water reabsorption.

Q2:
Possible tool of investigation might help?
a. Skull x-ray
b. MRI brain.
c. Abd. U/S.

190
Nephrology

Jan 2013
1. 6 yrs old with learning difficulties presented with 24hr history of vomiting
and drowsiness. labs were Na 180, K 4.3 ,Cl 130 , urea and creatine normal
urine osmolality: 180 , urine sodium : 75
Q1. what is the likely diagnosis?
a. hypernatremia dehydration
b. salt poisoning
c. Diabetes insipidus
Q2. next step of management?

191
Cardiology

192
Cardiology

Sept 2021
1. EMQ
A.VSD
B. ASD
C. INNOCENT MURMER
D.PDA
E. MITRAL INCONTINANCE
F. PS
G.INNOCENT VENOUS HUM
H. Coarctation OF AORTA
A. Systolic 2/6 musical murmur, heard in lower left sternal border, best heard
when patient is lying. INNOCENT MURMER
B. Pansystolic 3/6 murmur best heard in apex of heart and radiating laterally.
MITRAL INCONTINANCE
C. Continuous 2/6 murmur heard on infraclavicular area that changes with
position. INNOCENT VENOUS HUM

2. Scenario of pt. had fever, tachypnea, tachycardia, enlarged liver, tender


upper abdomen, and dyspnea on exertion. O/E: apex on the sixth
intercostal space. What is the
diagnosis?

a. Dilated cardiomyopathy
b. Viral hepatitis
c. Pneumonia

193
Cardiology

3. Scenario of teenage girl who was taking caffeinated drinks then became
dizzy, LOC, her mom said that she was a bit pale when she found her.

a. AF
b. RBBB
c. SVT
d. Vasovagal attack

4. X-ray chest of newborn 3 days old. He needed mask ventilation for 2min,
after 1 day developed dyspnea, tachypnea, O2 sat 91% & systolic
mummer. What is the Dx?

a. TGA
b. TOF
c. Pulmonary stenosis

194
Cardiology

5. 14 years old with cQT 490 msec. What is the treatment?


a. Enalapril
b. Propranolol
c. Atenolol
d. Digoxin
e. Amiodarone

6. 12 yrs old had 6 months duration of emotional liability,clumsiness,


slurred speech and pansystolic murmur radiating to axilla. There was
abnormal movement in face and hands mainly.What is the diagnosis?
a. Wilson
b. Huntington
c. Psychosomatic
d. Cerebellar stroke
e. Sydenham chorea

May 2021
1. A newborn, just delivered and cardiac catheterization was done which
showed:
What is the diagnosis? saturation pressure
a. TOF SVT 56
b. TGA with large VSD RA 54 4/2
c. TGA with atrial septostomy RV 55 85/
PA 78
LA 90 9/2
LV 78
Aorta 55

195
Cardiology

2. A newborn was delivered by an emergency CS for fetal tachycardia. What


does this ECG show?

a. Atrial flutter
b. AF
c. Av node re-entry pathway
d. VT
e. Sinus tachycardia

3.

What does this ECG show?


a. RBBB
b. R V hypertrophy
c. Left axis deviation

196
Cardiology

4. A case of Prader Willi syndrome with obstructive sleep apnea. What 2


abnormalities does this ECG show?

a. RVH
b. P-Pulmonale
c. Right axis deviation
d. Left atrial dilatation

Jan 2021
1. pale child (2 yrs) has tachycardia 180 tachypnea, retractions fine basal
creps with tender Hepatomegaly ..Post viral illness (URTI)
a. viral myocarditis
b. SVT

2. 3 years old child with small PDA discovered incidentally by echo which
done before elective surgery (tonsillectomy) management?

a. trans thoracoscopic surgery ligation


b. Transcutaneous occlusion by catheter
c. Ibuprofen
d. wait to close spontaneously
e. transesophageal ligation

197
Cardiology

3. EMQ:
A. VSD
B. AS
C. HOCM
D. COA
E. Peripheral pulmonary stenosis
F. Fallot tetralogy

A. girl neonate WITH MURMER AND PEDAL OEDEMA WHAT IS THE


HEART LESION
COA

B. Infant is 4.8 kg with pan systolic murmur over 4th lt ics on CPAP for
48 hrs after birth what is the cause of murmur?
VSD

C. Patient has jaundice with x-ray pic of butterfly vertebrae


Peripheral pulmonary artery stenosis

4. 12years with chest pain for 24 hours ECG was taken when the patient
lying down
a. Myocarditis
b. Pericarditis
c. LCAPA
d. viral pneumonia

198
Cardiology

5. ECG choose THREE options:

a. left vent Hypertrophy


b. LV strain
c. Rt ventricular hypertrophy
d. Lt axis deviation
e. heart block
f. Delta wave
g. Rt axis deviation
h. LBBB
i. Junctional rhythm

6. Preterm with RD. X-ray pic shows butterfly vertebrae, what is the
cardiac lesion?

a. Peripheral Pulmonary
stenosis
b. aortic stenosis
c. oarctation of aorta
d. VSD

199
Cardiology

Sept 2020
1. Female with webbed neck and wide spaced nipple who attends an
ordinary school but needs extra help, shorter than her peers, asking for
cardiac association?
a. Bicuspid aortic valve
b. Supra valvular AS
c. TOF
d. VSD
e. Pulmonary valve stenosis
f. Pulmonary artery stenosis

2. 14 years old girl presented with pallor and developed 2 episodes of


seizures, one with eyes rolling up (with white sclera only) and
unresponsive for 30 minutes ( I think the second seizure she had longer
post ictal) and after minutes develop episode with seizure with extended
limbs, before the episode she was helping her brother who hurt himself in
his hand, there was history of uncle have seizures, dx?
a. Vasovagal attack
b. Generalized tonic colonic seizures

3. 8 years girl with mixed ethnicity low grade fever came with sudden loss
of vision had large spleen below costal margin, she has tender bulbs of
the fingers on deep pressure and swollen fingers. Paronychia of the big
toe, ESR 40 what to do, two options?
a. ECHO
b. Blood culture
c. CT orbit
d. HB S haemoglobinopathy
e. Doppler us on calf muscle
f. Sickling test
g. Lupus serology

200
Cardiology

4. 10-months infants with history of attacks of pallor and sweating, he is


otherwise well, what is dx?
ECG is given (there was LVH in ECG)
a. WPW
b. LBBB
c. HOCM
d. Atrial fibrillation

5. A teenage child fell from chair in class yesterday. Today he falls again
from chair and become unresponsive and develops seizures then gets
back to himself after 2 minute, asking for initial investigation?
a. ECG
b. EEG
c. Sleep study for narcolepsy
d. Electrolytes

201
Cardiology

5. 13 years old child develops sore throat 15 days before admission, then 10
days develops haematuria, 6 days he develops face swelling and headache,
the parents treated by paracetamol now develops tonic colonic
convulsions and GCS was 11. Respiratory rate 20 Blood pressure 180/110
mmHg, urine 3+ blood and 3+proteim he was afebrile.
A. What is the cause of convulsions?
a. Systemic HTN
b. Encephalitis
c. Cerebral vasculitis
d. Brain abscess
e. HSP
B. What treatment to give?
a. Labetalol
b. Phenytoin
c. Acyclovir and IV antibiotics
d. Mannitoll
e. Steroids
f. Intubation and ventilation

7. ECG of a girl presented with fever diagnosis?


a. Complete HB
b. First degree HB with prolonged PR
c. Extra vent systole
d. Sinus Bradycardia

202
Cardiology

Jan 2020
1. EMQ:
A. infraclavicular continuous murmur change with position
Innocent venous hum
B. pansystolic murmur radiating laterally
Mitral incompetence
C. systolic murmur grade 2/6 change with position
Innocent murmur

2. ECG of adolescent girl


a. long QT
b. ventricular hypertrophy

3. 20-months girl with VSD, tiredness after feeding with congestive H


already on oral captopril, frusemide, spironolactone, referred because of
decrease of weight below 2nd centile, on maximum bottle feeding but
tired easily Tachycardia, cx ray cardiomegaly plethora, Na 132, k 3.2
urea 6.5, creatinine normal, Hb 10, what best management?
a. Decrease feeding 80%
b. Decrease fluid
c. Used high caloric formula
d. increase no of feeding
e. iron supplement

203
Cardiology

Sept 2019
1. new born baby 5-days old with x-ray boat
shaped heart and oligemic lung, history of
baby need mask ventilation for 2-min after
1-day developed dyspnea tachypnea
sat 91% & systolic mummer
a. TOF
b. PS

2. Given 4 limp BP Which was high in upper right limp and low on left
lower (pre and post ductal spo2 and BP) high creatinine and urea ask
diagnose
a. cortication of aorta
b. renal artery stenosis
c. CRF

3. patient with intermittent cyanosis in finger, otherwise well, improve


when putting hand and limp in warm water
(Raynaud phenomena)
A. what investigation?
no need for investigation
B. what to do? need one option
a. advice avoid nicotine and caffeine
b. nifedipine
c. nitroglycerin patch
d. low dose aspirin

204
Cardiology

4. ECG in paper one baby deliver with C/S due to foetal tachycardia
(check hr regularity to reach diagnose)
a. AF
b. A trial flutter

5. 12-yrs ECG in paper 2 not clear but patient was having chest pain
a. ST segment elevation with inverted T wave
b. VT

205
Cardiology

6. one question about baby 8-yrs with tachypnoea tender hypochondrium


difficult to exam the abdomen ,,fever,, vomiting,, apex was in
6 intercostals space,, liver palpable ,,picture of x-ray what diagnose?
a. dilated cardiomyopathy
b. acute appendicitis
c. pancreatitis
d. hepatitis

May 2019
1. EMQ
A. VSD
B. COARTICATION OF AORTA
C. AS
D. PS
E. HOCM
F. PPAS
A. girl neonate WITH MURMER AND PEDAL OEDEMA WHAT IS THE HEART
LEISION? COA

B. Infant of diabetic mother 4.9 kg with pansystolic murmur over 4th lt ics
on CPAP for 48 hrs after birth what is the cause of murmer VSD
C. patient with butterfly vertebrae Peripheral Pulmonary artery stenosis

2. HISTORY OF FAINTING most serious


a. WPW
b. Rbbb

206
Cardiology

2. 3-year old girl with Prader willi syndrome snore at sleep.


ECG (2 options)

a. RXD.
b. Delta wave
c. Prolonged QT
d. p.pulmonale
e. RVH

3. patient with suspected Infective endocarditis past history of fever


1-month now (afebrile).. irregularity in the echo.. diagnosis?
a. Repeat culture 30min
b. repeat culture wait result to start antibiotics
c. heparin
d. culture with peak of fever
e. 2 culture 30 minutes apart and start antibiotics
(we think he mentioned did one culture )
Jan 2019
1. A case of enlarged adenoids and obstructive sleep apnea, high bp, what
is the cause of cardiomegaly?
a. left ventricular hypertrophy
b. Right ventricular hypertrophy
c. pericardial effusion
d. biventriculer hypetrophy

207
Cardiology

2. 3 years old with small PDA discovered incidentally as a continuous


murmur in the Lt upper sternal edge, ttt:
a. trans-thoracoscopic surgery ligation
b. Transcutaneous occlusion by catheter
c. Ibuprofen
d. wait to close spontaneously
e. transoesophageal ligation

3. ECG: A teenager took heavy caffeinated fluids, became dizzy , loss of


consciousness,her mom said that she was a bit pale when she found her ?

a. AF
b. Vasovagal attack
c. long qt

4. 6 weeks old with cyanosis, ejection systolic murmur at the upper left
sternal edge and single second heart sound, lung fields are clear,
diagnosis?
TOF

5. NEONATE with SVT, HR=300 not shocked:


a. ice back to the face
b. IV adenosine

208
Cardiology

6. ECG: ( 3 Answers)

a. left vent Hypertrophy


b. LV strain
c. Rt ventricular hypertrophy
d. Lt axis deviation
e. heart block
f. wpw

Sept 2018
1. 5 yrs old boy, absence seizers, ejection systolic murmur on pulmonary area
ECG?
RBBB

209
Cardiology

2. Cardiac catheter:
RA = 54 , 4/2
RV = 55, 85/5
Pulm = 78 , 33/15
LA= 90 , 9/2
LV = 78 , 60/5
Aorta = 54 , 80/50
a. TGA with large vsd
b. TGA after septoplasty
c. TOF

3. ECG 12-yrs-old child with chest pain, on lying down it showed ST segment
elevation, diagnosis?

a. pericarditis
b. myocarditis
c. anomalous left coronary artery

210
Cardiology

4. Scenario with cyanosis & heart murmur & x ray boot shape heart

TOF

May 2018
1. 14 years old with collapse in gymnasiums?
12 leads ECG collapse after exercise mostly long QT syndrome

2. 6-weeks-old unwell with ventricular tachycardia, he was given glucose and


insulin and salbutamol, amiodarone, K=6.7 , the patient was unwell with
pulse 200 b/m and in VT, asked what to do?
Cardioversion

3. ECG -- complete heart block

211
Cardiology

4. ECG:
WPW

5. a child was running to his mother when she came to visit him at school
but suddenly fell down and loss his consciousness when he came to
hospital ,what you are going to do?
a. CT brain
b. 12 lead ECG (I think it was prolonged QT)
c. ECHO
d. EEG

6. cardiac catheter data:

Saturation Pressure
SVC=60 %n RV= 40/20
RA =60 % PA =40/20
RV= 80% LV = 120/75
PA = 80% AO =120/ 75
AO = 96%

VSD with pulmonary HTN *

212
Cardiology

Jan 2018
1. ECG i think same as in AKP book
2. 6-months unoperated TOF not on ttt developed blue episode on crying
spo2 88 what to do 2 options?
a. admission to hospital
b. Urgent EEG
c. Verapamil
d. oral beta blockers
e. morphine
f. buccal midazolam

3. EMQ:
A. pansystolic at apex radiating to lateral side-- mitral reg
B. grade 2 ejection systolic murmur medial to apex increased by supine
and decreased by standing --- innocent
C. grade 2 continuous murmur below clavicle disappear by lying flat
venous hum

Sept 2017
1. Chest x-ray preterm baby chronic lung disease➔
a. Scimitar—PAPVD

2. TOF with cyanotic spells sleeping in between with history confusing


a. cyanotic spills
b. convulsion

213
Cardiology

3. neonates may present with cyanosis, pulmonary hypoplasia,


heart failure, cardiomegaly
Epstein

4. Girl with Turner - increased Bp?


Echo

5. young girl 3 episode of fell down in bathroom unconscious without jerky


movement- ? ECG ?
Prolonged QT syndrome
From beginning of Q till
end of T > 13 small squares

6. ECG-Aberrant coronary artery

214
Cardiology

7. The photo of the central line, what's the place of the tip??
Right brachiocephalic vein

8. Question of the cardiac catheter:


VSD with pulmonary hypertension

9. ECG-RVH and obstructive sleep apnea

215
Cardiology

May 2017
1. ECG new born irritable ST elevation
anomalous coronary artery

2. girl fainting episode when she get up to drink she saw black color in front
eyes ..can’t remember what happened …..there is ECG normal
a. long QT ?
b. Vagal attack

3. pale child tachycardia 180, Post viral illness, Has Hepatomegaly


a. viral myocarditis
b. SVT

4. CXR with scenario Pneumopericardium, There was shadow around heart


asking for management?
small— no ttt
Large — precardiocentisis then search for the cause

5. PROLONG QT TTT???
Propranolol

216
Cardiology

Jan 2017
1. ECG question, History of collapse with exercise, Family history of
previous death long QT syndrome

2. ECG picture with history of arrhythmia . . Initial slurring of QRS complex


was evident
WPWS

3. case history with upper & lower limb blood pressure measurements.
Difference about 20mmhg between upper and lower , Impaired Renal
function . . Diagnosis?
coarctation of Aorta

4. newborn, cardiomegaly, cyanosed asks about diagnosis


Epstein anomaly

217
Cardiology

May 2016
1. Picture of webbing neck girl with learning difficulties ask about cardiac
lesion
suspected?
a. supravalvular aortic stenosis
b. bicuspid aortic valve (if turner confirm)
c. VSD
d .peripheral pulmonary stenosis (if Noonan

2. ECG 15-year-old boy sudden collapsed with Hx of uncle died suddenly


HOCM (hypertrophic obstructive cardiomyopathy)

3. Cardiac catheterization of VSD and pulmonary HTN


Eisenmenger syndrome or VSD with RT to left shunt

218
Cardiology

4. Case presented with abdominal pain, vomiting , apex of the heart on 6th
space , CXR show big heart ?
Dilated cardiomyopathy

Jan 2016
1. Pt with truncus arteriosus waiting surgery cardiology team put him
on diuretics he presents with O above 95 % and high lactic acid with
low PH what is the explanation for high lactic acid:
a. Chronic diuretic use
b. prolonged poor perfusion to the kidney
c. Lung perfusion is more than systemic

2. ECG of 5 y girl present with fever


a. heart block first degree with prolong PR
b. Complete heart block
c. Sinus rythm

May 2015
1. A child with collapse..ecg showing torsades/VT..management
DC shock
[N.B. if pt stable Mg sulphate if not DC

219
Cardiology

2. ECG with left axis deviation and RBBB


ASD

N.B.

LVH+RBBB

if young age:

partial AVSD

if OLDER = CoA

RBBB+ RAD =ostium


secundum ASD

RBBB+LAD= ostium
premium ASD

3. ECG ,there was RVH and RAD, asked dx

The ECG above belongs to a 3.5 years-‐old boy who has both pulmonary stenosis and an 18mm wide
non-‐restrictive type ASD.

This patient has both right ventricular (RV) volume overload due to non-‐restrictive type ASD and RV
pressure overload due to pulmonary stenosis.

The heart rate is 118/min. (normal for his age). There is right axis deviation. The increased amplitude
of R’ wave in C1 suggests RV hypertrophy.

220
Cardiology

4. CXR which looked like Figure of 8/snowman..dx?


a. partial APVD
b. TAPVD
b. lymphoma
c. teratoma

5. Cardiac catheterization data showed TGA with left to right shunt at vent
level..options were TGA with VSD,TGA post septostomy
TGA cardiac cath findings: pressures and sats of PA match the LV,
aorta match RV. (Switch

6. A child with PDA..CANT REMEMBER THE AGE..question was management


a. wait to close
b. indomethacin
c. Ligation
d. percut obliteration

7. An x-ray which showed subcutaneous air and pneumopericardium I think

221
Cardiology

8. There was a quest related to prophylaxis for infective


9. a child with fever and a murmur..prolonged hx of fever..BC
negative..further investigations..
a. repeat cultures
b. angio
c. echo

Sept 2014
1. Chest x-ray Scimitar sign

2. ECG:
Anomalous left coronary artery from pulmonary artery ALCAPA

deep Q in V and aVL


raised ST in V5,6

222
Cardiology

3. ECG of pt. with snoring? RVH


Right ventricular enlargement – corepulmonale --- high R in V1
Obstructive sleep apnea upright T in V12

4.Cardiac catheterizetion finding--‐?


VSD with Left to R shunt with Pulmonary HTN

5. 3-month-old baby cyanosis not improved by resuscitation x-ray,


cardiomegaly+ small lungs? Ebstein anomaly

6. TOF with cyanotic spells sleeping in between 2-answer needed


a. EEG
b. Morphine
c. b-blocker

7.Girl with Turner syndrome, increased Bp , investigation Echo


Jan 2013
8. EMQ:
what the cardiac lesions associated with syndromes
A. allagale syndrome

223
Respiratory
ENT

224
Respiratory & ENT

Sept 2021
1. X-ray chest asking about the findings. 2 answers: (different photo)

a. Lingual consolidation
b. Rt middle lobe consolidation
c. Rt upper lobe consolidation
d. Lt hailer lymphadenopathy

2. Pulmonary function test:


FEV-1 = 1.15
FVC = 1.55 (N= 1.5: 1.65)
RV = 0.65 (N= 0.5: 0.6)
What is the diagnosis?
a. Normal
b. Asthma
c. Cystic fibrosis
d. Scoliosis

3. Infant with brief acute febrile illness and cough, his x-ray chest is shown.
What is next step?

a. CT chest
b. Us chest
c. No investigation
d. Urinary
catecholamines

225
Respiratory & ENT

4. A 12-week-old baby was found by his parents blue and apparently lifeless
in his cot after they had heard him choking. His father revived him with
mouth-to-mouth resuscitation and brought him to the hospital. He was
born at 36 weeks gestation weighing 2.4 kg. He had no perinatal problems
and fed well by bottle although he frequently regurgitated his feeds. His
parents reported that he had been unwell with a cough for a few days
before admission. His two siblings aged 5 years and 3 years had both been
unwell recently with chest infections. Father had been unemployed for
18 months and there were considerable financial problems. On
examination he was pale and had a nasal discharge. His weight was on the
10th centile. Slight intercostal recession and a respiratory rate of
40/minute were noted together with scattered fine crepitations and
a high pitched expiratory wheeze. The remainder of the examination was
normal. Examination of the urine revealed no protein or sugar and no
cells were seen on microscopy. A chest x-ray showed a degree of
overinflation with no localised opacities.
What is the most likely diagnosis?
a. congenital cardiac abnormality
b. cystic fibrosis
c. gastro-oesophageal reflux
d. milk aspiration
e. pneumonitis pertussis infection
f. viral bronchiolitis

5. Preterm neonate was intubated for 5wk now 5month, treated for
recurrent pneumonia, O/E: high arched palate, X-ray show patchy
change. What is cause?
a. Recurrent aspiration
b. GOR
c. TOF

226
Respiratory & ENT

6. Scenario of a patient with cough and high-grade fever who received ABx
without improvement. On examination was decrease breath sound on RT
side. X-ray chest was done. What is the first step?
a. Bronchoscopy
b. US chest
c. Aspiration from
pleural effusion

May 2021
1. EMQ
A. Bacterial tracheitis
B. Laryngotracheobronchitis
C. Viral-induced wheeze
D. Bronchiolitis
E. Vascular ring
F. Laryngomalacia
G. FB
A. A 3-year-old boy with a 2-week history of cough. Auscultation revealed
wheezes on the right side of the lung.
FB
B. A 6-month-old boy presents with worsening of inspiratory and
expiratory sounds which started at the age of 3 months.
Vascular ring
C. A 2-year-old boy with a 2-day history of cough,inspiratory noisy
breathing, a runny nose and fever of 38.1 C’. RR was 60/min and there
were mild intercostal retractions
Laryngotracheobronchitis

227
Respiratory & ENT

2. A 4-year-old child who was ventilated postoperatively, (PIP 18, PEEP 5,


Fio2 0.6). He developed hypoxia. On examination there was equal air entry
bilaterally and the tube was clear. The chest was not rising adequately but
chest movements improved on bag mask ventilation. Spo2 90% , PH: 7.2 ,
PCO2: 7.5 , PO2: 5 , HCO3 :32 What is the next step in management?
a. Increase PEEP
b. Increase PIP
c. Decrease ventilatory rate
d. Change ETT
e. Give nitric oxide

3. A 2-year-old girl presents with cough and fever for 2 days then she
developed biphasic stridor and was unable to take anything by mouth. Her
RR was 50/min with Spo2 of 93%. What is your management action? Select
one answer.
a. Humidified O2 only
b. Nebulized budesonide
c. Nebulized adrenaline
d. I.V adrenaline
e. Nebulized saline

4. A patient from Pakistan, came with fever and cough for 1 month. He was
given oral antibiotics without improvement then a chest Xray was done.
What is the next investigation to request?

a. Sputum C/S
b. 3 early morning gastric
wash?
c. Bronchoscopy with
bronchoalveolar
lavash?
d. Mantoux test

228
Respiratory & ENT

5. The following is an X-ray of a... year-old child with respiratory tract


infection who received oral antibiotics without improvement.
What is the next investigation?

a. US chest
b. CT chest
c. Pleurocentesis
d. Blood C/S

6. A 4-year-old child with a history of viral infection 6 weeks ago after which
he developed hyperinflation, generalizes wheezing and clubbing. What is
the diagnosis:
a. Bronchial asthma
b. Bronchiolitis obliterans
c. Bronchiectasis

7. A preterm baby with diaphragmatic hernia that was corrected surgically,


presents with respiratory distress and milky aspirate. X-ray showed
opacification of the hemithorax. What is the cause?
a. Injury to the thoracic duct
b. Pleural effusion

229
Respiratory & ENT

8.
A. A 5-years-old boy with history of Hemophilus influenzae meningitis
at age of 4 years which improved on antibiotics. This AUDIOGRAM
was done. Comment on the left side.

a. Mild SNHL
b. Moderate SNHL
c. Severe SNHL
d. Conductive hearing
loss

B. The child had a history of 2 previous attacks of chest infection. He


was first diagnosed as having asthma but did not improve with
asthma medication and now he presented with fever, cough and
tachypnoea. Sputum C/S showed pseudomonas What does this X-
ray show?

a. Right sided hyperinflation


b. Left lower lobe consolidation
c. Right sided pneumothorax
d. Left sided pleural effusion
e. Pneumatocele

9. A patient with annual allergic rhinitis was not responding to oral


antihistamine. What is the next step in management?
a. Oral Prednisolone
b. Intranasal steroid
c. Intranasal decongestant

230
Respiratory & ENT

10. A 3-year-old presents with his grandmother with this picture, what is the
diagnosis?

a. NAI
b. Mastoiditis
c. Eczema herpeticum
d. TB adenitis

Jan 2021
1.
A. The case with the nasal polyps and recurrent OM and x ray with hilar
lymphadenopathy and clubbing? 100% respiratory
a. CF
b. 1ry ciliary dyskinesia
c. Bronchiectasis
d. Hypogammaglobinemia

B. What is X-ray finding?


a. Middle lobe collapse
b. Middle lobe consolidation

2. Infant has biphasic stridor increases with supine position with


wheezing on auscultation + frequent vomiting and multiple admission
in 10 days.What’s the investigation?

a. Barium swallow
b. High resolution CT chest
c. Echo
d. CXR
e. Laryngo-tracheo-bronchoscopy

231
Respiratory & ENT

3. Asian boy with weight loss, prolonged fever, what’s X-ray finding?

a. Consolidation
b. Rt hilar
Lymphadenopathy
c. Interstitial infiltration

4. 18 months old came with symptoms of bronchiolitis, asked for X-ray


finding?

a. wide mediastinum
b. posterior rib fracture
c. rachitic rosary

5. Child with tibia fracture with cast came after 6 hrs with sudden dyspnea,
raised jvp ? chest x ray pic

a. fat embolism
b. pulmonary embolism

232
Respiratory & ENT

Sept 2020
1. X-ray picture of a term baby who needed bag and mask ventilation on
resuscitation only, after 2 hours become tachypnoea and respiratory
distress SPO2 90 % to 92 % on oxygen 40 %
What is the abnormality in
the x ray?
(Left side is completely opaque, right
lung has small triangle of air in the
bottom)

a. Diaphragmatic hernia
b. Cardiomegaly
c. Ground glass appearance
d. Pneumothorax
e. Left pleural effusion

2. 8-years known Sickle cell came with shortness of breath and fever
started on antibiotics O2 saturation 94% with this X-ray of chest, dx?

a. Lingual consolidation
b. Left lower lobe
consolidation

233
Respiratory & ENT

3. Chest x-ray of a child had pneumonia in Rt. lower lobe 4 weeks ago and
treated, now presented with cough only, management?

a. US diaphragm
b. CT chest
c. Chest tube
d. CT thorax
e. Urgent referral to surgeon

4. 8 weeks infant, he was PT delivered at 29 weeks. Now at 39 weeks


corrected age, he came with the following ABG in room air: PH 7.36, Hco3
38, Co2 8 what is the cause?
a. Broncho Pulmonary Dysplasia
b. Bartter
c. CHD
d. Hypertrophic pyloric stenosis

5. An infant was found breathless in his cot, few days ago he had nasal
discharge, his father tried to resuscitate by mouth-to-mouth breathing,
Father is unemployed, 2 siblings have common cold, x-ray showed
hyperinflation, what to do?
a. Nasopharyngeal swab Sample paper
b. Per-nasal swab

234
Respiratory & ENT

6. X-ray chest as below. Heart sounds are heard on the right,


What is the diagnosis?

a. Congenital lobar emphysema


b. CCAM
c. Diaphragmatic hernia

Jan 2020
1. Known acute asthmatic attack on salbutamol came by lactic acidosis what
is the cause?
a. IV Salbutamol
b. Mg so4
c. Aminophylline
d. Hydrocortisone
e. Ipratropium bromide

2. Indication of cochlear implant?


a. bilateral CHL>90db
b. Bilateral SNHL >90 db
c. unilateral SNHL>90 db
d. bilateral SNHL more than 50

235
Respiratory & ENT

3. picture of pneumatocele Staph infection, ask about the organism

4. Scenario of pleural effusion with loculated what to do?


a. chest drain
b. Chest drain with fibrinolytic
c. Antibiotic

5. picture of 9-month child persistent respiratory symptoms


a. Congenital cystic adenomatoid malformation (CCAM)
b. diaphragmatic hernia
c. consolidation

236
Respiratory & ENT

6. Chest x-ray ,12yr with cough, clear sputum, receive Ab no improvement?


a. bronchial thickening
b. consolidation
c. lingular pneumonia
d. superior mediastium
e. Hilar lymphadenopathy

7. 7 years old child with pulmonary function test


FEV1 1.15 (1.15 - 1.6)
FVC 1.55 (1.40 - 1.8)
PEF 165 (190 – 220)
RV 0.65 (0.35 - 0.60)
Total lung Capacity 2.20 (1.85 - 2.25)

A. What is the cause of increase RV?


a. Poor expiratory effort
b. Air trapping
c. Mucous plug
d. Equipment failure
e. Poor inspiratory effort

B. What is the diagnosis?


a. Bronchial asthma
b. CF
c. Scoliosis
d. Duchenne Muscular dystrophy
e. Normal child

237
Respiratory & ENT

8. preterm 30 weeks oxygen dependent admitted at 3-months with


bronchiolitis & at 6-months with fine crackles &low oxygen, causative
organism?
a. RSV
b. Adenovirus (bronchiolitis obliterans)
c. Pertussis
d. Para-influenza

9. Scenario of cystic fibrosis and given ceftazidime, tobramycin , normal IgE ,


high eosinophils, deteriorating what is the cause?
Allergic bronchopulmonary aspergellosis in (sep 2017)

Sept 2019
1. 5-yrs old boy with history of fever & cough and shortness of breath
received 7-days of amoxicillin but didn’t improve?
a. RT upper lobe pneumonia
b. Lymphoma
c. T.B
d. Teratoma
e. Thymus

238
Respiratory & ENT

2. x-ray neonate with shortness of


breath
a. left congenital emphysema
b. pneumothorax
c. hypo-plastic left lung
d. upper lobe collapse

3. patient with fever and cough and x-ray


of rounded homogeneous opacity
a. right rounded pneumonia
b. upper lope pneumonia

4. x-ray need diagnosis patient with sudden onset of chest pain, unstable
a. pnemo-mediastinum
b. pulmonary edema
c. loeffers syndrome
d. millary

Pneumomediastinum

239
Respiratory & ENT

5. baby has been well and there


was decoration at home,
present with cough and wheeze
and decrease air entry in left
(x ray picture)
a. right side foreign body
b. left side foreign body

6. one question of patient with cough for 3-months and has cough and
apnoea WBCS 17 and mainly lymphocytosis ask treatment
a. Clarithromycin pertussis

7. Scenario about patient they mention that chest x ray show white out
(no x-ray picture) ,,with dullness ,,decrease air entry in percussion I
think ask what to do with decrease?
a. u/s chest
b. diagnostic aspiration
c. CT chest scan
d. Antibiotic

8. one question baby has upper respiratory tract infection and other
sibling has same has apnoea not remember??(sample paper question)

a. nasopharyngeal aspiration Bronchiolitis


b. throat swab
c. skeletal survey

240
Respiratory & ENT

9. one case cystic fibrosis receive correction for Na 135 and k 2.5
Discharge came again after 5-days with low Na 134 and k 2.9
what to give?
a. oral Na and k
b. FLUID RESRTCTION
c. ORAL K
d. Desmopressin

10. CXR of pt with signs of bronchiolitis (9 ribs above dipheragm)


a. lt side pneumothorax
b. hyperinflation

May 2019
1. x-ray for patient with pneumonia
with left opacity asked about next step

a. Chest US
b. CT
c. Bronchoscopy

241
Respiratory & ENT

2. patient with diaphragmatic hernia REPAIR deteriorated with milky drain-


a. lymph rupture
b. Injury thoracic duct

3. x-ray need 2 answers


a. Lingula consolidation
b. Rt middle lobe opacity
c. plural effusion

4. Asthmatic child came with sever attack, received salbutamol ,hydrocort iv,
but still tachypneic ,tachycardic with metabolic Acidosis
PH.7.25 ..RBs 22 , Lactate 7 mmol.... AIR ENTARY is good
WHAT IS THE CAUSE OF HIS BREATHLESSNESS??
a. salbutamol toxicity
b. DKA

5. teenage known asthmatic patient came with cough for 1 month


pneumonia receive ttt but not improved have yellwish fruity sputum
(mycoplasma scenario) there is x-ray with no localised consolidation
exercise intolerance what is the diagnostic investigation?
a. mycoplasma serology
b. CT chest
c. Bronchoscopy
d. lung function test

6. previous viral infection came with clubbing now severe wheezes, diagnosis
a. Bronchiolitis obliterance
b. Broncictasis
c. CF
d. Mycoplasma

242
Respiratory & ENT

7. female CF with vomiting after each meal and abdominal distension


metabolic alkalosis
a. Psuedobartter
b. hypertrophied pyloric stenosis

8. patient known diaphragmatic hernia, transillumination test done


negative improved with manual ventilation, po2 low, pco2 high,
ventilator sitting on minimum
a. change tube
b. increase pip
c. increase peep

Jan 2019
1. CXR with cough, fever:
a. left inferior lobe consolidation
b. lingula consolidation
c. loculated empyema

2. A 8 YRS on vent, High CO2 , High O2 , inspiratory time .08 VR 20 ...PIP 28 ,


PEEP 6 PO2 20 ,PCO2 "15, what to do?
a. Inc Rate
b. Inc PEEP
c. decrease PEEP

243
Respiratory & ENT

3. CXR:(2 Answers):
a. left pulmonary interstitial emphysema
b. surgical emphysema
c. upper lobe consolidation

4. patient have left side positioning and abnormal sound at night, normal
EEG, behavioural abnormality
a. Obstructive sleep apnoea ??
b. Rolandic epilepsy
c. frontal epilepsy??

5. CXR of massive right pleural effusion, tracheal shifting, next


investigation?

a. needle aspirate
b. ultrasound chest
c. CT chest
d. needle aspiration

6. An asthmatic boy with 4 weeks paroxysmss of cough, losing weight


fever, a lot of investigation the only positive finding was wbcs 24
lymphocytosis 16:
a. TB
b. Pertussis

244
Respiratory & ENT

7. A child with acute severe asthma, took salbutamol nebuliser and IV, IV
hydrocortisone, mg sulphate has hypoxia, metabolic acidosis, ↓ k , ↑
lactate 6 ,air entry is ok what is the best action?
a. CXR
b. intubate and ventilate
c. bolus if Mg sulphate
d. Taper and wean off salbutamol
e. Aminophylline
f. correct potassium

Sept 2018
1. 12-yrs-Pakistani, BCG vaccinated , dyspnoea on playing , salbutamol no
improve , persist cough , what are 2 finding in x ray ?

a. RT hilar LNs
b. ghons complex
c. RT mid lobe consolidation
d. fluid in rt middle fissure

245
Respiratory & ENT

2. Long history of pneumonia had oral antibiotics not improved then come
back for iv antibiotics for 48 hrs still no improvement, ultrasound chest
loculated pleural effusion 5 cm, what to do ?
a. insert drain & fibrinolytic
b. insert chest tube
c. continue antibiotics

3. 3-yrs old Child with repeat 2 episodes of otitis media , delayed speech
deafness of 70 db high frequency(means sensorineural)
next best 2 steps ?
a. ENT for hearing aid
b. ENT for gromet insertion
c. speech therapy
d. MRI

4. 12-yrs old Girl with sudden short of breath , picture of x ray chest ,what’s
finding 1?
a. RT hyperinflation
b. rt pneumothorax
c. lt lingula collapse
d. lt lower lobe collapse
e. rt middle lobe consolidation

246
Respiratory & ENT

5. Child with long bone leg fracture with cast came after 6 hrs with sudden
dyspnoea, high JVP? chest x ray pic

a. fat embolism
b. pulmonary embolism
c. sickle chest crisis

6. Neonate on ventilator deteriorate PIP = 18, PEEP = 5, RR= 30, FIO2 = 0.6,
PH =7.15, co2 = 10.7, o2 = 5.8, HCO3 = 27, transillumination normal.
what to do next?
a. increase pip
b. increase peep
c. increase rate
d. suction tube
e. change tube

May 2018
1. 5-years-old with asthma developed strange cough and had high
lymphocytosis? Pertussis

2. 12-weeks-old with chocking episode his father found him unconscious in


his cot and rescued with mouth to mouth berating from father , the baby
has nasal discharge , father unemployed , has 2 sibling have common
cold like symptoms ?
RSV bronchiolitis

247
Respiratory & ENT

3. baby with corrective surgery for left diaphragmatic hernia and start
breast milk by nasogastrc tube , in the third day deteriorate and now his
lt side is totally weight out in x- ray and on drainage of fluid its milky
colour ,what is the cause?
Injury to thorathic duct --- chylothorax

4. chest x-ray with PH probe in left main bronchus. asked what to do?
Remove it

5. x-ray with scenario patient was given amoxicillin but did not improve,
rt middle lobe consolidation then after couple of days he deteriorate ,
they gave us another x ray I think it was rt side pleural effusion asked
A. about how to diagnoses?
Ultrasound Chest
B. What’s your management?
a. Drain aspiration
b. Floxacillin --- in case of culture with staph

248
Respiratory & ENT

6. 6-years-old asthmatic on salbutamol and steroid 100 micg Bid inhaler not
controlled what to do?
a. salmetrol inhaler
b. Increase steroid dose
c. Montelukast
d. Oral corticosteroid

7. blood gas of 8-weeks old born at 29 weeks, High Pco2, bicarb =28,
PH= 7.3
bronchopulmonary dysplasia ---- resp acidosis

8. chest x-ray (air trapping) of child of 18 months with cough, the family
have decoration changes in
their house

a. left side foreign body


b. Tension pneumothorax
c. CCAM
d. Chemical pneumonia
e. Rt side foreign body

9. chest x- ray showing they asked


for diagnosis what to do and
next step in management
a. Middle lobe pneumonia
b. US chest
c. Chest drian

Not same picture in exam it was


lower lobe pneumonia

249
Respiratory & ENT

10. chest x ray of 3-month-old baby with bronchiolitis no improving with 3


month treatment, 2 organisms?
a. Mycoplasma
b. Adenovirus
c. RSV
d. Parainfluenza

11. 18-months-with sever bronchiolitis no improving in medication, chest x-


ray they asked what is abnormality?
a. widened mediastinum ‫حسب االشعه باالمتحان‬
b. Thymus shadow
c. Over inflation
d. Normal chest x- ray

12. a child on SIMV changed to CPAP with pressure 5 and then deteriorate
after 2 h, acidotic, pco2 =high, what to do?
a. Shift to SIMV >>> if only hypercapnic
b. Drain pneumothorax >>> if generally unwell ,poor perfusion,shock & term baby
c. Increase CPAP pressure
But exclude DOPE first, in sample paper the option was chick if CPAP
working correctly

Jan 2018
1. 11-yrs-old with cough for 3 months now yellow frothy sputum had
decreased exercise tolerance received co-amoxiclav for 2 weeks and did
not improve what to do?
a. Lung function test
b. CT chest
c. mycoplasma serology
d. bronchoscopy

250
Respiratory & ENT

2. 11 months child sudden cough, had localized wheezes over rt lung


FB

3. x-ray congenital lobar emphysema

4. picture of CT chest I think pathology over Rt lung was 8-yrs with cough and
chest pain for 7 days with fever what 3 best management to do?
a. IV antibiotics
b. oral antibiotics
c. anti TB
d. Refer for thoracotomy
e. aspiration of abscess cavity
f. diagnostic pleural aspiration
g. percutaneous chest drain
h. biopsy

251
Respiratory & ENT

5. X-ray 12 yrs with history of shortness of breath sever dyspnoea,


what are the features seen?
a. Pneumopericardium
b. rib fractures
c. Cardiomegaly
d. SC emphysema

6. 18-months boy had lack of response to loud voices delayed speech what
are indications for cochlear implantation
a. bilateral sensorineural deafness of 90
b. bilateral conductive
c. bilateral sensorineural deafness of 50

7. Child allergic can not sleep at night received cetirizine no improvement


what to give?
a. intranasal steroids
b. intranasal ant histamine
c. leukotrines inhibitors
d. nasal decongestant
e. systemic steroid

8. Lung function test ... shows Low normal FVC .. Low FEV1 ... high Residual
volume ..
A. cause of high RV ?
Air trapping
B. Diagnosis of this case?
Asthma

252
Respiratory & ENT

9. A boy with history of cough , received Antibiotics, no response, current


CXR is displayed on the screen
(shows eventration of the
diaghragm).
What is best diagnostic
tool ?
Ultrasound of the
diaghragm

Sept 2017
1. after acute severe lower respiratory viral infection, Child with clubbing
wide spread crackles frequent resp infections
bronchiolitis obliterans

2. 6-years, asthma on ffluticasone 100 bd salbutamol 4 times, not


controlled

a. add salmeterol
b. add leukotrienes

3. Spirometry values FVC low normal, FVC1 low normal. Residual volume
increased, decrease forced resp. effort???
A. why increased RV?
a. Due to decrease exp effort
b. Trapped air
c. Mucus blug
B. Cause?
a. Asthma
b. Scoliosis

253
Respiratory & ENT

4. chest x ray-
pneumatocele/staph aureus

5. Photo-ET tube
remove and reinsert

May 2017
1. CXR of infant .. inflation displacement to the right . heart sounds to the
right .

a. dextrocardia?? (apex of the heart was to the left)


b. emphysema [ most likely)
c. diaphramatic hernia (Diaphragm was intact)

254
Respiratory & ENT

2. child had acute chest infection post viral infection 4 month ago … now
has diffuse creps and wheezes and clubbing
a. chronic aspergliosis
b. bronchiolitis obelitrans
c. CF
d. foreign body

3. clear CXR OF pneumonia what is the ttt need 2 ?


a. amoxicillin
b. paracetamol

4. asthmatic child has episodes failed to controlled with salbutamol he is


hyperventilating with high sugar was 18 mmol and ABG OF metabolic
acidosis???
Salbutamol toxicity

5. epiglottitis scenario just like pastest


don’t exam don’t do x-ray prepare intubation

Jan 2017
1. x-ray of pneumomediastinum

255
Respiratory & ENT

2. question with an x-ray like this one in 5 -months old baby


round pneumonia

3. x-ray of lingula

256
Respiratory & ENT

4. x-ray

A. diagnosis?
pneumatoceles

B. responsible organism:
staph aureus

May 2016
1. Long scenario about girl with lower lobe pneumonia GP gave her
amoxicillin and cefotaxime without any improvement mother has URTI
recently resolved ask about investigations?
a. Bronchoalveolar lavage
b. sputum culture
c. mycoplasma serology
d. Mantoux test

If CT chest or US chest in options will be better option


If TB suspected or pt descent from high endemic area with features of
pulmonary TB mantoux test performed immediately.

2. Scenario of developmental deterioration admit with extensive


antibiotics, low weight Ask about investigation to help in current
management? 2 options
a. Tissue IgA ant glutaminase
b. Mantoux test It may be a case of CF
c. Glucose tolerance test complicated by DM
d. sputum culture
e. urine glucose

257
Respiratory & ENT

3. Chest x-ray patient need intubation


a. CCAM (congenital cystic adenomatoid malformation)
b. Congenital diaphragmatic hernia
c. Congenital lobar emphysema

Congenital diaphragmatic hernia

Congenital lobar emphysema

258
Respiratory & ENT

4. Neonate in mechanical ventilator developed pink frothy secretion?


Q1. Dx?
PH pulmonary hemorrhage

Q2. what is your action?


a. Re-intubation
b. chest x ray
c. trans-illumination (used for pneumothorax)
d. increase Rate

5. Sudden Respiratory deterioration after bone fracture?


a. Fat embolism
b. Pneumonia
c. hemothorax

Jan 2016
1. Pt with cough lung function, TLC normal, only high is RV
A. the explanation for RV?
a. Poor technique
b. Mucous blug
c. Malfunction machine
d. air trapping

B. What is the diagnosis:


a. Normal child
b. Cystic fibrosis
c. Asthma
d. Sever scoliosis

259
Respiratory & ENT

2. Infant X ray chest for localized blackness ask confirmatory test:


a. CT chest
b. Bronchoscopy
c. Lateral chest X-ray

3. X ray chest with mediastinal shift and....? Ask finding

4.Infant present with apnea mother give history resuscitate home twice now
resuscitated but develop a systole and died.
Has two other sibling has runny nose

May 2015
1. An asthmatic child with persistent cough for months.FBC showed
lymphocytosis? Dx
Pertussis

2. A child with pleural effusion on xray..next step of management?

a. US guided small bore drain


b. US guided large bore chest drain

3. child on CPAP of 5..deteriorated gas..what to do next..?


a. put back to SIMV
b. Inc CPAPA pressure

260
Respiratory & ENT

Sept 2014
1. chest x-ray Position of central line: Subclavian vein?

2. Chest-xray:
left lower lobe consolidation

261
Respiratory & ENT

3. chest x-ray
Cystic fibrosis

4. ETT Malpositioned deep inserted

Pull it back 2cm

262
Respiratory & ENT

5. Child with clubbing wide spread crackles frequant resp infec5ons


a. Bronchilitis oblitrans
b. Bronchactasis
c. Cystic fibrosis.

6. 6-year old with persistent asthma--‐although he is on fluticasone 100 bd


salbutamol 4 times/day.
Add leukotriene

7. Spirometry values:
FVC normal
FEV1 low normal
Residual volume increased??

a. obstructive
b. normal
c. poor expiratory

263
Infections

264
Infections

Sept 2021
1. EMQ
A.HSP
B. MEASLES
C. NAI
D.ROSEOLA INFANTUM
E. ERYTHEMA INFECTIOUSUM (5TH DISEASE)
F. CHECKEN BOX
G.MENINGIOCOCCIMIA
H.LYME DISEASE
I. ERYTHEMA MULTIFORM
A. Previously healthy baby boy aged 11 months had fever 40 C for 3 days on
examination he was unwell, admitted then developed tonic-clonic
convulsion for 3 min, on exam he was normal, 24 h later the fever
subsided and developed erythamatous rash.
ROSEOLA INFANTUM
B. 5 years old boy had fever for 1 day then developed redness on cheeks then
progressed to maculo-papular rash in buttocks, back of leg and feet
(all over the body).
ERYTHEMA INFECTIOUSUM (5TH DISEASE)
C. ?? Years old boy, looks unwell, had fever 38.5 c, swollen joints, then
developed morbiliform rash, after 12 hr the condition deteriorated, CRT 4
sec, B/P: (low), the rash become purpric, nonblanching and raised.
MENINGIOCOCCIMIA

265
Infections

2. Scenario of 4 years old from Malaysia lived with her extended family, no
pets. She had fever for 3 weeks and cough for 1 week. After 3 days pt.
presented with abnormal movement then developed decelerate posture.
What is the Dx?

a. Brain abscess
b. Toxoplasmosis
c. CMV
d. Disseminated TB
e. Tuberous sclerosis
f. Cysticercosis

3. Scenario with Hx of balanitis and pneumonia. Ask about cause?

a. Mycoplasma infection
b. Carbazepine
c. Strepococcal infection
d. Sulphonamide

4. 2 weeks old pt. Presented with microcephally, mild dysmorphic features


and can’t hear sounds. O/E: murmur best heard at upper left sternal
border and radiating to back. Diagnosis?
a. CMV
b. Congenital rubella syndrome
c. William
d. Toxoplasmosis

266
Infections

5. What is the causative organism?


a. Staph auras
b. Hemophilus influenza
c. Pseudomanas
d. Klebsiella

6. 12 years old with sore throat. What is the Dx?


a. Glandular fever
b. Scarelt fever
c. Pharyngitis
d. ALL

May 2021
1. What test leads to a definitive diagnosis?
(Age not given)
a. Monospot test
b. EBV titer

267
Infections

2. A 14-year-old female who came recently from Nigeria presents with


diarrhoea for 6 days and fever for 4 days. Two days back she passed dark
urine and developed convulsions. There was no history of medication.
What is the most appropriate investigation to reach a diagnosis?
a. Urine microscopy
b. Blood film
c. Stool microscopy and C/S
d. Urea and electrolytes
e. G6PD
f. Bone Marrow

3. EMQ about causative organisms (TAS Question)


A. Toxoplasmosis
B. E. coli
C. GBS
D. CMV
E. Parvo virus

A. A newborn with HB of 4, ascites, generalized oedema, pleural effusion


and microcephaly.
Parvo virus
B. A case with symmetrical IUGR, microcephaly and periventricular
calcification.
CMV
C. A 6-hour-old newborn with PROM presents with collapse and
tachypnoea.
GBS

268
Infections

4. A 6-week-old infant presented with poor feeding and vomiting. On


examination he looks lethargic and irritable with fever of 38.1 C’. A clean
catch urine was taken for analysis and culture.
Urine analysis showed: WBCs: 20x109/L (within normal range)
Hyaline cast +
Scanty organisms
What is management?
a. Repeat urine clean catch and start oral Trimethoprim
b. Start oral Trimethoprim
c. Start oral Co-amoxiclav
d. Do suprapubic catheter aspiration and start IV Ceftriaxone
e. LP and blood C/S and IV Ceftriaxone

5. An 8-month-old infant came with a 2-day history of fever up to 40 C’. He


was admitted to the ward for 2 days after which the fever subsided, and
a rash appeared. What is the diagnosis?

a. Measles
b. Rubella
c. Roseola infantum

6. A child with this picture came with ataxia after 7 days... What is the cause?

a. Chicken box
b. Langerhans cell histiocytosis
c. Shingles

269
Infections

7. A patient from Pakistan, came with fever and cough for 1 month. He was
given oral antibiotics without improvement then a chest Xray was done.
What is the next investigation to request?

a. Sputum C/S
b. 3 early morning gastric
wash?
c. Bronchoscopy with
bronchoalveolar lavash?
d. Mantoux test

8. A 3-year unwell child presented with fever and he developed the lesions
shown in the picture below. What is treatment?

a. Systemic antibiotic
b. Systemic antiviral
c. Topical antiviral
d. Steroids

270
Infections

Jan 2021
1. Photo of SSSS what is the treatment to be given IV ?

a. Floxacillin
b. Penicillin

2. 10 months Asian infant had 4 hours unsteady, drowsy was feeding well
with no vomiting term non-consanguineous marriage had TOF with
esophageal atresia operated when he was younger but had delayed taking
solid food and no post-operative complications.On examination he was
afebrile, mottled capillary refill 5 sec PR 160 grunting soft abdomen not
tender blood sugar 17mmol PH7.1 pCO2 3.4 bicarb 9 base def -14, ammonia
57, lactate 10, ketones 0.4, XRAY shown what is diagnosis?
No air in pelvic area

a. Salicylate poisoning
b. DKA
c. Intussusception
d. Volvulus
e. inborn errors of metabolism
f. sepsis

271
Infections

3. EMQ:
A. IV acyclovir
B. IV Ganciclovir
C. Check immunity
D. VZIG
E. give varicella zoster vaccine &VZIG
F. oral acyclovir
G. Isolate
H. observe

A. Preterm neonate with mother has chickenpox 5 days before delivery,


intervention for newborn?
VZIG
B. 4 years old on wheel chair SMA type 2 with recurrence of chest
infection, he didn’t have chicken pox before. His brother developed
chickenpox rash 2 days ago.
Observe
C. Girl with JIA on methotrexate once weekly With multiple cases of
chickenpox in her nursery.
Check immunity

4. Child with lymphadenopathy in left posterior triangle of the neck matted


with overlying skin erythema, non-mobile, non-tender, living in a farm?
a. typical mycobacterium
b. psittacosis
c. mycoplasma
d. cat scratch disease
e. Infectious mononucleosis

272
Infections

Sept 2020
1. Picture of oral lesions with difficulty in feeding, the child was well and no
other lesions in other parts of the body, What is the dx?

a. Herpes stomatitis
b. Coxackie virus
c. Mollascum Contagiosum
d. Impetigo
e. Scarlet fever

2. A newborn (D1), his mother became positive for CMV with


seroconversion during pregnancy. He is well and hearing test is normal,
asking about how to check for his CMV status?

a. CMV antigen
b. CMV serology
c. CMV PCR in urine
d. Urine early antigen CMV

3. Patient visited Scotland with parents 3 weeks ago develops fleeting


arthritis, and feverish then now the fever and arthritis resolved and
presents now with sore throat. On examination; tender calf muscle,
bilateral cervical lymphadenopathy, no liver nor spleen, all labs normal;
ESR and CRP and No rash, What is diagnosis?
a. Lyme
b. ALL
c. EBV
d. Parvovirus
e. JIA
f. Coxsackie virus

273
Infections

4. An asthmatic 5 yr. old child, on low dose corticosteroids, well controlled,


persistent bouts of cough, especially at night sometimes end with
vomiting, not improved with bronchodilator inhaler, CBC, lymphocytosis,
Wheezes and rhonchi heard bilaterally, dx?
a. TB
b. CF
c. Pertussis
d. Poor compliance
e. Mycoplasma

Jan 2020
1. 3-year child in nursery has watery offensive diarrhoea 3 weeks. .stool
analysis done. cyst discovered by special stain +ve,abd.cramps, all children
in nursery have also diarrhea. What is the causative organism?
a. E.histolytica
b. Giardia
c. cryptosporidium
d. Rota virus

2. Adolescent girl sexual relation since 2years, now has recurrent vaginal
discharge +dysuria+ abdominal pain, post coital bleeding after sexual
relation, 1st time occur.
a. PID
b. Chlamydia
c. Ectopic pregnancy
d. UTI

274
Infections

3. CT picture of child with Ewing sarcoma on chemotherapy, have


neutropenia, given extensive antibiotic with no response still febrile
Fungal abscess

4. EMQ cases given Ceftriaxone what to add?


A. Infant has Chicken box, develop erythema, fever, CRP more than 6,
hypotensive 70/54 benzyl Penicillin & floxacillin??
B. Case abdominal distension, appendectomy, on gentamycin, cefuroxime
Metronidazole
C. Knee replacement central catheter Vancomycin??

Sept 2019
1. picture like sample paper, ask
about ask about investigation at
this Stage (throat picture)
a. mono spot
b. EBV titers

275
Infections

2. patient received amoxicillin and given CSF picture Lymphocytosis,


protein 0.8
a. partially treated meningitis
b. TB meningitis
c. GBS

3. CSF data with very low glucose high protein and WBCs show
lymphocytosis ask about investigation?
a. zeil Nelsin stain

4. Patient with fever 5-day and clear description of Kawasaki


(mention all manifestation)
a. Kawasaki disease
b. Scarlet Fever
c. Measles

5. 6-yrs old female with fever for 7-days, sore throat (mild infection) and
ear pain (not sure) refuse to eat for 36-hr with BP 80/56 ,cervical and
inguinal LN, ,what to?

a. admition and observe for oral feed


b. IVF and ceftriaxone
c. immediate full blood count
d. NGT feed
e. Give paracetamol and discharge home

276
Infections

6. Picture of Steven Johnson syndrome ask about


the cause, patient was have balanitis, chest
infection (cough),,need one option
a. mycoplasma
b. Staph
c. Sulphonamide
d. Carbamazepine

7. picture of child with orbital cellulitis and pneumonia,


patient is clinically unwell, have pets at home,
vaccinated child ask about
causative organism?

a. staph aureus
b. kelebsella
c. pseudomonas
d. cat scratch
e. haemophilus influenza

May 2019
1. picture then Q About ttt
floxacillin IV

277
Infections

2. 3-weeks neonate came with poor feeding cough and shortness of breath
at 10 days old have conjunctivitis treated, diagnosis?
a. chlamydia pneumonitis?
b. mycoplasma
c. bronchiolitis

3. history of eczema not responding

A. asked about diagnosis


a. Chickenpox
b. eczema herpeticum
B. management
systemic antiviral

4. patient 5 yrs came from blangladish 1-month cough,, ask about


investigation?
a. gastric wash
b. bronchioalveolar lavage wash
c. mantoux test

5. patient with scenario of toxic shock syndrome what is the commonest


organisms (2 options)
a. Staph
b. streptococcus pyogenes
c. beta heamolytic strepto
d. Meningiococal
e. streptococcus pneumoniae

278
Infections

6. patient with ataxia, diagnosis?

d. chicken pox
e. ADEM
f. brain tumour

7. history of restricted painful eye movement

a. Orbital cellulites
b. periorbital cellulitis
c. optic glyoma
d. retinal vein thrombosis

Jan 2019
1. A picture of a swollen red periorbital region, for 24 hours, ttt?

a. IV ceftriaxone+ flucloxacillin
b. oral co-amoxiclav
c. social referral
d. chloramphenicol

279
Infections

2. picture of palatal petechiae (Sample paper Q)


glandular fever

3. A boy from Bangladesh with headache, received antibiotics and acyclovir


then deteriorating, Hb 4,5, CSF Findings: pr: 30 , Pr: 0.9 , WCC: 8 ,
glucose :slightly raised2.6?
a. viral meningitis
b. idiopathic intracranial Hypertension
c. TB

Sept 2018
1. Child picture rash appear after fever =39 subsided

a. Measles
b. Rubella
c. roseola infantum

280
Infections

2. Picture of rash appear on child neck


and shoulder?
a. herpes zoster
b. pemphigoid
c. herpes simplex
d. imt ego

3. Picture of baby irritable cry, fever , skin lesion ?


Scaled skin

4. +ve Ig M to toxoplasmosis asymptomatic, What to do??


a. Serial head circumference
b. serial ophthalmological exam

May 2018
1. patient came from the farm with chest x- ray with (strep pneumonia) but
the parent refuse to treatment and the child was become sever sick
A. asked about organism?
a. Legionella
b. Streptococcus pneumonia
c. Mycoplasma
d. Viral pneumonia

B. The parent refuse antibiotic and the patient desaturated what to do?
Court order???
If there is option of proceed without consent will be better

281
Infections

2. 15- years-old female with sudden onset of diarrhea and abdominal pain ,
fever 41 c headache ,congested conjunctiva her investigation was low hb
,low plt, high urea and creatinine developed rash
toxic shock syndrome girl, tampon

3. CMV confirmatory test?


Urine CMV PCR – if age less than 3 weeks- also can use saliva PCR

Jan 2018
1. 3 -days term baby weight 2.3 kg dysmorphic no response to sound
microcephaly and systolic murmur, what is the diagnosis?
a. Williams
b. Rubella
c. Turner
d. DiGeorge
e. Toxoplasmosis

2. Pakistani child with chest x-ray picture I think miliary TB what to do to


reach diagnosis?
a. Mantoux
b. gastric lavage
c. bronchoscopy and lavage

3. I think it was a picture about skin rash and unsteady gait


a. chicken pox
b. histiocytosis
c. measles
d. rubella

282
Infections

4. Picture of SSSS asking about ttt


Iv floxacillin

5. Pic of glandular fever-palatal petichie (repeated Q)

6. A boy with maculo-papular rash over chest and abdomen and fever since
1 week, looking bad .. red throat ... lab results were not conclusive ...
no much data (can't remeber) ... what is best next treatment ?
Aspirin + IVIG

Sept 2017
1. left eye swelling and enema photo
iv antibiotic
There were 2 IV

283
Infections

2. young girl with rash on back bullying

a. Shingles
b. Self induced

3. Post chicken pox presented with weakness and ataxia➔


varicella cerebllitis

May 2017
1. fall from tree before 1 week, presented now with fever, abdomen pain
he had difficulty in opening mouth
a. tetanus
b. rupture spleen

2. scenario clear for infectious mononucleosis


3. I think there was Lyme disease scenario
Jan 2017
1. picture of herpes zoster (spot
diagnosis)

284
Infections

2. specimen paper picture of palate. . Asks about diagnostic investigation?

Ebstein bar virus serology


(monospot test)

May 2016
1. EMQ
A. teenage with exercise intolerance +wheezing has cervical
lymphadenopathy and x ray show compressed trachea (mass) Dx?
Lymphoma
B. child on Farm developed cervical lymphadenopathy posterior group
with erythematous base??
Atypical mycobacterium lymphadenitis
C. fever for 6 days ,maculppapular rash,cervical lymphadenopathy?
Kawasaki Disease
2. Child has cervical lymphadenopathy and scratching mark, fever fatigue
night sweating, pets at home what Dx?
a. TB
b. cat scratch disease ???
c. non-Hodgkin lymphoma

Jan 2016
1. Same pic in sample paper of palatal
hemorrhage tell he has fever and sore
throat for two weeks ask what is the
diagnosis:
a. Glandular fever
b. Leukemia

285
Infections

2. 13 y girl with vomiting and fever start at night, morning she has profuse
Diarrhea and headache on exam temp 41 negative Kernig sign:
a. Meningitis
b. Meningococcal septicemia C
c. toxic shock syndrome

3. 8 weeks with fever and vomiting routine urine bag test normal what is
action:
a. Obtain clear catch test
b. Start antibiotic
c. Abd. US
d. repeat urinalysis

May 201
1. a neonate with eye infection and later on developed sepsis..no risk factors
for sepsis..
a. GBS
b. Herpes

286
Infections

2. a child with possible Lyme

3. child with SJS rash..had hx of cough recently..possible cause:


a. Sulphonamide
b. Mycoplasma
c. streptococcal infection
d. carbamazepine

4. child from Pakistan with long standing cough. Would u do..?


a. Mantoux
b. 3 consecutive gastric washing

5. a child with hx suggestive of Kawasaki. Asked about management?

287
Infections

Sept 2014
1. picture of Periorbital cellulitis --treatment?
Oral antibiotic co-amoxiclav

2. Picture of Shingles

3. Post chicken pox Presented with weakness Varicella cerebillitis

4. child with possible TB.to confirm? AFB in gastric aspirate


Jan 2013
1. a child about 12 years old, previously well, live in a farm with his
vegetarian alternative family, he complains of cough ,, fever ,, that get
worse over one weak ,,on exam : RD,, dull left lung base ,, x-ray :-
massive left pleural effusion , what is the causative organism ?
a. staphelococcus areus
b. mycoplasma
c. atypical mycobacterium
d. TB
e. Legionella

288
Immunization

289
Immunization

Sept 2021
1. Indication of palivizumab?
a. Baby 31 weeks who ventilated for 3 days and discharged at 61 days on
0.2 l/min O2
b. 29 weeks ventilated for 2 wks then discharged on 58 days off o2 for
3 weeks
c. Non-significant PDA
d. Hemodynamically stable CHD

Jan 2021
1. parents with child worry about MMR vaccine which of following
contraindicated?

a. child has finished course of steroid (oral prednisolone) for nephrotic


syndrome 1month ago
b. use intraarticular steroid for 2 days one month ago.
c. pt with asthma 100 micg inhaler bid
d. pt with severe eczema on topical steroids

Sept 2020
1. 14-years-old boy is a known case of egg allergy came with mother. He
was not given MMR vaccine before. Mom wants to give him the vaccine
now, what to do?

a. Take in the hospital


b. Take MMR vaccine at the GP surgery
c. Do not give MMR
d. Pediatrician should do allergic test before vaccine as he has egg allergy

290
Immunization

2. A preterm was born 26 wks. Gestational age and admitted to NICU. Now
he is 3 weeks. His 5 years old brother developed chicken pox at home 48
hours ago, their mother is visiting the neonate in the NICU daily, and his
sick brother isn’t visiting him, mother doesn't have any symptoms, what’s
the best management?

a. Check maternal antibody


b. Give VZIG to the neonate
c. Give VZIG plus acyclovir
d. Stop Mother visit for 21 days
e. Isolate baby

3. Baby born to a mother HBsAg positive when to give hepatitis B


immunoglobulin to the child?

a. Give regardless mother results


b. Mother had PCR DNA copies more than 1 million antenatally
c. HBsAb and HBcAb negative
d. HBeAg negative and HBeAb positive
e. HBsAg >100

Jan 2020
1. Infant ex preterm given steroids discharge on home oxygen before
discharge, which you will do about vaccine?
a. No vaccine till term
b. Postpone vaccine
c. RSV vaccine Palivizumab
2. 16-yrs old came with her boyfriend for HPV vaccination what to do?
a. check pregnancy test (excluded)
b. give the vaccine
c. she will take 3 doses
d. she can’t take if sexual active
e. check antibody

291
Immunization

May 2019
1. EMQ
a. iv immunoglobulin
b. iv acyclovir
c. admission for observation
d.reassure no thing to do
e. discharge home and advise to bring the baby if symptomatic
f. urine pcr
h. blood for viral PCR
i. LP
J. ORAL ACYCLOVIR
h. viral serology

A. baby born at 27 wks.950 gm. with chronic lung disease.39 temp.


his brother developed checkin pox. baby presented with fever 38 and
poor feeding
viral serology or IVIG??
B. baby 35 days his brother developed checkin pox. his mom has chicken pox
before
C. neonate mouth ulcer and rash IV acyclovir

Jan 2019
1. contraindications of rota vaccine: (3 Answers)
a. Previous intussusception
b. age > 24 months
c. SCID
d. previous mild reaction to rota virus
e. previous admission with rota
f. GERD
g. Neutropenia
h. lactose intolerance

292
Immunization

Sept 2018
1. Full term healthy 3 weeks old baby, sister has chickenpox, mother say she
had chickenpox as child , what to do ?
a. No-thing
b. give VZ Ig
c. give acyclovir

2. Influenza vaccine 2 indication?


a. 3-month with screening CF +ve
b. 4 year with Eczema has fever
c. 8 year on inhaled corticosteroids for asthma
d. 11 year with non-cystic fibrosis bronchiectasis
e. 7 year ASD closure 2 years ago
f. 3 years with recurrent hospital admissions with croup

May 2018
1. PZV prophylaxis to whom you will give?

a. 29-weeks on ventilator for 2-weeks


b. 32weeks on ventilator for 2-days and complete on oxygen at home for
60-days (BPD

2. 13-years-old with severe egg allergy came for MMR has not taken any
vaccine before what to do?
give in GP surgery

Jan 2018
1. A 5 yr child infected with chicken pox ... his baby brother is 21 days old
currently @ a NICU ... the mother visits the baby daily ... best action?
a. Prevent the mother from visiting the baby
b. Administer IGs to the baby
c. the baby can't get infection due to transplacental antibodies
d. checking the maternal antibodies against chicken pox first

293
Immunization

Sept 2017
1. HPV vaccine?
a. protect against the two HPV types (HPV-16 and HPV-18) that cause
70% of cervical cancers

2. Father positive for HBVe and s Ag ,, mother HBVs AB plan for baby
Vaccine only

Jan 2017
1. EMQ about chicken pox:
A. pregnant woman exposed ti ochicken pox just before delivery
give VZIG for baby
B. child with JIA on methotrexate exposed to chicken pox
chose to check immunity
C. don’t remember

May 2015
1. A child with eczema herpeticum..asked about management
[Acyclovir iv or oral according to Pt situation]
if generally unwell we give IV

2. child with 3 hrs of persistent crying post immunizations. What to do for


next vaccines...?

a. give at Gp surgey
b. give in hospital --- if more than 3 hrs
c. do not givep

3. a question about immunization of a per term with home oxygen..2 options


a. fitted with immunizations he should have
b. palivizumab

will take all his immunization with adding plavizumap

294
Immunization

Sept 2014
1.HPV vaccine
Cover 70% of cancer.

2. father Hbsag +ve, HbeAB +ve.mother nurse and anti Hbs titre in serum
more than 10.baby should receive

a. HBV vaccine only.


b. HBV + immunoglbulins

Jan 2013
1. 13 years old girl, have a boyfriend come for vaccine, what you till about
HPV?
a. HPV can prevent cervical carcinoma by about 90%
b. ask her if sexually active
c. don’t give it before pregnancy test

295
Neonatology

296
Neonatology

Sept 2021
1. 3 days old neonate who was discharged at 48 HR, presented with history
of intermittent non bilious vomiting which became bilious later. On
examination baby was unwell and pale, CRT 3 seconds, abdomen was
distended with visible dilated bowel loops. Intestinal sounds were not
audible. X-ray without contrast (x ray with dilated bowel loops, no air fluid
level):
(x-ray of Hirschsprung, but scenario suggestive of Malrotation)

a. Meconium ileus
b. Malrotation
c. Hirschsprung disease
d. Small bowel
obstruction
e. Intussusception

2. 29 weeks gestation, 3 hours old, ventilated PIP /PEEP 20/5, X-ray (chest
and abdomen with RDS, ETT, UVC and UAC; bad quality), what is
abnormality in this X-ray? )‫الصورة غي‬
‫( واضحة نهائيا‬100% neonate)

a. Pulmonary interstitial emphysema


b. Malpositioned UAC
c. Malpositioned UVC
d. Small intestinal obstruction
e. Malpositioned ETT

297
Neonatology

3. 5 days old baby was in passive cooling. How to diagnose Brain death?
a. Increase CO2 more than15
b. Rewarm for 24 hr.
c. Brain stem death cannot be assessed in this age
d. EEG
e. Decrease oxygen

4. Newborn after vacuum delivery. What is the diagnosis?


a. Cephalohematoma
b. Caput succedaneum
c. Subgaleal hematoma
d. NAI
e. Chignon

5. EMQ
A. ABO INCOMPATIBILITY
B. RH ISOIMMUNIZATION
C. SEPSIS
D. NAI
E. ALLAGILE SYNDROME
F. CRIGLER NAJJAR SYNDROME
G. PHYSIOLOGICAL JAUNDICE
H. CYSTIC FIBROSIS
I. G6PD
J. GELBIRET SYNDROME
A. Caucasian boy 18 hours age, developed jaundice, direct coomb's test is
-ve, he uninterested in feeding and passed meconium.SEPSIS

B. Caucasian boy 18 hours age, developed jaundice, mother's blood group


O +ve and direct coomb's test is +ve, good feeding and passed meconium.
ABO

C. Caucasian boy 30 hours old developed jaundice direct coomb's test is –ve
he is feeding well and passed meconium.PHYSIOLOGICAL JAUNDICE

298
Neonatology

May 2021
1. Below is an abdominal x-ray of a preterm of 26 weeks on TPN and NGT
milk 2 ml / 2 hourly. He was pale, with metabolic acidosis and respiratory
distress. Select one answer.

a. Intramural gas
b. ETT in right main bronchus
c. Sub diaphragmatic air
d. Portal gas
e. Distended bowel

2. A 5-day old newborn collapsed. He was mottled, tachypnoeic,


tachycardiac and has hepatomegaly. There was metabolic acidosis with
RBS of 6. All peripheral pulses were weak. What is the diagnosis?

a. Inborn error of metabolism


b. Sepsis
c. Coarctation of aorta (not cardio)
d. NAI

3. A term patient (42 weeks) now came after 5 weeks with failure to gain
weight, 3-times loose motion and he did not open his bowel for 2 days.
There was a history passing meconium at 30 hours. On examination: The
abdomen was distended. PR showed expulsive yellow stool and gas. X-ray
showed no air in the pelvis. What investigation should be done?

a. Sweat test
b. Rectal biopsy
c. Rectal enema
d. Upper GI contrast study

299
Neonatology

4. A 1-week infant with persistent chest symptoms since birth. What does
this X-ray show?

a. Congenital lobar
emphysema of the left
upper lobe
b. Pneumothorax
c. Pulmonary interstitial
emphysema

Jan 2021
1. Patient with hypocalcemia seizure on infusion, cannula removed and after
6 hours a lesion was noted as shown in photo (extravasation),
what is the tx?
a. Topical emollient
b. Topical GTN
c. Saline infiltration
d. limb elevation

2. Scenario about Newborn and heart sounds heard on the right with Xray

a. congenital lobar emphysema


b. CCAM
c. Dextrocardia

300
Neonatology

3. Photo of term newborn boy with skin rash (ecchymotic) and (swelling
lesion on right eyelid (hematoma)), the baby was well and photo showed
connected UVC.Mother is 36 years old and has 2 previous well babies.The
baby has thrombocytopenia and leukocytosis.Asking about investigation?

a. blood film
b. TORCH screen
c. Direct coombs test
d. Blood c/s
e. Coagulation screen

4. 17days intermittent vomiting since birth become bilious in the last 24


hours had poor wt gain and was floppy with depressed fontanelle and
soft lax abdomen. labs showed hb 150, platelets 238, Na 132, K 3.2,
Cl 96, creatinine 60, glucose 4, urine mix growth of E coli and staph,
What is the investigation to confirm the diagnosis?
a. PH monitoring
b. suprapubic urine for culture
c. Galactose 1 phosphatase
d. cranial u/s
e. upper GI contrast

5. EMQ:
A. congenital heart disease
B. neonatal sepsis
C. NAI

A. 3 days with O2 sat 79% given O2 saturation increased to 84%


Congenital heart disease
B. Mottling saturation increased with O2, prolonged CRT, Temp 39
Neonatal sepsis
C. Nose bleeding, full fontanel, poor feeding.
NAI

301
Neonatology

6. month-old with history of IVH (RESOLVE SPONTANOUSLY), normal


motor development have minimal social problem but more language
and speech problem asking about Cause of this.

a. Haemorrhagic hydrocephalus.
b. PVL (frontal)
c. hearing loss

7. Neonate 5 days has thrombocytopenia, neutropenia with rash in face and


limbs increased with exposure to sun, the photo included limbs also What
was the investigation needed?
The photo showed limbs also
a. Anti Ro and anti La antibodies
b. Bone Marrow
c. Urinary A.A level

Sept 2020
1. A newborn (D1), his mother became positive for CMV with
seroconversion during pregnancy. He is well and hearing test is normal,
asking about how to check for his CMV status?
a. CMV antigen
b. CMV serology
c. CMV PCR in urine
d. Urine early antigen CMV

302
Neonatology

2. X-ray Abd of a neonate after 2 days came with bilious vomiting; he is


unwell with tender abdomen, what is the diagnosis?
(A big bubble and a small bubble below it)

a. Duodenal atresia
b. Intestinal obstruction
c. Midgut volvulus
d. Intussusception

3. 6-days neonate FT was born at home given oral vitamin k, passed


meconium on day 3 and was constipated, presented with 6 hours
bleeding per rectum, shocked CRT 3 seconds, HR 180, slightly distended
abdomen tender on palpation, What is the dx?
a. Hirschsprung with enterocolitis 100 neonatology and infection
b. NEC
c. Cow milk allergy
d. Coagulopathy
e. Malrotation

4. 16-days old baby boy with hypo plastic heart Post cardiac surgery
develop bloody diarrhea and distended abdomen What is diagnosis?
a. Ischemic bowel
b. Intussusception

303
Neonatology

5. Baby 26 weeks 12 days old with


Abd. Distension and greenish
Aspirations what are the 2 finding
in X ray?

a. Intestinal obstruction
b. Dilated Small Intestinal loops
c. Dilated Large bowl
d. Free air in the peritoneum
e. Intramural air
f. Thickened Intestinal wall
g. Situs inversus

6. Picture of newborn genitalia (there was meconium at the base of


scrotum and mostly no opening for the anus)
What is the immediate action?

a. Urgent surgical referral


b. 17 oh progesterone
c. Refer to orthopedic
d. Abdominal X ray
e. Hip U/S

304
Neonatology

Jan 2020

1. Picture of intestinal malrotation 4weeks old

a. gut malrotation
b. Atresia
c. pyloric stenosis
d. duodenal stenosis

2. Mother drug abuse known to social worker her baby has recurrent
vomiting occasionally bilious, doctors decreased amount of feeds but still
vomiting
a. barium enema
b. abdominal u/s
c. upper GIT study

3. Picture of abdominal x-ray double bubble appearance


Duodenal atresia

305
Neonatology

4. Baby 23 weeks + 6 days, providing ventilatory support who’s opinions


influence last decision to resuscitate?
a. parent ??
b. Consultant NICU ??
c. Parent religious head

5. picture of extravasation

6. X ray of abdomen of preterm baby what is important finding?


a. air in portal vein?
b. increase thickness of intestinal wall
c. pneumoperitoneal
d. air in the intestinal wall

306
Neonatology

7. 10 days, loss weight in spite of increase feeding, blood sugar 23.5, Na 156,
alert not dehydrated (similar to newformate case1.3) diagnosis?
a. DI
b. Dehydration
c. Transient DM
d. RTA
e. CAH
f. neonatal DM

8. mother febrile, GBS positive, have history of child died from sepsis, PROM
36hr, baby now well, mother refused antibiotic for previous child and this
child what to do?
a. Discharge and ask her to come back if there is symptoms
b. Discharge on oral antibiotics
c. Give antibiotic despite her refusal as this for child best interest
d. Call social services to get court order

9. Pulse oximetry left foot 92% what to do?


a. Echo before discharge ??
b. Follow up after 2 weeks
c. Oxygen ??

10. Infant ex preterm given steroids discharge on home oxygen before


discharge, which you will do about vaccine?
a. No vaccine till term
b. Postpone vaccine
c. RSV vaccine Palivizumab

307
Neonatology

Sept 2019
1. x-ray neonate with shortness of breath
a. left congenital emphysema
b. Pneumothorax
c. hypo-plastic left lung
d. upper lobe collapse

2. x-ray neonate on day 3 with bilious vomiting,


(x-ray not clear but there
was dextro-cardia),
ask what is diagnosis?
a. Malrotation
b. Intussusption
c. NEC
d. small bowel obstruction

3. picture of bilateral cephalohematoma

308
Neonatology

4. picture of u/s brain ,, neonate expt 28wk on day 15 of age ask finding
3 option finding

a. Dilated 3rd ventricle


b. dilated 4th ventricle??
c. ivh
d. pvl
e. dilated lateral ventricle
f. subdural haemorrhage
g. partial absent corpus collosum

5. picture of neonate face presentation with swelling and redness around


the eye ask
A. what to do?
a. reassurance and discharge
b. admitted for observation
c. midwife follow up at home after 4day
d. follow up in postnatal word
B. what investigation will do?
no investigation

309
Neonatology

6. patient with HIE ask about what indication for cooling?


a. change in muscular tone
b. poor feeding
c. excessive cry & irritability

7. neonate just deliver antenatal u/s show bilateral dysplastic kidney


oligohydramnios, what to do need 2 option?
a. urgent u/s abdomen
b. follow urine stream
c. U&E
d. prophylaxis antibiotic
e. refer to nephrology
f. insert catheter

8. mother came at 28wk gestation with PROM, to hospital with out 24h
neonatal care with facility for short term ventilation, Expected to go in
labour, mother admitted to word, positive gbs swab what will be most
benfit for baby out come

a. arrange for transfer to tertiary hospital


b. give Betamethazone IM ??
c. surfactant
d. admitted for short term ventilation
e. tocolytic
f. antibiotic ??

310
Neonatology

May 2019
1. Xray neonate heart heard in rt side picture ,clinically stable
a. congenital lobar
emphysema
b. dextro cardia
c. CCAM
d. CDH
e. PIE

2. 24 weeks GA XRAY ABdomen 7days old feeding 2ml/2h expressed milk


detariorated became pale and shocked pt... CHEST

a. portal vein gas


b. intramural gas
c. pulmonery hage

3. 5 days old , baby was in passive cooling HOW TO DIAGNOSE Brain death
a. Increase CO2 more than15
b. Rewarm for 24 hr
c. brain stem death cann’t assessed in this age
d. EEG
e. decrease oxygen
d. EET in rt bronchus

4. scenario of baby with bilious vomiting (malrotation )


a. Upper GI study
b. abd u/s
c. xray abdominan

311
Neonatology

5. 6-day collapse newborn having severe metabolic acidosis all peripheral


pulses weak, hepatomegaly 6cm ...RBS 6
a. sepsis
b. COA
c. organic aciduria

7. neonate preterm with mottling and low bp, bradycardic vent map 38 ...
co2 normal ....po2 low fio2 90%... with good air entry (pphn)
deteriorated after 4 hrs, how to treat?
a. nitric oxide
b. prostaglandin

8. 3-weeks neonate came with poor feeding cough and shortness of breath
at 10 days old have conjunctivitis treated, diagnosis?
a. chlamydia pneumonitis?
b. Mycoplasma
c. bronchiolitis

Jan 2019
1. EMQ:
a. congenital heart
b. Neonatal sepsis
c. Congenital pneumonia
d. NAI
e. Haemorage
f. Hydrocephalus
g. Meningitiss
h. meningococcal septicaemia
i. tracheoesophageal fistula

312
Neonatology

*Apnoea in a 5 day old, dusky in colour, after feeding :


A. Cyanosis, not much improved by O2 --- congenital heart
B. 39 °c , Na : 130, capillary refill time 3 sec,hepatomegaly-- Neonatal sepsis
C. WCC: 3.5 ,nasal bleeding, full fontanelle – NAI

2. A neonate 5-days with signs of sepsis, unilateral conjunctivitis, Inc PT ,


PTT hepatic necrosis no risk for GBS :
a. Neonatal herpes
b. Chlamydia
c. Glactoceamia

3. A neonate had his last feed at 10 pm ,he had seizures for 5 min just
before his 6:30 am feed, his blood glucose was measured at that time and
found to be very low and started on dextrose after which it retained to
normal, he had a similar episode after a couple of hours and he continued
to have them despite the high GIR he was on(10
A. what is the best action?
Insulin assay at time of hypoglycaemia
B. Management?
a. Hydrocortisone
b. Glucagon
c. increase rate of glucose or concentration

4. mother ↓ platelets, baby pale has soft swelling over his vertex , cord
blood platelets: 35, Next stept?
a. Repeat FBC of venous blood of baby
b. father and mother platelet antigen
c. cross matching and direct antiglobulin test
d. cranial u/s

313
Neonatology

5. x ray:
a. small bowel obstruction
b. misplaced UAC
c. misplaced UVC
d. right ventricular hypertrophy

UAC in the high position


a. The UAC is at T6, which is
satisfactory. The UVC is at
T7.
b. Ideally, both should be
above the diaphragm.
c. The UAC should be
between T6 and T9 1
d. The UVC should be in the
IVC as it enters the right
atrium.

6. 5 days old Hepatomegaly, deranged liver functions


jaundice (UNCONJIOCATED), high PT & APTT, low fibrinogen bleeding
from umbilical stump , negative blood and urine culture ,platelet
normal?
a. RBCS GAL1PUT
b. urine CMV
c. liver biopsy
d. alpha 1 antitrypsin

314
Neonatology

7. AT sight of canula (Picture ) of extravasation , ttt?

a. lift arm
b. trinitrite
c. saline
d. deroofing
e. emollient
f. intralesional triamcilinone

8. A mother Keen to breast feeding, but has trouble initiating breast


feeding and blood glucose is low 2.4 , ttt?
a. Complementary bottle feeding until breast feeding
b. iv dextrose
c. mother express breast milk in bottle
d. trial of breast feeding now

Sept 2018
1. 4-days-old Neonate, sepsis, difficult to extubate , fluids 120 ml /kg
high urea & creat, on 90 ml/kg TPN (part from fluids ), on antibiotics for
48 hrs , culture negative so stopped antibiotics 3 days ago ,
UOP = 2.9 ml/kg/hr what to do ?
a. check gentamycin level
b. decrease protein in TPN
c. increase fluid bolus
d. iv furosemide
e. increase fluid 150 ml/kg

315
Neonatology

2. Neonate didn’t pass stool for 36 hrs with abdominal distention, picture x
ray at 40 hrs with absent gas beyond duodenum
a. Hirschsprung
b. CF
c. duodenal atresia
d. NEC

3. EMQ
A. Neonate, normal, sometimes apnoea then normal breath
periodic breath
B. 3 weeks infant scenario viral illness, apnea
bronchiolitis
C. Neonate irritable and apnoea on change dippers, mother 19-yrs, father 20
unemployed, full fontanel at rest, increase head size
abusive head trauma

4. Two drugs not given in AUC (arterial umblical catheter)?


a. Morphine
b. Adrenaline
c. Dopamine
d. cefotax
e. dextrose 10%,
f. vitamins

316
Neonatology

5. Picture of neonate 7 days old with eye infection purulent discharge

a. Chlamydia
b. gonohrrea

6. 3-weeks Neonate polyuria, polydipsia, high RBS 23


transient neonatal DM

7. Neonate after feed and lying down become pale chocking, overweight,
doctor reduce feeds ,attacks less but still present , investigate ?

oesophageal Ph study

May 2018
1. what the benefit of therapeutic hypothermia
decrease mortality and improved outcome

2. 2-days-old with I think pain in the abdomen with air level at duodenum,
NG aspiration it was greenish color

a. duodenal atresia
b. Malrotaion
c. Hirschsprnug
d. Small intestinal obstruction
e. Pyloric stenosis

317
Neonatology

3. x-ray abdomen with double bubble sing asked what the diagnosis ?
Duodenal atresia

4. x- ray abdomen malrotation

Jan 2018
1. newborn to mother with BMI 37 before pregnancy birth weight 1.9 kg
glucose 1.9 developed apnoea, what is diagnosis?
a. infant of diabetic mother??
b. IUGR ??
c. Sepsis

318
Neonatology

2. 23-weeks of gestation mother came with rupture of membranes who


should decide for resuscitation and stabilization of baby
a. Parents
b. NICU consultant
c. mid wife
d. ob doctor

3. 28-weeks of gestation baby what would be of clinical benefit for him?


a. CPAP
b. NPO for 24 hours
c. Azithromycin
d. TPN

4. 17-days intermittent vomiting since birth become bilious in the past 24


hours had poor weight gain and was floppy with depressed fontanel had
soft lax abdomen labs showed, hb 150, platelets 238, Na 132, K 3.2 , Cl 96,
creatnine 60, glucose 4, urine mix growth of E coli and staph what
investigation to do?
a. PH monitoring
b. suprapubic urine for culture
c. galactose 1 phosphatase
d. cranial u/s
e. upper GI contrast

5. baby boy NSVD to primigravida 37 weeks of gestation weight 3.25 on


examination at 24 hours before discharge spo2 92 hours left foot but
clinically normal what management?
a. Echo
b. O2
c. opd follow up at 6 weeks
d. CPAP
e. Antibiotic

319
Neonatology

6. Picture of neonate with RDS what is diagnosis?


a. Pneumothorax
b. congenital diaphragmatic hernia

7. picture of x-ray chest and abdomen with ETT 4 hours old baby has
vomiting and imperforated, anus what is your immediate 2 actions?
a. withdraw ETT
b. urgent surgery referral
c. TPN
d. CT
e. Karyotyping

Sept 2017
1. Thrombocytopenia mother on carbimazole
drug induce thrombocytopenia

2. Nurse notes spo2 in left foot 92%?


Arrange echo prior to discharge

3. newborn baby good condition HR 50-60


Observation in NICU

4. Pregnant 28-weeks women came to you in local primary NICU with


PROM, what is the best action to improve the neonatal survival before
transfer to tertiary NICU???
a. Tocolytics
b. Antibiotics
c. Dexamethasone
d. Surfactant

320
Neonatology

May 2017
1. picture cephalohematoma

2. picture of newborn presented by face has bilateral orbital bruises


A. what to do?
Nothing
B. Second part of question?
reassure parent

3. mother keen to Breast feed newborn but it is difficult and has


hypoglycemia 2.2,Baby is ok
a. give expressed milk
b. bottle feed
c. iv dextrose

4. collodion baby picture

321
Neonatology

5. A premature infant developing abdominal distension and developing


shock like condition, investigation?
abdominal X-RAY (maybe NEC)

6. indications of cooling for neonate


a. Poor consciousness and decreased reflexes
b. PH<7
c. MAP <30
d. Poor Apgar score

7. picture for neonate has hematoma in eye lid and chest petechial rash
asking how to investigate need 2?
c. Platelet antibodies
d. sepsis screen

8. upper GI STUDY I think it was double bubble?


duodenal atresia

322
Neonatology

Jan 2017
1. baby7 -weeks, constipation, open mouth in pic with relatively protruded
tongue

Hypothyroidism

2. intestinal obstruction case in a newborn with x-ray of double bubble


It wasn't that clear (more than two bubbles )
Duodenal atresia

3. sepsis like picture, 5 days old, 5-cm hepatomegaly, Conjunctivitis


a. E-Coli sepsis
b. GBS
c. Chlamydia

4. a case of neonate with very high blood sugar, Dehydration, Similar case
in new format book. . Diagnosis?
Neonatal DM

5. long case of a neonate. . I don't remember it was vague . . I remember he


asks for 2 options management . .
I chose dobutamine & mechanical ventilation

323
Neonatology

6. a case of PPHN. .
A. diagnosis-- PPHN
B. ttt ---nitric Oxide

May 2016
1. X-ray of neonate developed cyanosis soon after birth when he feeding ask
about immediate management? 2 options

a. refer to paediatrics surgery


b. feeding and NG tube
c. NPO+ iv fluids

(coiling OGT tube Diagnostic of CDH or hiatus hernia)

2. Neonate developed cyanosis and apnoea soon after birth mother 25 year
has placenta previa investigation revealed glucose 14 mmol/l
what is cause of neonatal hyperglycaemia?

a. stress and pain (can cause transient neonatal hyperglycaemia)


b. overfeeding
c. DM
d. external glucose (IV dextrose)

324
Neonatology

3. Large scenario of neonatal umbilical bleeding Ask about Dx?


a. alpha_1_antitrypsin deficiency
b. vit K deficiency

4. Picture of abd .X_ray lateral position NEC with features of


pneumoperitoneum and perforation ask about managment??
3 options
a. NPO+NG for suction +iv fluid
b. emperical antibiotics (prevent sepsis)
c. increased enteral feeding
d. refer for pediatric surgery
e. laparotomy

Pneumoperitoneum appear just behind anterior abd wall suggest of


perforation in a case with NEC

5. Very large scenario about Asian neonate to non-consagious parents the


baby has persistent hypoglycaemia need 15% dextrose to maintain normal
glucose mother has SIDS asked about diagnostic investigations.
c-peptide:
high —— hyperinsulinism
low — exogenous insulin

325
Neonatology

6. Neonate at birth, cyanosis deteriorated with grunting investigation


requested antibiotics was given there was heart murmur and tachypnoea
also tachycardia what Dx?
a. Sepsis
b. PPH (persistent pulmonary HTN

7. EMQ:
A. Mother stopped breastfeeding & baby developed convulsions
Hypoglycemia
B. Mother has back pain, osteoporosis, Baby develop convulsions
Hypocalcemia

Jan 2016
1. X ray double bubble
Duodenal atresia

2. Picture of recently delivered neonate with history of forceps delivery has


bilateral lid (light ecchymosis /redness) ask what management:
a. Discharge and reassure
b. Cranial USS
c. No action

3. Scenario of hypernatremic dehydration as in sample paper

326
Neonatology

4. X ray pt 6 hr old with history of progressive distress and established


feeding well Dx?
congenital diaphragmatic hernia

May 2015
1. there was 2 weeks old baby with distended bowels
a. NEC
b. meconium ileus

2 . A baby born with hb 4.5..what inv to next?


Kleihauer test in fetomaternal transfusion ( appear fetal cells in mothers blood)

3. newborn baby good condition HR 50-60


a. observation in NICU
b. adrenaline
c. ventilation

Sept 2014
1. Nurse notes spo2 in left foot 92%?
a. Echo before discharge
b. Give O2
c. Discharge and review later.
d. Urgent ECHO

327
Psychiatry

328
Psychiatry

Sept 2021
1. Scenario about 4 years old, mild ADHD. Asking about the initial
management:
a. Refer parents to parental training and education program
b. Refer child to behavioural therapy sessions
c. Prescribe methylphenidate
d. Refer to educational health service

2. 9-year-old boy at school making trouble and fighting, he has set fire in a
local park and also ran away from school, Dx?
a. ADHD
b. Conduct
c. Oppositional defiant disorder
d. Autism

3. 13 yr. old boy, who is studying science and he is good school achiever,
presented with recurrent episodes of tiredness of 6 months duration,
which affect his school attendance for the last 2 weeks.Past Medical Hx:
recent viral infection with cervical lymphadenitis which improved. His
systemic examination was unremarkable.
What is the cause of his condition?
a. Chronic fatigue syndrome
b. Depression

May 2021
1. A13-year-old girl with anorexia nervosa and BMI of less than 14.5. Serum K
is 3.4. Which of the following is a high-risk sign?

a. Tympanic temp < 35.5C


b. BMI <15 (0.4-2nd centile)
c. QT < 400
d. BP 80/60
e. Hypokalemia 3.4 (normal 3.5-5.5)

329
Psychiatry

2. A patient was diagnosed with ADHD and you are going to prescribe
Methylphenidate for him. What is the expected side effect to discuss with
the parents?
a. anxiety disorder
b. Bradycardia
c. Seizure
d. Weight gain
e. Hypotension
3. A girl with juvenile chronic arthritis on methotrexate who lost her follow
up presents with joint contracture. The patient and her parents refused to
have the drug as they believe in homeopathic medicine. She is withdrawn
and unwilling to discuss the treatment.
What is the next appropriate action?

a. Arrange outpatient appointment


b. Refer to CAMS
c. Refer to social service
d. Child protection plan

4. A teenager took Paracetamol and was admitted to hospital.Paracetamol


level was below treatment line. What is your next action?

a. Refer to psychiatrist
b. Give N acetyl cysteine
c. Discharge

5. A teenager with a history of viral illness for 1 moth before the


presentation. For the last 3 months she developed fatigue and muscle
aches with frequent school absence. What are the next steps in
management? (Choose 3 options)
a. CBT
b. Graded exercise
c. liaise with school welfare

330
Psychiatry

Jan 2021
1. 12 yr old Girl has hallucinations, which of the following will rise most
clinical concern?
a. hearing people she knows in last attacks of migraine
b. hear friends’ voices when they are not here which bother my mother
c. voices telling bad things about her
d. heard newsreader on TV telling her not to do homework ??
e. voice calling my name while want to sleep

2. Adolescent girl Took paracetamol over dose, but below toxic level
what to do next?
a. discharge with parents
b. psychiatric assessment
c. do pregnancy test

3. A 14 years child who is expelled from school, difficult to control by


teacher, blame others for his mistakes, mother says child behave the same
at home, the child is friendly (affectionate) with his dog. Asking about
behavioral problem
a. oppositional defiant disorder
b. conduct disorder
c. pathological demand avoidance
d. ADHD
4. pt with different foster parents has no friends, no problem if carer is there
or no, Child does not care who is caring for him and has aggressive
behavior, but is friendly with strangers.

a. ADHD
b. attachment disorder

5. 8 years old child with chronic central abdominal pain, he didn’t go to


the school for many days, all investigations are normal.
What to do next..?
a. psychology referral

331
Psychiatry

6. 12 years old boy with gynecomastia teased at school, he misses the sports
activities class in the school, on examination his testes 8ml and has pubic
hair, management?
a. psychology referral
b. start testosterone small dose
c. review after 6 months
d. refer to plastic surgery
e. discharge & reassurance

7. Teenager girl diabetic having multiple DkA not injecting insulin, recent
history of drug abuse, when you talk to her and revise her that she was on
good control of DM previously and knows about the disease and its
management but when you tried to re impose this knowledge, she laughed
out at you, asking which of the following explain her condition.

a. adolescent risk-taking behaviour


b. puberty
c. poor adherence to treatment
d. she wanted to be free like her peers

Sept 2020
1. Question about Anorexia nervosa, BMI 15 what make you think of another
organic dx other than anorexia?
a. Abdominal pain
b. BMI
c. Baggy clothes
d. Amenorrhea
e. High cortisol
f. Hypokalemia

332
Psychiatry

2. Picture of a boy he is 14 years old with progressive weight loss & poor
appetite over the last 6 months, height on 9th centile, very active in the
school activities and sports. He has good school performance, and
otherwise normal. FBC, Electrolytes are normal. Normal bowl habits
‫ولد شكلو حلو‬
What is the dx?
(Eyes were covered and the child was
not dysmorphic)

a. Anorexia nervosa
b. Hypothyroidism
c. Celiac disease
d. Diabetes mellitus

3. 9-year-old boy at school making trouble and fighting, he has set fire in
a local park and also ran away from school, dx?
a. ADHD
b. Conduct
c. Oppositional defiant disorder
d. Autism

4. 3 YEARS old child fighting with mom and brother, throws things when
angry, sees father only in the weekend and mother is treated for
depression, he spends nice time in nursery. What is the management?
a. Parent training program
b. Refer to psychology
c. Multi-disciplinary team for autism
d. Referral for ADHD assessment

333
Psychiatry

6. Teenage GIRL with paracetamol overdose, after 4 hours, the paracetamol


level did not reach level of treatment. She is treated for depression, she
started to be concerned by her weight and tries to reduce it, she is talking
about death, there was history of previous paracetamol overdose, and
there is family history of depression. What is the most important risk
factor for suicide?
a. Previous suicidal attempt
b. Talking about death
c. Father taking antidepressants
d. Psychological therapy
e. Fear of weight gain
f. Family history of depression

7. A teenage girl 14 years, she wants to socialize but cannot deal with her
friends at school, her peers try to avoid her and not asking her to
participate in group activities because she is tampered and opinionated,
she has difficulty with sleeping, tired during the day, she was good at
classwork but not good in group activities. ‫كانت شاطرة مع نفسها‬
What is the Dx?
a. Depression
b. Oppositional defiant
c. ADHD
d. Dyspraxia
e. Drug abuse
f. Autistic spectrum disorder

334
Psychiatry

Jan 2020
1. 15 years, pain in neck and shoulder for 3 months, go to bed late, arouse
multiple times at night, loss of weight, mom said she skips meals but the
girl denies, history of parental separation and living with her grandfather
who died soon, on examination she was restless, withdrawn with poor eye
contact.
a. Generalized anxiety disorder
b. Evolving psychosis
c. Anorexia nervosa

2. Boy about 8 years make trouble in home and school teacher and parent
complain, disruptive, unfollow rules, deteriorates in school, poor
concentration.
a. oppositional defiant disorder
b. ADHD
c. conduct disorder

3. case of conduct disorder 2 choice character


a. cruel with people and animals
b. stealing
c. lies

4. ADHD treat by methylphenidate, weight loss, controlled


a. Shift to atomoxetine
b. Continue, review take measurement after 3 months
c. Stop methylphenidate
d. Refer to dietician

335
Psychiatry

Sept 2019
1. child 10-years old has no friend on school, attach to mother, remain
isolated in their own room quite baby stay long time look at sky or phone
not sure enjoy his own world poor eye contact ,not angry when children
took his toy, walk at 14 months, speech on 2yrs, does not pointed to thing
a. autistic spectrum disorder
b. ADHD
c. attachment disorder
d. personality disorder

2. 3-years old case of girl sleep at 7 pm and wake up start to jump and
crying not remember the episode (not sure) parent concerns what to
do?
a. reassurance Night terrors
b. EEG with sleep deprivation

3. patient with ping eating BMI 28, self-induced vomiting


what is most suggestive of eating disorder in this patient?
a. laxative abuse
b. fine hair in back
c. delayed menarche

May 2019
1. 13-yrs Girl with anorexia nervosa teenage BMI less than 14.5 serum k 3.4
which of the following indicate high risk?
a. BMI
b. k level
c. axillary temp less than 34
d. bp 85/60
e. weight loss <15kg

336
Psychiatry

2. patient presented with binge and self-induced vomiting. Treatment?


a. Admission
b. control diet with calories
c. cognitive behavioural therapy
d. group therapy for child

3. patient with different foster parents have no friends, no problem if carer


is there or no ,friendly with strangers
a. ADHD
b. attachment disorders

Jan 2019
1. Mild to moderate ADHD first line ttt:
a. family cognitive behavioural therapy
b. methylphenidate

2. A case of 4 months duration of muscle aches and fatigue ,what is most


suggestive of the diagnosis of chronic fatigue syndrome?
a. Un refreshing sleep
b. Participating in school running competitors
c. shortness of breath on exercise

3. A case of anorexia nervosa BMI 14.5, what is against the diagnosis of


anorexia nervosa?
a. Low potassium and high cortisol
b. Abdominal pain complaints
c. Amenorrhoea
d. dressing in baggy clothes

337
Psychiatry

4. A girl in foster care I think with low IQ with history of escape from home
and school was found once near club with alcohol intoxication She looks
older than her years ,quite ,withdrawn significant weight loss, sleep
disturbance?
a. Depression
b. Sexual abuse
c. Sexual exploitation
d. drug mis use

Sept 2018
1. 14-yrs old girl, polyarticular juvenile arthritis , her condition didn’t require
emergent treatment ,you discovered joint deformity as she don’t take her
methotrexate , she is withdrawn and didn’t want to talk about ttt , after
discussion she and her family think she is better without treatment
what you do ?
a. refer to CAMHS
b. police protection
c. emergency protection order
d. outpatient appointment
e. non-emergency social service

2. 14-yr-old Girl say she hear voices, which will rise most clinical concern?
a. hearing people she knows in last attacks of migraine
b. hear friends’ voices when they are not here which bother my mother
c. voices telling bad things about her
d. heard newsreader on TV telling her not to do homework
e. voice calling my name while want to sleep

338
Psychiatry

3. Picture of 13 years old boy with 6 month of losing weight, normal bowel
habits, poor appetite ,active and likes to play sports , glucose = 7, no
change in behaviour in school and home
a. coeliac sept 2020 ‫مكرر‬
b. DM
c. anorexia nervosa
d. lanugo hair

2. 15-yr old Girl with upper respiratory tract infection absent from school for
1 month feels difficult to go back, muscle ache for 2 month, feels cold
chronic fatigue syndrome ,family has problems to keep with her medication
what 3 management ?
a. cognitive therapy
b. graded exercise
c. laise education &welfare officer

3. Type 1 DM teenage girl, 6 weeks intermittent vomit, increase HbA1C from


52>>78 , weight dropped 2 kg in 3 months , HR = 56 , BL pr = 100/72,
ketones +1 , NA& K lower limits , cause of her weight loss ?
a. Addison
b. Bulimia Diabulimia is a media- term that refers to
c. Thyrotoxicosis an eating disorder in a person with
d. DKA diabetes, typically type I diabetes,
wherein the person purposefully restricts
e. poor adherent insulin in order to lose weight.

4. Child living with foster carer, always active in home and school ,disruptive
behaviour always on the go, that’s the third foster care , his gaining weight
, in good health
a. ADHD
b. Neglect
c. Anxiety

339
Psychiatry

May 2018
1. what are the features of conduct disorder?
a. Cruel to animals and human
b. Stealing

2. 8-years-old child disruptive at school develop unilateral tics what is your


diagnosis?
ADHD

3. a child with vague symptoms waking at night moving from his bed and
crying but return to his bed and continuous his sleep, he can not recall
the event, there is step father, with mother with agoraphobia
a. night mares
b. Night terrors
c. Centrotemporal seizure
d. Complex partial seizure
e. Psychosis

4. 8 years old with recent onset of development of facial tics (in paper 1)
ADHD

5. 17-years-old girl with eating disorder binge eater, BmI 19 they asked
which factor will point toward it ?
use of laxative also excessive vomit & russel sign

Jan 2018
1. child academic achiever has BMI 18 what would suggest diagnosis other
than anorexia nervosa?
a. low K and high cortisol
b. abdominal pain
c. BMI
d. Bradycardia
e. Amenorrhea

340
Psychiatry

2. Took paracetamol over dose level is below toxic effect what to do?
a. discharge with parents
b. psychiatric assessment
c. do pregnancy test

Sept 2017
1. EMQ:
A. conduct disorder
B. Asperger
C. generalized anxiety
D. learning difficulty
E. Landau-Kleffner syndrome
F. ADHD
All 6-years with deteriorated academic achievement
A. Has a problem with his beers Conduct disorder
B. Has hearing problem --- Landau-kliffner
C. ADHD

2. management of temper tantrum?


a. Ignore and behaviour therapy referral

May 2017
1. child on treatment for epilepsy was controlled on Na-val diagnosed with
ADHD and seizers reoccur after methylphenidate what to do??
a. Stop the methylphenidate

2. ADHD first line in management?


Behavioral therapy

341
Psychiatry

Jan 2017
1. A classical case of ADHD. . asks about diagnosis

2. A question asks about suicide intent signs. . Asks for 2 options ?


a. deep cut wounds
b. deliberate ingestion of 10 tablets of paracetamol
c. previous admission with alcohol toxicity

3. criteria for chronic fatigue syndrome


cognitive impairment (NOT improvement with rest)

May 2016
1. Girl with criteria of chronic fatigue syndrome asked about management?
a. Serotonin
b. admits for intensive exercise
c. Bed rest
d. slow exercise
e. School welfare

2. EMQ
A. A 3-year-old child he says NO to his parents and his other children's no
body like to play with him Oppositional Defiant Disorder
B. A child with behaviour problem and poor attention as well as parents
notice hyperactivity at home. ADHD

342
Psychiatry

Jan 2016
1. Early teenage has argue with her family b/c of her boy friend , she took
Many paracetamol tabs , in ER she was fine and paracetamol level was low
not need administration of N Acetyl cystine , she deny pregnancy or any
emotional problem, what is next action:
a. Test for pregnancy
b. Take her back to go with her parents
c. Refer to psychologist

May 2015
1. question about anorexia nervosa. Asking which feature is less likely to be
because of anorexia nervosa
Abdominal pain

2. child with central abdominal pain, all investigations normal.


what to do next?
a. psychology referral

Sept 2014
1. EMQ: behavioral
A. ADHD
B. Asperger
C. Conduct disorders

2. Management of tantrum
a. Ignore.
b. Reward.
c. Distraction

343
Psychiatry

Jan 2013
1. 12-years old with severe learning difficulties attending school for learning
disability exhibits a sudden change in behaviour difficult for his parents to
cope with. what next appropriate step will u take?
a. take a detailed history from the parents
b. take a detailed history form the school
c. check for the buccal cavity
d. urine toxicology screen
e. other options regarding alcohol ingestion.

2. A child girl is very friendly with strangers. sitting on their laps, sexualized
behaviour taking her pants off in other children, no significant antenatal
and perinatal complications. expect that the mother was a substance
abuser n is a prostitute; the girl was given in the care of foster parents for
two years. her convulsions have settled now after treatment but her
behaviour is much more sexualized, what to do now?
a. stop anticonvulsant
b. remove from foster care
c. arrange child psychiatrist

344
Musculo-
skeletal

345
Musculoskeletal

Sept 2021
1. 14 years old boy, not obese, BMI 18 with Hx of playing rugby sport. He
had 1 month history of left groin pain with pain in left hip joint which was
internally rotated. Cannot flex his thigh and pain is increasing by cough.
What is the diagnosis?
a. SUFE
b. Chronic epidedmitis
c. Osteosarcoma
d. Perth’s
May 2021
1. A Patient came with persistent cough for 2 weeks. A chest X-ray was
requested, what is the abnormality?
a. Pneumothorax
b. Hyper inflation
c. Skeletal dysplasia
d. Hilar lymph nodes
e. Right middle lobe consolidation

2. An Asian child comes with a 5-month history of lower limp pain and deep
pain in shoulder and leg. O\E: He was pale with quadriceps muscle wasting.
The CBC was normal, AST and ALT (slightly high) but GGT and ALP were
within normal range. What TWO investigations could make a diagnosis?

a. CK
b. ESR
c. Liver biopsy
d. Vit D
e. MRI thigh muscle
f. Viral serology

346
Musculoskeletal

3. EMQ
A. Lyme disease (1)
B. Reactive arthritis (3)
C. Septic arthritis
D. Acute rheumatic fever (2) --(100% cardio,00% MSK)
E. ALL
F. JIA

A. A 14-year-old who was in a camp presented 6 weeks later with


lethargy, right knee joint pain and swelling. The CBC showed
neutrophilic leucocytosis.
Lyme disease

B. A 6-year-old presented with over a month left shoulder pain that


moved to the left wrist, after that she developed Rt knee pain. Now all
symptoms resolved but there was a systolic murmur over the apex.
Acute rheumatic fever

C. A 5-year-old boy presented 10 days after having a URTI with pain in


the right knee and left wrist, there was no fever nor swealing.
Reactive arthritis
Jan 2021
1. x ray of a girl 4yrs on wheelchair,
developmentally delayed, became irritable
when lifted, diagnosis?
a. Perth’s
b. slipped upper femoral
c. dislocated hip
d. fracture head of femur

347
Musculoskeletal

2. A mother known heterogeneous sickle cell, brought her 5 months old baby
with a 3-day history of leg pain history of falling from sofa and became
more irritated during nappy change. In electrophoresis baby is
homogenous and positive neonatal screening, has URTI 2days before,
snuffle nose, temp 38, diagnosis?
a. Osteomyelitis
b. Septic arthritis
c. NAI
d. Sickle cell crisis

3. 5 years old boy, obese with history of limping for 6 months, on


examination hip not tender or hot, diagnosis?
a. Perth’s disease
b. SUFE
c. dislocated hip
Sept 2020
1. 10 years old girl with skin rash on the hands on the knuckles and
violaceous rash under the eyes with difficulty in writing and stiffness in
the small joints in the hand. asking about Two diagnostic investigations
at this stage?
(Eyes were covered; hand was also covered by white gauze!!!)

a. Ck
b. EMG ?
c. ANA
d. ESR
e. MRI brain
f. Muscle biopsy
g. Skin biopsy ?

348
Musculoskeletal

2. 6-weeks ago a boy presented with right limping and knee stiffness after
a fall and was treated only as mechanical injury and discharged with
ibuprofen and now limping improved but came with dilated left pupil,
what is the important action?
a. Urgent MRI
b. Schedule MRI
c. Neurosurgery
d. MRI knee
e. Urgent ophthalmology referral

3. 14-month child X-ray picture with history of delayed walking dx?


RIGHT DDH

4. Overweight 12 years girl presented with limping, what dx?


a. SUFE
b. Perthes disease
c. DDH

5. A newborn was delivered well. On


examination: large head and narrow
chest, relatively short limbs, developed
respiratory distress after 4 hours
what is, dx?
(X ray with fracture femur or telephone
handle vs fracture tibia with normal
femur (
a. Achondroplasia
b. Thanatothrophic dwarfism
c. Osteogenesis imperfect

349
Musculoskeletal

Jan 2020
1. X-ray of Hip joint 13 years old girl leg shorter than the other Rt side
repeated episodes of pain
a. SUFE
b. dislocated hip
c. Perth’s
d. Femoral head
necrosis

Like this picture but


unilateral (right hip)

2. 7yr with 1 yr. bone pain, woke him from sleep x ray normal, all
investigation normal, diagnosis?
a. Behavior problem
b. Osgood
c. Leukemia
d. JIA
e. Lymphoma
f. Benign hypermobility syndrome

3. Scenario 4 yr. old girl, have knee joint swelling , HSM , pink rash ,fever ,
lymphadenopathy, high ESR, CRP, high platelet:
a. Systemic onset JIA
b. ALL

350
Musculoskeletal

Sept 2019
1. PT 2yrs with history of 2nd degree IVH tip toe waking but can stand
on heel can run & jumb, not yet toilet train, sacral dimple
a. habitual tip toe waking
b. hereditary motor and sensory neuropathy
c. spastic diplegia

May 2019
1. X ray of child with grand mother with history of trauma and had swelling
in the head
Left parietal fracture

351
Musculoskeletal

2. EMQ:
A. Spinal tumour
B. Ankylosing spondylitis
C. spinal neuroma
D. Trauma
E. benign muscular pain
F. epidural abscess
G. pyelonephritis

A. 9 years boy fall from bycicle 2 weeks back ...c/o of back pain for 1 month
temp 38 Ankylosing spondylitis
B. boy back pain for 1 year skin marks neuroma
C. numbness in left LL wbc 22 temp 39 sever back tenderness
epidural abscess

3. 2 yrs Acute history fever 39 shivering awake in night with rt iliac fossa
pain hip flexion and slight external rotation for one day any hip
movement result in sever pain Blood picture showed wbc 22
raised CRP,, no abdomen guarding

(in towards case)


A. DIAGNOSIS?
a. Psause abscess
b. Appendicitis
c. septic arthritis

B. INVESTIGATION of choice?
a. MRI HIP
b. bone scan
c. ABD u/s
d. CT abdomen and pelvis
e. pelvis and abdominal x-ray

352
Musculoskeletal

Jan 2019
1. x-ray of a girl 4yrs on wheelchair, developmentally delayed, became
irritable when lifted, diagnosis?

a. Perth’s
b. slipped upper femoral
c. dislocated hip

2. A boy with high blood pressure, fever, rash in the face which changes
with sun, muscle aches,(proximal myopathy),difficult to climb
stairs, cannot get out of bed easily?
a. Dermatomyositis
b. SLE

3. Xray of spine, patient has


backache , arthritis and on
long term oral
steroids ?(2 answers)
a. Discitis
b. collapsed vertebral body
c. L1 , L2 lytic lesions
d. compressed spinal cord
e. osteoporosis

353
Musculoskeletal

4. A mother who has sickle trait, URTI 2days, brought her baby with a 3 day
history of leg pain history of falling from sofa which became more
prominent during nappy change, in electrophoresis baby homogenous
and mother trait temperature 38, diagnosis?
a. Osteomyelitis
b. Septic arthritis
c. NAI
d. transient synovitis

5. X-ray of a child who has pain around the knee, no swelling, no redness?
a. osteosarcoma
b. osgood schlater
c. ewings

Sept 2018
1. 14 year Asian .. limp pain for 5-month, normal all examination,pale, mild
wasting quadriceps, deep pain in thigh and shoulders, CBC, RF,CRP -ve,
3 initial investigation???
a. Vit D
b. Bone scan
c. ASO titer
d. Mantoux test
e. Xray lower limp
f. BM aspiration
g. Borelia serology
h. ESR
i. TFT
j. Ck

354
Musculoskeletal

2. EMQ
A. 6-yrs-old girl with breast development , hyperpigmentation , fractures
mcCune-albright
B. Multiple fracture, hearing problems, callus formation ,repeated trauma
osteogenis imperfecta
C. NAI

May 2018
1. 7-years-old with fall down lumpiness that improve now but 6-weeks later
developed irregular dilated pupil
ophthalmological opinion --- uveitis

2. 8-years-old girl developed recently lower limb weakness and change of


behaviour has nodules on her knuckle asked for investigation to confirm
(2 options) (dermatomyositis )
a. MRI
b. CK
c. EMG

3. EMQ:
A. 3-years with URTI
Transient synovitis
B. 4-5 years limping and limitation, muscle wasting, long duration
Perth’s disease
C. hypothyroidism, obese, developed sudden pain in the hip
SUFE

355
Musculoskeletal

4. previous fit child with conjunctivitis, cough, polyuria, polydipsia


intermittent vomiting and increased urea and creatinine ,high calcium
high ALK p, low phosphate
Sarcoidosis

Sept 2017
1. known case of cystic fibrosis with fever joint pain which will be helpful for
diagnosis?
a. joint aspiration ??
b. bone scan
c. blood c/s??

May 2017
1. case of joint pain at night child scream from pain exam normal exam
a. hypermobility joint syndrome
b. Osgood schatler
c. Growing pain

2. GIRL WITH long scenario has past history of rash joint problem and renal
problem
a. SLE
b. TB
3. Infant with severe crying at night, presented with external rotation of the
hip, on examination free knee and ankle movement, pain with any hip
movement, abdominal guarding in the lower RT abdominal, high CRP
A. diagnosis??
a. Hip osteomyelitis
b. Psoas abscess
c. Hip arthritis
B. Investigation??
a. CT abdomen
b. X-RAY

356
Musculoskeletal

4. obese child with slipped epiphysis scenario—SUFE


Jan 2017
1. History of delayed walking in a male 2 years old
Same picture in radiology book
DDH

2. x-ray of collapsed vertebrae

With:
TB
Ankylosing Spondylitis

3. EMQ about arthritis


A. a child developed arthritis 6 weeks after a camp visit, Skin rash
JIA
B. child with upper limb pain after an infection
reactive arthritis
C. heart murmur, Skin rash, Arthritis
rheumatic fever

357
Musculoskeletal

May 2016
1. Teenage with BMI 32 has knee pain and abnormal gait she cannot
weight bearing Dx?
a. AML
b. perthes disease
c. slipped femoral epiphysis

2. Boy has bilateral knee pain, fever and rash ASOT ! , ANA +ve !
a. Rheumatic fever
b. JIA

May 2015
1. A CHILD WITH TIP TOES WALKING AND A SACRAL DIMPLE..
a. Tethered cord syndrome Lesions
b. charcot marrie tooth
c. habitual toe walking

2. there was an x-ray --- SUFE

358
Metabolic

359
Metabolic

Sept 2021
1. 4 years old girl who was known as a case of citrullinemia, Presented with
vomiting. What is investigation to help in management?
a. Blood glucose
b. Blood c/s
c. Blood ammonia

2. 3 wks. Baby presented with vomiting, drowsiness, poor feeding and he


was afebrile. There was family history of +ve consanguinity.
Investigations were done and antibiotics started. Investigation showed
RBS 11, CBC, U&E were normal. Serum ammonia was 350. What is next
management step?
a. IV D10 + 0.9% NaCL
b. NGT feeding
c. LP
d. Peritoneal dialysis
e. Urine analysis
f. CT brain

3. 3 days old neonate presented with vomiting and refusal of feeding. On


examination was afebrile and hypotonic. Investigations:
ABG: PH 7.2, BE -18, RBS 1.3, CRP 9. What is the cause? ‫خالف‬
‫ي‬ ‫)سؤال‬
a. HIE
b. MCAD?? (100% Metabolic)
c. Sepsis
d. Organic academia (100% Metabolic)

4. 14 years old girl with ammonia= 1350. Patient was drowsy and tachypnic.
ABG PH 7.5, PCO2: 2.6, HCO3: 22, other labs were normal. What is
diagnosis? (100% Metabolic)
a. Organic academia
b. UCD
c. RTA

360
Metabolic

May 2021
1. A neonate presents with vomiting and collapse Investigations: Ammonia
was 60 (45 –60) Glucose: normal , Na: 135 , K: 4.5 , Cl: 97 , PH: 7.2 , PCO2:
25 , HCO3: 9 What is the most likely diagnosis?
a. GSD
b. MCCAD
c. RTA
d. PKU
e. MSUD

2. A 6-week-old girl came with jaundice, not feeding well and hepatomegaly.
Her brother died of E. coli sepsis What is the diagnostic investigation?
a. Immunoglobulin's level
b. GALUT test
c. TORSH screen

Jan 2021
1. Ex preterm 26 weeks, now 4 months with underlying metabolic bone
disease of prematurity, what is the most important during for follow
up?
a. parathyroid hormone
b. Ca
c. Po4
d. Vitamin D
e. urine ca: creatinine ratio

2. infant 3 month roving eye & hypoglycemia glucose 1.2, pallor , drowsy
proceeded by coryzal illness.
a. septo optic dysplasia
b. neuro blastoma
c. retinoblastoma
d. MCAD
e. Galactosemia

361
Metabolic

3. Neonate with jaundice mainly conjugated, prolonged PT, PTT, deranged


LFT, normal blood sugar, urine reducing substance negative, diagnosis?
CMV
a. Galactosemia
b. alpha 1 antitrypsin deficiency

4. Baby boy with sever acidosis, ammonia 950


OTC

Sept 2020
1. A full term neonate who was well, on day 2 the baby was found collapsed
in his cot and unresponsive, PH 7.49, CO2 3.5, RBS 3.5, (respiratory
alkalosis) polycythaemia, hypotonia, LFT normal.
What initial investigation to help you in diagnosis?

a. Serum ammonia
b. Organic acids
c. Amino acids
d. GBS
e. Blood culture
f. Serum lactate

2. Neonate who collapsed at 3rd day had history of PROM 24h hours CS
breech took vit k admitted for IV antibiotics and fluids then collapsed
rapidly with hypertonia and seizures needed respiratory supporting Inv
shows: Ph7.21 (Respiratory acidosis), Ammonia was level 1454 umol/L
Hb is 23, What is the cause?
a. Polycythemia
b. Urea cycle defect
c. Herpes encephalopathy
d. GBS sepsis
e. HIE

362
Metabolic

3. Uncle died with MI, father takes his 2 children; 5 and 7 years to check.
What to do initially?
a. Check father lipid profile
b. Exercise ECG for father
c. Lipid profile for the children
d. Exercise ECG for children

JAN 2020
1. Long scenario about 11 months jaundiced, hypoglycemia high
triglycerides, hepatomegaly 10cm, glucose 4, uric acid high
A. What is the diagnosis?
a. GSD
b. Neuroblastoma
c. Auto-immune hepatitis
d. Wilson
B. What is the confirmatory test?
a. liver biopsy
b. bone marrow
c. cupper level

2. The boy 4 yr short with hepatomegaly and faltering growth since age of
6-months, father has one functioning kidney,he wasn’t icteric, liver 10 cm,
no spleen, soft liver,AST,ALT normal , increase cholesterol, high urate, low
socioeconomic, history of febrile seizure, not dysmorphic.
a. Glycogen storage disease
b. Mucopolysacaridosis
c. autoimmune hepatitis
d. Galactosemia

363
Metabolic

Sept 2019
1. long case of patient with blood gas show alkalosis ph 7.55 and high
ammonia 1300
a. urea cycle defect

2. case of metabolic with normal glucose metabolic acidosis ,and given U&E
result normal ,,ask diagnose ammonia was normal,,(if you calculate anion
gab it was high 37.5) URINE ph 5
a. urea cycle defect
b. MSUD
c. phenyl ketone urea
d. glactosemia
e. renal tubular acidosis
f. MACAD

3. screening test was done on mother demand because father died at 40yrs
with MI, cholesterol high triglyceride normal, what will be abnormal?
a. LDL
b. ECG

May 2019
1. GSD SENARIO hepatomegaly, g6phosphatase low, associated with?
a. high triglecride
b. hyper glycaemia
c. cataract

2. SENARIO OF jaundice, hepatomegaly, his brother died with e coli sepsis )


ASKED ABOUT DIAGNOSTIC INVESTIGATION:
GALPUT 1

364
Metabolic

Jan 2019
1. EMQ:
a. glucose polymers (read about it please)
b. maintenance 10% glucose+ saline .45
c. 20 ml saline
d. glucose bolus 2ml/kg 10% and maintenance
e. Measure acylcarnitine
f. check the degree of metabolic compromise
g. ORS through NGT
h. nothing discharge
MCADD Vomiting his oral formula:
A. Glucose normal 4,5 , 5-10% dehydrated
maintenance 10% glucose+ saline .45
B. Glucose decreased RBS 1,8 , 5% dehydrated
glucose bolus 2ml/kg 10% and maintenance
C. Glucose normal, not dehydrated
glucose polymers

2. patient with hepatomegaly, glycogen storage disease ,


asking about investigation ?white cell enzymes

3. EMQ:
A. diffuse cortical atrophy
B. cerebellar atrophy
C. Tay Sach
D. leigh disease
A. spastic child, exaggerated startle, ataxia, white matter change, CSF
protein high metachromatic
B. pale optic disc ............batten
C. menkes disease

365
Metabolic

Sept 2018
1. Case diagnosed with citrullinemia, normal respiratory rate, deteriorates,
best initial investigation to decide management?
a. blood gas
b. blood ammonia
c. blood culture

2. Scenario infant 3 month roving eye & hypoglycaemia glucose = 1.2 , pallor
drawsy, coryzal illness
a. Septo-optic dysplasia
b. neuro blastoma
c. retinoblastoma
d. MCAD
e. Galactosemia

3. uncle 35-yrs died with ischaemic heart disease, father bring children 5 and
6 -yr-old, he wants to check if they will have it too , what to do ?
a. check father lipid profile
b. fasting lipid profile of children

May 2018
1. EMQ:
A. 2 café au lait patch, murmur, learning difficulty
Fanconi anaemia
B. psychomotor regress, hypotonia, weakness, encephalopathy,
nystagmus, ataxia, deafness, myoclonic epilepsy, cardiomyopathy
Leigh syndrome

C. pancreatic insufficiency, malabsorption, fail to thrive, neutropenia,


gingivitis, Hhepatosplenomegaly
Schwachman-diamond

366
Metabolic

2. EMQ:
A. baby with microcephaly, seizure. Rash PKU
B. developmental delay, myopathy Liegh disease
C. have posterior suture open, myopathy, constipation Hypothyroidism

3. baby 9-days-old with hepatomegaly and UTI symptoms, reducing


substance =negative with jaundice and hepatomegaly
a. GSD
b. Galactosemia

4. jaundice, hepatomegaly, RBS 4 mmol/l


Galactosemia

Jan 2018
1. 4-yrs-old with increasing abdominal size had history of convulsions
diagnosed as febrile ones at age of 2 yrs had mild learning difficulties had
a brother reported as normal he walked at 20 months, on examination
liver was 7 cm below costal margin, height < 2nd centile, weight 50th
centile no splenomegaly, labs albumin 43, glucose 3, urine was clear with
no reducing substance AST 47 , cholestrol 7 (3.9 - 6), uric acid 0.6 ,ca 2.6
phosphate 1.6,
A. what to do?
a. Alphafeto proteins
b. Hepatts serolony
c. Bone marrow biopsy
d. HIDA
e. Autoimmune serolony
f. Liver biopsy

367
Metabolic

B. What is you diagnosis?


a. GSD
b. Budd chiarri malformation
c. Autoimmune hepatits
d. Neuroblastoma
e. Portal vein thrombosis
f. Wilson
g. Chronic active hepatits

2. child with hepatomegaly and hypotonia low G-6-phosphatase i think it was


in his family what it is associated with? ( ?? GSD 1)
a. cataract
b. low platelets
c. hyper triglycerides
d. Hypogonadism

Sept 2017
1. Previous baby cot death, mother on fluoxetine, high liver enzymes?
a. fatty acids oxidation
b. MCAD MACAD:

a. history of previous SID


b. Non ketotic hypoglycaemia
c. deranged liver enzymes

2. Uncle died of MI - check fathers’ lipid

3. 4-weeks with diarrhea, vomiting and fever, dehydrated acidotic??


a. Gastroenteritis
b. propionic academia ?

368
Metabolic

May 2017
1. Baby born in India, hypoglycaemic and fits . Normal LFTs. urine -ve for
ketones and reducing substances .ammonia 65 . Partially compensated
severe metabolic acidosis . Normal plasma organic acids .
Urine : deoxy ..acid
a. OTC
b. MCAD
c. GSD

2. neonate 6 weeks with bilateral subdural haemorrhage refuse feeding


jaundice
a. Glutaric aciduria
b. NAI
c. Haemophilia

Jan 2017
1. case history of hypoglycaemia Family history of sudden death in
previous baby
MCAD

2. a case of citrullinemia . .most app Investigation?


a. Ammonia
b. blood gas

May 2016
1. 6-month boy with history of hypoglycaemia parents has child with Hx of
SIDS (sudden infant death syndrome), urine ketone (-ve) Urea 60 ask
about Dx?
a. urea cycle defect (urea will be too high & respiratory alkalosis)
b. GSD (ketotic)
c. MCAD (non ketotic)

369
Metabolic

2. Baby with jaundice, vomiting, diarrhea, hepatomegaly, hypoglycemia,


urine contain reducing substance
Sepsis due to galactosemia

3. Mother with poorly controlled PKU, presented with prolonged rupture of


membranes what will be problem to the baby?
a. CHD
b. Sepsis
c. Poor control PKU

4. 5-days old baby, poor feeding, lethargy, mild rise in ammonia, severe
metabolic acidosis
a. UCD
b. MCAD
c. Organic aciduria

May 2015
1. There was a data where with fasting glucose dropped and child developed
hypoglycaemia and acidosis..ketones +..?
a.Glycogen Storage Disease [ but also scenario incomplete to support it]
b. ketotic hypoglycemia

2. a question with child with features of galactosemia. Investigation?


GAL -1-PUT

Sept 2014
1.Previous baby cot death, mother on fluxetine?
Fatty acids oxidation defect
Reyes syndrome

370
Metabolic

2. 4 weeks with diarrhea, vomiting and fever, dehydrated , acidotic


Proprinc acideamia

3. Uncle died of MI, sibling also early affected and expired


Check father’s lipid

4.Increased cholesterol triglycerides, hypoglycemia and jaundice


GSD…..Liver biobsy

5. baby not fed night time morning, RBS – low, Ketone in urine +
Ketotic hypoglycemia , or MACAD ????

Jan 2013
1. father had heart attack and died at 40, mother bring the child and insist
to investigation examination: free, his investigations U&E, CBC>>all
normal except, high cholesterol level (with normal triglycerides)
WHAT NEXT TO DO?
a. LDL level
b. homocystinuria genotyping

2. neonate IUGR with small head, come with seizure his investigations
show: hypoglycaemia, hypocalcaemia, thrombocytopenia seizure persists
next day what to do?
a. see urine organic acids levels
b. see serum AA level
c. try pyridoxine

371
Emergency

372
Emergency

Sept 2021
1. Child with G/E manifestation presented with dehydration, CRT 3 sec,
S.Na: 119, RBS: 3.2, other labs were NL, during examination he developed
tonic clonic convulsions. Which type of IV fluid you should give??
a. 20ml/kg NS
b. 2 ml/kg D10
c. 4ml/kg 3% saline
d. 10ml/kg NS

2. 2 yrs. has 2 days vomiting and loose motions, her vital signs were PR 120,
RR 28, temp 37.5, GCS 10, CRT 4 seconds and RBS 3. Half hour after
arrival her vitals were PR 160, RR 20 irregular, temp 38, GCS 8 and
SpO2 92. What next step?
a. Bullous D 10
b. Bullous NS
c. IV Antibiotics
d. Secure airways

3. X-ray chest baby 18 month has been well and there was decoration at
home, presented with cough, wheeze and decrease air entry in LT side.
What is the Dx?

a. CCAM
b. Right side foreign body
c. Left side foreign body
d. Rt side air trapping
e. Left sided pneumothorax

373
Emergency

4. Scenario about child with tachypnea, vomiting, abdominal pain,


examination of chest is free, heart is normal, dehydrated, decrease
skin turgor, dry tongue, blood gases showed severe metabolic acidosis
PH was 7.1, high serum osmolarity, RBS 9. Asking about the diagnosis?
a. Hypernatremic dehydration
b. DKA
c. Viral lower respiratory infection

5. Scenario about amitriptyline ingestion, in an infant living with his


grandmother, develop tachycardia, HR was 150, mydriasis, wide complex
tachycardia, pt. was hemodynamically stable. Asking about initial TTT?
a. Synchronized DC shock
b. IV amiodarone
c. IV adenosine
d. IV NaHCO3

May 2021
1. An 8-month-old patient presented with their mother after a fall 30
minutes back. There was a bruise of 6 cm in the head, no vomiting and
GCS was 15. He is otherwise well. What to do?
a. CT within 4 hours
b. CT within 1 hour
c. Discharge
d. Observe for a while then ask her to come back if symptoms appeared
e. Skeletal Survey

2. A child with RTA was given fluid boluses and blood transfusion. After 12
hours the BP was 130/70 but he was anuric and investigations showed K of
8 and creatinine of 200. What are the next TWO initial management steps?
a. Give another fluid challenge
b. Nebulized Salbutamol
c. Peritoneal dialysis
d. Immediate haemodialysis
e. Diuretic challenge

374
Emergency

3. A 3-year-old child came with an acute onset of fever and drooling. He was
toxic. What is your action?
a. Intubate in theatre
b. O2 by facial mask
c. I.V antibiotics

4. An intubated child was arrested in the ambulance. What would you do?
a. Uninterrupted cardiac compression with breaths
b. 15 compressions with 2 breaths
c. 3 compressions with 1 breath
d. 5 inflation breaths

Jan 2021
1.
A. Neonate with hyperkalaemia 8.3, hyponatremia, vomiting. metabolic
acidosis, normal glucose, He asked about 2 actions after resuscitation?
The patient received 2 saline boluses and still hypotensive and has
prolonged CRT.
a. hydrocortisone iv
b. Ca gluconate
c. iv insulin and glucose
d. Sodium bicarbonate
e. salbutamol neb
f. repeat saline bolus.

B. Second question asking about diagnosis?


a. Congenital adrenal hyperplasia
b. Gastroenteritis

2. Child in HDU with tracheostomy tube on O2 mask suddenly out of


breath silent chest, cyanosis. What’s the immediate action?
a. Suction from tracheostomy tube
b. insert ETT
c. ENT referral

375
Emergency

3. Case of burn, his CRT <2, oxygen sat: 98%, management?


a. saline bolus
b. call anaesthesia
c. start analgesia
d. call burn team

4. 5month came to ER with chocking, the parents asking what they


should do if the choking occurred again in the future? TWO options
a. 5 back blow &abdominal trust
b. 5 back blow & chest trust
c. recovery position
d. Call for help
e. finger sweep
f. 5 inflated breath

Sept 2020
1. X-ray of asthmatic patient came with rapid onset of chest pain and
increasing breathlessness within 2 hours, not improved with nebulizer,
on 6 liters face mask, he is tachypneic, distressed, retraction,
O2 saturation decreased to 88%, What is the next immediate action?
(Pneumothorax with lung collapse on Rt. side)

a. Intubation
b. increase oxygen of
face mask to 15 L
c. iv aminophylline
d. Morphine for pain
e. Urgent Ct chest
f. Large bore chest tube
g. Needle
thoracothentesis

376
Emergency

2. 13 years old child develops sore throat 15 days before admission, then 10
days develops haematuria, 6 days he develops face swelling and headache,
the parents treated by paracetamol now develops tonic colonic
convulsions and GCS was 11. Respiratory rate 20 Blood pressure 180/110
mmHg, urine 3+ blood and 3+proteim he was afebrile.
A. What is the cause of convulsions?
a. Systemic HTN
b. Encephalitis
c. Cerebral vasculitis
d. Brain abscess
e. HSP
B. What treatment to give?
a. Labetalol
b. Phenytoin
c. Acyclovir and IV antibiotics
d. Mannitoll
e. Steroids
f. Intubation and ventilation

3. 2 years child, blood pressure 90/50, HR 200, ABC was done and despite of
normal saline boluses HR remains 210/min, Bl/Pr 80/50, RR 80/min and
on Nasal Cannula still same SPO2 88%, low Hg normal platelets, WBCS,
What to do? Two Options
a. Intubate and ventilate
b. Inotropes
c. Blood transfusion
d. Platelet transfusion
e. Sodium bicarbonate
f. Granulocytes transfusion

377
Emergency

4. A girl after drowning, arrested, developed VT, resuscitated, received 3 DC


shocks and epinephrine. Her Temp is 26 C, PH 6.68, what next?
a. Amiodarone
b. Warm NGT fluids and warm IV fluids
c. Dc shock with Adrenaline again
d. Give Na Bicarbonate
e. Stop resuscitation

5. Anaphylaxis took 150 IM adrenaline at home and reached hospital after 5


minutes, has IV line and was pale CRT 5 sec what to give?
a. Repeat 150 IM adrenaline 1/1000
b. Repeat 150 IM adrenaline 1//10000
c. Normal saline bolus
d. IV hydrocortisone

Jan 2020
1. 2-y old, 12 kg shocked has fever diarrhea, vomiting, drowsy, received ORS,
sibling died SIDS, decrease GCS, pallor, apnea Investigations:
Hypoglycemia (glucose 0.5) hyponatremia, hypokalemia, CRT 5sec, Bp
80/40, respiratory acidosis, ketone urine +, what to give?
a. 240ml saline, 24 ml dextrose 10%
b. 120ml saline, 24ml dextrose 10%
c. 240ml saline, 60ml dextrose 10%
d. glucagon

378
Emergency

Sept 2019
1. EMQ
a. 10ml/kg NS
b. 20ML/KG NS
c. PRBCS 20ML/KG
d. PRBCS 10ML/KG
e. 20% ALBUMIN

A. child fall from tree CRT 3 SEC PAIN in abdomen, PALE BP 90/65
20ml/kg NS
B. baby with meningitis on 50ml/kg maintenance, BP 71/45 CRT 3 sec
10ml /kg NS
C. other patient not remember receive 40ML /KG NS CRT 5 sec
iv 4.5 albumin??

if sepsis give another bolus (up to 60 ml/kg) then consider inotropes

“p.100 in SOP”

May 2019
1. (5month) came to ER with chocking what advices you can give to the
parent if happened again? 2 options

a. 5 back blow &abdominal trust


b. 5 back blow & chest trust
c. recovery position
d. call for help
e. finger sweep
f. 5 inflated breath

379
Emergency

2. intubated patient ARRESTED in ambulance air entry equal next step


a. 15 chest compression and 2 breath then repeat
b. 30 chest compression and 1 breath
c. un interrupted chest compression and breath
d. recovory position

3. baby presented 11:30 PM with his parents with ingestion battery 8.5 mm
detected by x-ray at the oesophagus. what to do?
a. admit for observation
b. admit over night. iv fluids then refer tomorrow for endoscopy
c. urgent referral for endoscopy removal
d. NPO repeat x-ray in the morning if still in place for removal
e. let him eat and drink ...

4. toxic patient with Stridor and drooling what is your action?


a. adrenaline neb
b. intubate in theatre
c. prednisolone

5. patient of trauma NORMAL BP >> GCS 10 what you will give iv fluid
(vague question)
a. 80% of his maintenance of 5% dextrose with 1/2 normal saline
b. bollous 10 ml/kg N/S
c. BOLLUS dextrose
d. albumin 4,5 %
e. 80% of his maintenance of D5% with normal saline

380
Emergency

Jan 2019
1. A case of myasthenia gravis, can't lift her head, shallow difficult
breathing, paradoxical chest movement, appropriate management?
a. non invasive respiratory support
b. intubation and ventilation
c. Edrophonium
d. iv immunoglobulins

2. 15% burn, not tolerating orally best fluid?


a. Hartman solution
b. Normal saline
c. D5% with NS
d. D10% with 0.18 NS

Sept 2018
1. 2-yrs old girl full in pond, pulseless 25 min, intubated good chest movement
now no pulse VF, fixed dilated pupil , rectal temp 26 , PH= 6.96 , co2 = 9.4,
HCO3= 8.3 , BE= -28 , 3 DC shocks 4 J 2 min apart then 1 dose adrenaline , on
life support for 40 mins >> VF , next step ??
a. talk to parent to withdraw life support
b. give amiodarone
c. continue defibrillate every 2 min with adrenaline every 4 min
d. give bicarbe
e. rewarm with saline bolus and gastric lavage

2. Child with peanut allergy in first exposure, mom and dad allergic they
exclude peanuts from house, ask about what to do next pregnancy to
prevent allergy?
a. no measure will reduce risk
b. introduce nuts in small amounts
c. mom eat nuts during pregnancy
d. not eating nuts during pregnancy
e. don’t expose baby to nuts

381
Emergency

3. 6-yrs-old child, generalized urticaria, faint, tingling in lips , vomit after eat
brazeil nuts, has same episodes from two weeks what’s appropriate
management?
a. 2 adrenaline autoinjector to carry and 2 at school all time
b. 2 adrenaline autoinjector to carry & antihistamine
c. 1 adrenaline autoinjector to carry and 1 at school
d. 2 adrenaline autoinjector to carry and 2 at school all time and oral
antihistamine and salbutamol inhaler
e. Oral antihistamine and salbutamol inhaler at home
f. Food challenge

4. 10-months old Baby fall from mother lap , stable ,no vomit , 6 cm
hematoma , next step ?
a. CT brain in 1 hr
b. no thing charge after observation
c. CT with in 4 hrs

May 2018
1. 6-months-old got DAMA and when evening nurse went home to check
saturation it was below 90% what to do?
Call police &ambulance as he is desat

2. x-ray with nail in the right iliac fossa, what to do?


discharge and follow up if in pain

382
Emergency

3. a case of ketotic hypoglycemia with glucose 0.9 with weight of 12 kg,


shocked, asked about management
240ml 0.9 saline (20 ml /kg ) & 24 ml 10% dextrose( 2ml /kg)

Jan 2018
1. A case after Road traffic accident ..Potassium 7.9 ... what is the most
immediate ttt ... ?
calcium iv

2. 2-YRS from Bangladesh has 2 days vomitng and loose motions, HR 120,
RR 28, Temp 37.5, GCS 10, refill time 4 seconds, spo 294 ,sugar 3 half
hour after arrival PR 160 RR 20 irregular temp 38 GCS 8 SPO2 92
what next?
a. bullous D 10
b. bullous NS
c. IV antibiotics
d. Secure airways
e. Urgent CT

May 2017
1. clear scenario of pneumothorax neonate was well controlled on
ventilation then acidotic ABG suddenly +ve transillumination of the chest
what to do?
Needle thoracocentesis

Jan 2017
1. emergency hypertension questions asks about ttt ?
Na nitroprusside

2. case of anorexia nervosa with hypokalaemia and Hypocalcaemia . .


Similar question in specimen paper. . Asks about next action ?
correct hypokalemia

383
Emergency

3. SVT shocked..what to give?


a. Iv saline vs
b. Adenosine

4. case of drowning. . 40 minutes ago. . Severe metabolic acidosis . .


Ventricular tachycardia.. Had adernaline and defibrillator.. temperature
less than 30..what to do?
a. warm iv fluids
b. Correct acidosis
c. Continue resuscitation with amiodarone

May 2016
1. 15-year-old boy with headache (mother gave him paracetamol) went with
his friends and came back night mother when he was watching TV
suddenly collapsed GCS <8 what initial management?
ABC with Intubation
Sample paper
Dx is sub arachnid hemorrhage

2. A case of abd pelvic X ray child swallowed pin ( ‫ ) دبوس‬some hours earlier
pin appeared at lower bowel what’s your immediate management?

a. abdominal laparotomy
b. arrange for endoscopy
c. discharge and come next morning
d. discharge and ask parents to watch in stool
e. admit for observation and conult pediatrics
surgery (risk of perforation)

384
Emergency

Jan 2016
1. Girl 13 y old had road traffic accident and blunt trauma given fluids and
blood transfusion became after that tachypneic and anuric, K + 8 mmmol
creatinine 200 , what 2 management :
a. Neublized salbutamol ‫لغاية ما تجهز ال‬
dialysis
b. Hemodialysis
c. Perotoneal dialysis
d. Diuretic challenge

2. Hypoglycemic child with hypotension ask about fluid management ....

May 2015
1. A child with collapse..ecg showing torsades/VT..management
DC shock

2. child with hyperkalemia.2 steps of management ?

3. a child with epiglottitis...?


a. intubates in theater
b. give dexamethasone

4. a child with rt sided heart failure..low BP..what to do next?


a. frusemide
b. dobutamine
c. prostacyclin

385
Pharmacology
poisoning
Pt safety

386
Pharma, poisoning, Patient safety

Sept 2021
1. Teenage girl presented to ER at 10 am has taken12 tablets of paracetamol
at 11 pm then 8 tablets at 4 am then 4 tablets at 7 am, paracetamol level
was checked = 110 mg/l and INR 1.4 and liver enzymes within normal
range, high urea and creatinine. What are the suspected complications in
the next 72 hours? 2 answers:
a. Renal tubular necrosis
b. Hypoglycaemia
c. Hypotension
d. Coma
e. Hypertensive encephalopathy
f. Metabolic alkalosis
g. Haemorrhage

2. Asthmatic child came with sever attack. Received all medications then
saturation improved and air entry is good (clinically improved). ABG
showed metabolic acidosis PH 7.1, lactate 6.what is the cause?
a. IV MgSO4
b. IV salbutamol
c. Neublized ipratropium bromide
d. IV hydrocortisone

3. Case of girl with visual and auditory hallucination, came with


grandmother, she was well except that she has irritable bowel syndrome
and given antihistaminic and antispasmodic, what is cause of her
manifestation?
a. Drug interaction
b. Drug Abuse
c. Conversion

387
Pharma, poisoning, Patient safety

4. Drug interactions. 2 answers:


a. Azithromycin with ketoconazole
b. Erythromycin with cyclosporin
c. Cysplatin with erythromycin
d. Amoxicillin with clavulanic acid
e. Ciprofloxacillin with gentamycin

5. What is the treatment?


a. IV floxacillin
b. Topical fusidic acid

6. 14 years old with cQT 490 msec. What is the treatment?


a. Enalapril
b. Propranolol
c. Atenolol
d. Digoxin
e. Amiodarone

7. Scenario about amitriptyline ingestion, in an infant living with his


grandmother, develop tachycardia, HR was 150, mydriasis, wide complex
tachycardia, pt. was hemodynamically stable. Asking about initial TTT?
a. Synchronized DC shock
b. IV amiodarone
c. IV adenosine
d. IV NaHCO3

388
Pharma, poisoning, Patient safety

8. Patient will start lamotrigine what you advise them?


a. If skin rash seek urgent medical advice
b. If developed sore throat seek medical advice
c. If sedation developed gradually stop the medication

May 2021
1. A known case haemophilia A received factor VIII 3 times during this week
through subcutaneous implant catheter. He was febrile, tachypnoeic, and
tachycardic, with a BP of 100\70. He is on ceftriaxone, what will you add?

a. Flucloxacillin
b. Clindamycin
c. Vancomycin
d. Gentamycin
e. Benzyl penicillin

2. A girl with migraine presents with vomiting and dizziness and the GP
prescribed Metoclopramide and analgesia after which she had facial
grimace and twitching. What is the next course of action?

a. I.V Procyclidine
b. Benzodiazepine
c. Naloxone

3. A 15-year-old girl came to ER drowsy with mydriasis. The BP was 70/40


and HR 130/min , PH7:2 , PCO2: ,PO2: , HCO3: (Metabolic acidosis) ,
what is the cause?

a. Cocaine
b. Amitriptyline
c. Amphetamines
d. Ecstasy

389
Pharma, poisoning, Patient safety

4. What advice would you give to parents after diagnosing their child with
Rolandic seizures and prescribing Carbamazepine?
Choose TWO options.
a. If there is severe sore throat, contact the doctor immediately
b. Start it gradually to avoid sedation
c. Avoid NSAIDs
d. Give one dose daily
e. Check platelets
f. Do U&E
g. Check Carbamazepine level regularly
h. Stop Carbamazepine if he is seizure-free for 6 months

5. A 14-year-old patient with CP was on one day trip. When he returned


home, he was tired, and the mother gave him a medication for travel
sickness. There were hallucinations and the HR was 40, BP 110/54, RR:
normal. The child was on regular medication (Hyoscine, Levetiracetam,
Baclofen Diazepam and many other drugs) What is the cause of this
presentation?
a. Excess doe of Hyoscine batch
b. Missed dose of morning Levetiracetam
c. Cyclizine for motion sickness

6. A baby received 2 doses of Gentamicin during 12 hours instead of 36 hours


as a mistake by a nurse who documented the event in her notes and U&E
were requested. You ordered trough level (which was normal), and
documented the incident, what else will you do?
(THREE choices)
a. Call pharmacist
b. Inform family and apologize
c. Inform on-call consultant
d. Complete incident report
e. Complete child safety
f. Complete serious report
g. Perform hearing assessment
h. Inform the head nurse on call.
i. Stop gentamicin

390
Pharma, poisoning, Patient safety

Jan 2021
1. EMQ
A. Cyclosporin
B. Cyclophosphamide
C. Vincristine
D. Infliximab
E. Tacrolimus
A. Patient with haematuria and supra pubic tenderness
Cyclophosphamide
B. patient with hypertrichosis Cyclosporine

C. patient with Impaired OGTT Tacrolimus

2. Neonate was given gentamicin at 2hrs of age for sepsis. Weight was 2.9kg,
The nurse calculated the dose for 3.4kg. It’s 36hrs now and he’s due for
next dose, what is the immediate action,

a. Check U &Es
b. gentamicin level
c. skip dose ???
d. Make incident report
e. Inform the family and apologize
f. to them

Sept 2020
1. EMQ:
A. Carbamazepine
B. Ethosuximide
C. Sodium valproate (Wt. Gain)
D. Topiramate
E. Vigabatrin
F. Lamotrigine
G. Clozapine

391
Pharma, poisoning, Patient safety

A. Antiepileptic drug in certain ethnic group can cause Steven Johnson


Syndrome Carbamazepine
B. Specific antiepileptic only used for single type of seizure Ethosuximide
C. Antiepileptic for intractable epilepsy cause rapid weight loss Topiramate

2. 13 years old child develops sore throat 15 days before admission, then 10
days develops haematuria, 6 days he develops face swelling and headache,
the parents treated by paracetamol now develops tonic colonic
convulsions and GCS was 11. Respiratory rate 20 Blood pressure 180/110
mmHg, urine 3+ blood and 3+proteim he was afebrile.
A. What is the cause of convulsions?
f. Systemic HTN
g. Encephalitis
h. Cerebral vasculitis
i. Brain abscess
j. HSP
B. What treatment to give?
g. Labetalol
h. Phenytoin
i. Acyclovir and IV antibiotics
j. Mannitoll
k. Steroids
l. Intubation and ventilation

3. Child 9 year’s boy with bed wetting doctor prescribes oxybutynin,


what important side effect you should tell parents about?
a. Blurring of vision 100% Pharmacology
b. Diarrhea
c. Poor appetite
d. Bradycardia

392
Pharma, poisoning, Patient safety

4. Baby on vancomycin, trough level 22, take Vancomycin every 24 Hours N


(5-10) what to do?
Give the same dose every 36 hours

5. Child on Carbimazole for hyperthyroidism developed sore throat, no pus


on throat. Asking what next step?
a. Full blood count
b. Do drug level
c. Stop Carbimazole
d. Nasopharyngeal swab for viruses

Jan 2020
1. Known acute asthmatic attack on salbutamol came by lactic acidosis what
is the cause?
a. IV Salbutamol
b. Mg so4
c. Aminophylline
d. Hydrocortisone
e. Ipratropium bromide

2. Adolescent take paracetamol tablets in separate time, urea& creatinine


high INR 1.4, normal liver enzymes, what to worried about in next
72 hours?

a. Hypoglycemia
b. renal tubular necrosis no hepatic necrosis in the choice
c. Metabolic alkalosis
d. Hemorrhage
e. Hemolysis
f. Hypertensive encephalopathy

393
Pharma, poisoning, Patient safety

3. Patient admitted in PICU. what drug doesn’t infuse calcium with in this
antibiotic?
a. Cefotaxime.
b. cefuroxime.
c. Ceftriaxone
d. Cefepime
e. Ceftazidime

4. Drug not used in UAC


a. adrenaline
b. Dopamine

5. Case of burn with low platelet 80,000 which drug contraindicated in this
case?
a. Brufen
b. Paracetamol

6. EMQ cases given Ceftriaxone what to add?


A. Infant has Chicken box, develop erythema, fever, CRP more than 6,
hypotensive 70/54 benzyl Penicillin & floxacillin??
B. Case abdominal distension, appendectomy, on gentamycin, cefuroxime
Metronidazole
C. Knee replacement central catheter Vancomycin??

7. ADHD treat by methylphenidate, weight loss, controlled


a. Shift to atomoxetine
b. Continue, review take measurement after 3 months
c. Stop methylphenidate
d. Refer to dietician

394
Pharma, poisoning, Patient safety

8. Epileptic boy improved on valproate, school performance improved, but


aggressive at home What to do?
a. Stop valproate
b. substitution by carbamazepine withdrawal sodium valproate gradually
c. Check sodium valproate level
d. Continue and tell parent this is known side effect and is part of
disease
e. Change to carbamazepine

9. EMQ toxicology
A. Boy shallow breathing, tachycardia, pin point: methadone
B. Teenage girl fever 39 C, hypertension, Ecstasy
C. Boy mydriasis, wide QRS: Tricyclic antidepressant

10. Adolescent boy with different type of seizures treat


Valoprate

11. A case of anorexia nervosa BMI 15kg, started NG feeding after 24 hrs.
investigations show po4 very low (0,4), hyponatremic, hypotensive,
hypokalemic 3.2, what imp management?
a. IV phosphate
b. Potassium
c. Refer to ditetion
d. IV normal saline bolus
e. NGT

395
Pharma, poisoning, Patient safety

12. 15 years old with shortness of breath, fever and cough previously
diagnosed as SLE on steroids, enalapril and mycophenolate mofitl, poor
compliance to treatment She has mild learning problems.Now: pale,
alert, fever, tachypnea, tachycardia, bilateral lower zone crepitations,
decrease air entry bilaterally. Bp 160/100, with high renal
function, good urine output, creatinine 555, thrombocytopenia
K 3.2mmol, low Na, metabolic acidosis, bicarb 10, BE -6, hepatomegaly
6 cm, bilateral pedal odema

What is the most important next two actions?


a. increase dose of enalapril.
b. mechanical Ventilation
C. Oral nifedipine
d. iv furosemide.
e. platelet transfusion
F. Hemodialysis
g-Na bicarbonate iv
h. iv nitroglyceride
I. Iv K

396
Pharma, poisoning, Patient safety

13. Case of hypernatremia and low osmolarity in urine, urine output high,
Holoprosencephaly
A. Diagnosis?
Central Diabetes insipidus
B. What to do?
Desmopressin
14. one case cystic fibrosis receive correction for Na 135 and k 2.5
Discharge came again after 5-days with low Na 134 and k 2.9
what to give?
a. oral Na and k
b. FLUID RESRTCTION
c. ORAL K
d. Desmopressin

15. Asthmatic child came with sever attack, received salbutamol, hydrocort
iv, but still tachypneic ,tachycardic with metabolic Acidosis PH.7.25 ..
RBs 22 , Lactate 7 mmol.... AIR ENTARY is good
WHAT IS THE CAUSE OF HIS BREATHLESSNESS??
a. salbutamol toxicity
b. DKA

16. neonate preterm with mottling and low bp, bradycardic vent map 38 ...
co2 normal ....po2 low fio2 90%... with good air entry (pphn)
deteriorated after 4 hrs, how to treat?
a. nitric oxide
b. prostaglandin

397
Pharma, poisoning, Patient safety

Sept 2019
1. An epileptic child on carbamazepine develop respiratory
illness given erythromycin present with symptom like
drowsiness
a. check carbamazepine level
b. change antiepileptic
c. stop carbamazepine

2. baby with haemangioma on propranolol ask when to stop


a. striders
b. SVT
c. Diarrhea
d. loss of appetite

3. side effect of methylphenidate


a. anxiety disorder
b. seizure
c. hypotension
d. bradycardia
e. weight gain

4. case of girl with visual and auditory hallucination, came with


grandmother, she was well accept she has irritable bowel syndrome and
given antihistamine and hyoscine for fever(not remember),
what is cause?
a. drug interaction
b. drug abuse
c. conversion reaction

5. Turner syndrome on growth hormone came with symptom of


IIH LP done pressure high, normal MRI, what to do?
a. discontinues growth hormone
b. LP

398
Pharma, poisoning, Patient safety

6. female with myoclonic seizure, ask about drug of choice


a. Leviteracitam
b. Lamotrigen
c. NA valproate
d. Topiramate

7. question about obese (bmi 31) child type 2 DM high HBAIC 58 started
on life style no improvement what to do?
a. Metformin
b. Sulfonylurea
c. Insulin

8. patient Pakistani with absent epilepsy Control ON NA-valporate


presented with rt side knee injury came with ca 2.2 ph 0.9 ALP high
mild anemia mcv 76 what to do
a. continue Na valporate and give vit D
b. continue Na valporate and give folic acid
c. Change to ethiosuxamide

9. EMQ
A. neonate 3wk uti symptom==IV CEFOTAXIME
B. Cough and lymphocytosis= clarithromycin
C. fever and lion pain ==co amoxivcalv

10. pt with hypertension you need to start ACE Inhibitor, what advice to
give to parent?
a. urea and creatine after 2day
b. LFT before treatment

11. patient in breast feeding for 8-month not remember, came with ca 1.9
twisting of face
vit D

399
Pharma, poisoning, Patient safety

May 2019
1. CP patient on keppra , hyoscine & baclofen, mother give another ttt for
motion sickness then he develop halucination and midriasis
what is the cause?
a. hyosin bromide
b. cyclizine
c. metoclopramide

2. girl went to GP came with facial grimace after She get medication for
vomiting what is your ttt?
a. IV PROCYCLIDIN
b. buccal midazolam
c. diazepam

3. Asthmatic child came with sever attack, received salbutamol,


hydrocort iv, but still tachypneic ,tachycardic with metabolic Acidosis
PH.7.25 ..RBs 22 , Lactate 7 mmol.... AIR ENTARY is good
WHAT IS THE CAUSE OF HIS BREATHLESSNESS??
a. salbutamol toxicity
b. DKA

4. gentamycin given 1st dose given mistakenly with weight 3.5kg and
true weight 2kg
a. skip next dose
b. urea and creatinine first
c. hearing assessment
d. trough level

5. sever nephrotic scenario with frequent relapse given steroid and still not
improved, next step?
a. Tacrolimus
b. Infliximab
c. Cyclophosamide

400
Pharma, poisoning, Patient safety

6. EMQ:
cyclosporin
vencristin
infliximab
tacrolimus
steroid

A. patient with foot drop vencristin


B. nephrotic patient with hirsutism cyclosporine
C. Chron’s disease with medicine cause temperature 40c infliximab

7. patient has description of ( ronaldic epi.) causing disturbance to her


parents want to commence treatment. u decided to give carbamazepine
what instruction u will say to them (2options)

a. need regular check level


b. don’t give with ibuprofen
c. should be given once per day
d. if sore throat, report to your doctor immediately
e. should be started gradually to prevent sedation side effect
f. if free from convulsion for 6 months should be stop the gradually

8. teenager girl went to party brought unresponsive with severe metabolic


acidosis ph 7.2 ,hco3 very low and dilated pupil and bradycardic
a. amitriptyline
b. aspirin
c. cocaine

401
Pharma, poisoning, Patient safety

9. 15-years female myoclonic seizure on levetiracetam not controlled on oral


contraceptivee, with which drug we can replace?
a. Lamotrigine
b. sodium valproate
c. Clonazepam

10. 12-yrs boy with (senario of myoclonic epilepsy) clumsy in the morning
drug of choice?
a. Na valbroate
b. Lamotrigine

11. picture then Q About ttt


floxacillin IV

12. history of eczema not responding


A. asked about diagnosis
c. Chickenpox
d. eczema herpeticum
B. management
systemic antiviral

402
Pharma, poisoning, Patient safety

Jan 2019
1. oxybutynin side effects to tell the parents:
a. Blurred vision
b. Diarrhoea
c. abdominal pain

2. A picture of a swollen red periorbital region, for 24 hours, ttt?


a. IV ceftriaxone+ flucloxacillin
b. oral co-amoxiclav
c. social referral
d. chloramphenicol

3. drug interactions (2 Answers)


a. Ciprofloxacin & gentamicin
b. azithromycin & ketoconazole
c. erythromycin & cyclosporin
d. ibuprofen & carbamazepine

4. long term S/E of prednisolone:


a. behavioural changes
b. hair loss
c. hypoglycaemia
d. Weight loss

5. child with his grandmother brought with pinpoint pupil, investigation??


urine toxicology

403
Pharma, poisoning, Patient safety

6. patient with HF on spironolactone, before giving captopril ( 3 Answers)?


a. measure BP in first hour and whole first day
b. measure k
c. measure ca
d. measure Mg
e. measure blood glucose
f. measure renal function.
g. protein urea

7. A child with acute severe asthma, took salbutamol nebuliser and IV, IV
hydrocortisone, mg sulphate has hypoxia, metabolic acidosis, ↓ k , ↑
lactate 6 ,air entry is ok what is the best action?
a. CXR
b. intubate and ventilate
c. bolus if Mg sulphate
d. Taper and wean off salbutamol
e. Aminophylline
f. correct potassium

Sept 2018
1. 15-yr old girl, has boyfriend, juvenile myoclonic seizers insist to take
valproate after you told her teratogenic side effects
what’s 2 management?
a. check vit d
b. FBC monthly
c. high dose folic
d. make sure she take UpToDate contraceptives
e. EEG
f. liver enzymes

404
Pharma, poisoning, Patient safety

2. 10 days preterm, vancomycin 15mg/kg/24 hr , trough level 22 ( N = 5 – 10)


normal kidney function what to do ?
a. decrease dose by 20%
b. same dose every 36 hrs
c. stop dose and recheck trough level

3. EMQS
a. carboplatin
b. etopside
c. infliximab
d. rutixmab
e. ciclosporins
f. cyclophosamide
g. vincristine
h. vinblastine
i. tacrolmus

A. 8-yrs-old with psoriasis on ttt with hypertrichosis-- ciclosporins


B. 4-yrs with neuroblastoma, frank hematuria, subrapubic tenderness
cyclophosphamide
C. After renal transplant, on immunosuppressive, impaired OGTT Tacrolimus

4. EMQS
A. 15-yrs-old boy with friends, agitated, confused, hyperpyrexia, temp=41,
bp=140/95, dilated pupil-- ectasy
B. 18 month , decrease level of consciousness ,aperxia, low bp , shallow
breath , pin point pupil--- methadone
C. 15-yrs-old agitated ,confused, bp= 90/50,dilated pupil , ECG broad complex
QRS-- TCA

405
Pharma, poisoning, Patient safety

May 2018
1. 13-years-old with thyrotoxicosis has congested throat on carbemazole
treatment
a. EBV serology
b. CBC and differential *>>SE of carbemazole is agranulocytosis

2. 14-years-old girl with sever asthma given treatment and now has lactic
acidosis, and tachycardia, what is the reason for metabolic acidosis?
a. iv aminophylline
b. IV salbutamol
c. Hydrocortisone
d. Magnesium sulphate
e. Ipratropium

3. burn which drug to avoid, high liver enzymes, bleeding disorder?


a. Ibuprofen
b. Paracetamol
c. Morphine

4. nurse give vancomycin 5-fold more they asked what to do?


trough level before next dose

Jan 2018
1. 15-yrs had absence attack and 2 tonic colonic seizures what to give?
e. sodium valoproate
f. carbimazipines
g. topramate
h. lamotirogen

2. picture of baby developed rash genitalia and mouth had red watery eyes
a. Carbamazepine
b. Sulphonamides

406
Pharma, poisoning, Patient safety

3. Asian boy on carbamazepine and erythromycin developed unsteady gait


what to do?
a. EEG
b. Carbamazepine level
c. MRI brain
d. Toxicology

4. child on sodium valproates and is having controlled seizures but with


behavioural changes what to say for parents?
a. behavioural changes are common and they need to continue on
sodium valproates
b. stop treatment immediately
c. replace with carbimazipine

5. Causative agent:
Phenytoin

6. A boy with Single kidney after removal of the other dystrophic kidney ..
presented with hypertension ..prescribed an ACEI ... what is the most
important recommendation to give?
urea and creat after 2 days
No options for first dose hypotension

Sept 2017
1. on sodium valproate with sign of biochemical rickets?
c. change AED
d. add vit D and ca

407
Pharma, poisoning, Patient safety

2. EMQ: antiepileptics
A. if mouth ulcer should stop➔ Carbamazepine

B. 2nd line in absence seizure➔ lamotrigine

3. Thrombocytopenia mother on carbimazole


drug induce thrombocytopenia

4. infantile spasm Tuberous sclerosis- medicine used?


Vigabatrin

5. parents education-?
c. buccal midazolam is better than rectal diazepam
d. can have resp depression

May 2017
1.child known epilepsy on Na-valproate has investigation showing vit-D
deficiency what to do?
c. stop the drug
d. continue drug and give vit D

2. infant 2-month on ACE drugs asking for 2 side effects :


a. Renal
b. first dose hypotension

3. PROLONG QT TTT???
propranolol

4. asthmatic child has episodes failed to controlled with salbutamol he is


hyperventilating with high sugar was 18 mmol and ABG OF metabolic
acidosis???
Salbutamol toxicity

408
Pharma, poisoning, Patient safety

5. easy question of -side effects of steroids


6. question for drug combination contraindication need 2
a. ciclosporin and erythromycin
b. azithromycin and ketoconazol

Jan 2017
1. Emq about poisoning
A. pin point pupil --- methadone
B. hyperthermia, Agitated ---- ectasy
C. I went for tricyclic antidepressants. . Others went for heroin

2. case of vancomycin sepsis. .It's given every 12 hours Trough level


increased to 4.5 times.What to d?
a. increase intervals between doses
b. decrease dose by 20%
c. increasing interval to 36 hours

3. oxybutynin side effect


blurring of vision

4. EMQ About chemotherapy


A. Bone fracture >>predinsolone
B. Hypertrichosis>>Cyclosporin
C. Pruritis>>methotrexate

409
Pharma, poisoning, Patient safety

5. question of poisoning . . Mom had regular ttt. . History included GIT


symptoms (vomiting & diarrhea )

A. diagnosis -- lead poisoning


B. ttt---Dysferoxamine

6. absolute contraindications of brufen . . Asks for 2 options?


a. NEC
b. thrombocytopenia
c. Renal insufficiency

7. a questions about carbamazepine. . Choose 2 right answers.?


a. stop it if developed sore ulcers
b. monitor its level

May 2016
1. A case of 1- year girl has absence epilepsy what is treatment?
ethosuximide 1st line or lamotrigine ELS

2. Infant with TS has infantile spasm ask about treatment?


vigabatrin

Infentile spasm as general

First line treatment.➡prednisone /tetracosactide(ACTH) and vigabatrin

Second line treatment ➡Na valporate and nitrazepam

Third line treatment ➡vit B6 pyridoxine

410
Pharma, poisoning, Patient safety

Jan 2016
1. Pt with burn for dressing in need for analgesia with prolong bleeding
profile which of the following contraindicate:
a. inhaled NO
b. Paracetamol
c. Pethidine
d. Morphine
e. Ibuprofen

2. Pt on phenytoin traumatized his knee joint ca and ph normal only high ALP
ask what management:
a. Vit D supplement
b. Iso enzyme level

3. CP pt. black stool , low platele Dantroline --- SE of drug


4. Pt prescribed enalapril (ACE inhibitor) what advice to give parent:
a. Check urine in fortnight
b. Check creatinine clearance two days later

May 2015
1. 7 YEAR OLD WITH ABSENCE SEIZURES..Rx..?
a. Valproate
b. lamotrigine

2. a child with aspirin overdose. Asked about management?


Sept 2014
1. Pic of Gum:
phenytoin

411
Pharma, poisoning, Patient safety

2. EMQ: Antiepileptic drugs:


A. Don’t use in absence seizure Carbamazibine
B. Cause oral ulcers Carbamazibine
C. only medication used in infantile spasm with tuberous sclerosis
Vigabatrine

3. on sodium valproate with sign of biochemical rickets?


a. Change AED
b. Add vit D

4.Baby with seizure disorders, develop convulsion given first dose buccal
midazolam but not aborted, what advise u give to parents--‐?
(2 answer needed)
a. buccal midazolam is beter than rectal diazepam
b. midazolam can cause resp depression
c. parents can give 2nd dose of buccal midozolam.
d. Rectal diazepam can given by ambulance team.

Jan 2013
1. 3-years old girl came with her foster mother generalized tonic clonic
convulsions. Or vacant episodes, ttt?
a. sodium valproate??
b. topiramate
c. carbamazepine
d. no treatment
e. lamotrigine ??

2. ACE inhibitors side effects:


a. first dose hypotension
b. acute renal failure

412
Ethics

413
Ethics

Sept 2021
1. 23 wks +1 day of gestation. Mother came with rupture of membranes
who should decide for resuscitation and stabilization of baby?
a. Parents
b. NICU consultant
c. Mid wife
d. Obs doctor

2. An 8-year-old boy was in a school trip together with his mother. They
had an RTA and he sustained an injury and the mother was intubated
and admitted to the ICU.the boy came with the teacher, and urgent CT
brain requested. There was a gentleman talking to teacher who
confirmed that he was the biological father of the child who was married
to the mother and was divorced since 5 years, but his name was in the
birth certificate. Who is responsible to give the consent for CT?
a. Biological father
b. Teacher
c. Emergency consultant
d. Court
3. Scenario of life-threatening asthma worsening and going for intubation
by anaesthesia. You are the on call doctor and you called the consultant.
How to present the case?

414
Ethics

4. scenario regarding mother concern of mercury poisoning for her child, as


he use teeth filling, she went to GP who requested mercury level, which
was not available at NHS, mother heard that it can be done in one of the
private labs, mother came to phlebotomy nurse with the request, and
nurse ask you what to do?
a. Accept mother request
b. Discuss with the child GP the case??
c. Inform her that hair examination instead of serum level
d. Ask for father consent
e. Refer to child protection

May 2021
1. A 13-year-old with severe anorexia nervosa (BMI 15, PO4 0.8, body temp
36.1C’), came with her stepfather. The consultant planned to start
emergency treatment with NGT feeding and the stepfather agreed but she
refused. The mother had a suicidal attempt. Who will give the consent
a. The consultant paediatrician can proceed without the need for
further consent as it is the best interest for the child
b. Court order
c. The stepfather
d. Legal consent from social service
e. Respect her wish
f. Psychiatry to assess competence
2. An 8-year-old boy was in a school trip together with his mother. They had
an RTA and he sustained an injury and the mother was intubated and
admitted to the ICU. The boy came with the teacher, and he required an
urgent CT brain. There was a gentleman talking to the teacher who
confirmed that he was the biological father of the child who was married
to the mother for 2 years, but his name was not in the birth certificate as
the boy was named after his mother’s family name. Who is the person
responsible to give the consent for CT?
a. Emergency consultant
b. The father
c. The teacher
d. Court order

415
Ethics

Jan 2021
1. Neonate 24 wk gestation with bilateral renal agenesis and pulmonary
hypoplasia the medical team decided not to resuscitate but the parent
wants to resuscitate who would have the final decision.

a. Parents
b. Neonatal consultant
c. Ethics committee
d. Court order

2. The police caught the 16years old girl with a drug dealer and suspects she
is hiding drugs in vagina.She is understanding.Which of the following will
be (upheld) within her right?
a. she can go to bathroom unsupervised
b. Strip search
c. Strip search by police
d. Request adult be in examination
e. she can refuse to call social services

3. 8 years girl with suspected NAI She was bought in by stepmom, police and
duty social worker (Father now in police custody for suspected abuse),
who should Consent for examining her?
a. Police
b. social worker
c. child
d. father

4. Girl 15years old has acne and very concerned about it, she took treatment
but not improved, she wants to start retinol, parents refuse, she is
competent and aware about treatment and its side effect. From whom you
should take consent?
a. Parents
b. Girl

416
Ethics

Sept 2020
1. Girl was with her grandmother and teacher and fell in the garden with
hematoma in the head, mother is traveling and divorced, came at the ER
unwell, need urgent MRI scan under general anaesthesia, who will give
the consent?
a. Emergency consultant
b. Grandmother
c. Teacher

2. A14 Years old girl with ALL came with fever sweating and heart murmur
tachycardia gallop can’t complete a sentence. Hb 3.3 platelet 79.000 WBC
6.1 parents refused blood transfusion due to religious beliefs and also the
girl refused to have blood transfusion, you discussed with your consultant.
What is the next action?
a. Take court order
b. Seek emergency protection order
c. Transfuse as necessity
d. Refer to local social service
e. Don’t transfuse and follow family wishes

Jan 2020
1. 15-year girl of anorexia nervosa, hypotensive, heart rate 40,deteriorated
refuse treatment by nasogastric tube, what is the appropriate action?
a. parent ??
b. police
c. no thing
d. proceed treatment as this best child best interest ??

417
Ethics

2. mother febrile, GBS positive, have history of child died from sepsis, PROM
36hr, baby now well, mother refused antibiotic for previous child and this
child what to do?
a. Discharge and ask her to come back if there is symptoms
b. Discharge on oral antibiotic
c. Give antibiotic despite her refusal as this for child best interest
d. Call social services to get court order

3. 16-years boy with Cystic Fibrosis, lung function not good, not collect
prescription will be referred to follow in adult clinic What will you do?
a. Send letter to GP to highlight his recent lung function
b. arrange meeting, educate, involve nurse; arrange to transfer to Adult
clinic.
c. asks his mother to collect medication
d. Refer to adult, give next appointment
e. Make adult and child clinic together
f. Continue in pediatric clinic

4. A neonate diagnosed with hypoplastic left heart syndrome, parents are


discussing treatment options and still did not decide, baby desaturated,
best action?
a. Give prostaglandin without consent
b. Wait for the parents to decide
c. Transfer to cardiac centre
d. Put patient on oxygen
e. Give sildenafil until parents decide

418
Ethics

5. CP child presented with respiratory infection, after discussion and


agreement with the family to withdraw life sustaining measures and
supporting with only analgesics, apart from documenting the agreements
what to do?
a. Call hospital chaplain
b. Allow relatives to visit
c. Give muscle relaxants to ease gasping
d. Brain stem test first
e. Discuss with Hospital ethics committee

Sept 2019
1. consent for surgery needed and adopted father is there and teacher
know the father ask who will give consent
a. adopted father
b. teacher
c. consultant
d. court
e. do without consent
2. paracetamol over dose paracetamol level normal what to do?
a. psychiatry assessment
b. discharge home

3. question about
SBAR how to
present to
consultant

419
Ethics

4. SMA boy 7-month came with resp arrest ,,you are on hospital with out
picu ,,ressusitation of baby sussful and now he is intubated and
ventilated ,,in last week he become increasely week with frequent chest
infection,, has previous baby died with same condition, ,they aware
about diagnose but end of life plan was not discussed in previous visit,
,parent seem to be not ready for discusition now. what to do?
a. disconnect the baby from ventilator
b. call neurology specialist in tertiary hospital for their opinion
c. discuss with parent
d. do all the active management, and put in ventilator

5. 14yrs old parent refuse treatment , according to UK law what most


thing directing consent of child
a. competence of child
b. age of child

May 2019
1. CP patient presented with 4 episodes of aspiration pneumonia for the last
3 months. some problem in his gastrostomy tube un save to give
through it....and should be change to jejunostomy feeds. Parents
refuse invasive procedure or resuscitation what to do?
a. court order
b. do the parent wish
c. trial of medical treatment first
d. convince parents
e. give through gastrostomy

2. road traffic accident mother intubated child have head bruises born 2013
father came his name not in birth certificate and teacher knows him... who
will consent for anathesia emergency CT of the child?
a. biological father
b. emergency consultant
c. court
d. teacher

420
Ethics

3. gentamicin wrong dose 1st dose and second in 36 hrs level checked
normal 2 options ..
a. incident form
b. inform family
c. inform consultant
d. inform head nurse

4. 14 YRS old female competent, with 15YRS old male partner with
consensual unprotected sex asking for emergency contraceptive, you
tried to convince her to tell her family
a. refer to gyna and genitourinary
b. refer to social
c. give contraception
d. tell parents
e. inform STI team
f. to do investigation of STD

Jan 2019
1. Neonate on chemotherapy HB 4,9 severe anaemia, took 20 ml saline and
still hypotensive, needs blood, parents Jehovah's witnesses, what is the
best action: (2 options)
a. Senior consultation for second opinion
b. cross matched O-ve & emergency blood transfusion
c. cross matched packed RBCs urgent
d. social services make emergency protection
e. see and wait
f. give erythropoietin

421
Ethics

2. 28-weeks baby needs porcine surfactant, what to tell her Muslims'


parents who refused to take it:
a. tell them it is in-the child's best interest
b. call the hospital's imam to tell them that it is authorised and they can
give it to their baby??
c. offer recombinant surfactant ??
d. tell them it won't be absorbed
e. don't give the surfactant

3. A teenager wants to remove a keloid scar that is annoying her, foster


mother refusing and she retains connection with her biological mother,
who can give consent:
Birth mother

Sept 2018
1. New-born hypoplastic left heart syndrome, while waiting for parents
decision to do surgery or not , baby deteriorate, what to do?
a. give prostaglandins without parent consent
b. refer to cardiac centre

2. Newly diagnosed leukaemia, HB =3, very sick cant complete one sentence
he and his parents refuse blood transfusion due to religious believe,
u consult with consultant, what to do ?
a. transfuse
b. court order

422
Ethics

May 2018
1. patient came from the farm with chest x- ray with (strep pneumonia) but
the parent refuse to treatment and the child was become sever sick

A. asked about organism?


e. Legionella
f. Streptococcus pneumonia
g. Mycoplasma
h. Viral pneumonia

B. The parent refuse antibiotic and the patient desaturated what to do?
Court order???
If there is option of proceed without consent will be better

2. a child with Sickle Cell Anemia in care of maternal aunt from local
authority, the child need for blood transfusion in next 24h
who will give consent?
a. biological parent
b. Court
c. Aunt
d. Social worker
e. Non need for consent for this procedure

3. 5-years newly asthmatic with sever respiratory distress, with acidosis


admitted to the word the consultant doesn’t know about the patient,
GP call to onsultant to discuss a case of asthmatic
RR= 60 , spo2= 90% HR=150/m
A big answer (how to handle pt to the consultant

423
Ethics

4. 16-years with paracetamol over dose, admitted and stated


N acetylcystin and INR is 8 ,he refuse to continue treatment and scape
from the word ,and the securities found him in the barking and brought
back .asked what to do ?
a. Start ttt as paretnt agree -- 16 yrs cannot refuse ttt in UK

424
Ethics

Jan 2018
1. EMQ:
A. Girl 15-yrs with severe anorexia nervosa refusing ttt
Treat against her wish
B. 2.8 yrs child need urgent CT after trauma brought by grand mother
parents are not answering Phones
DR as doing best interest for the PT
C. 15 yrs with acne no improvement on ttt asking for another ttt which has
SE despite explanation she understands the consequences of this ttt
She can consent

Sept 2017
1. colleague taking 5 tablets of benzodiazepine
a. inform the senior
b. Discuss with your colleague first

May 2017
1. mother has history of child died with sepsis and she had fever and now
has newborn refuse to give antibiotics?
give antibiotics despite her refuse

2. child fall from balcony in party and died what to do??


Father cannot refuse post mortal investigations

3. Child who have urticaria (or rhinitis not sure ) and pharmacist gave
chlorpheniramine … asking?
a. should take it
b. change to other type of antihistamine in options

425
Ethics

Jan 2017
1. a child with blood haemoglobin about 5.. Deteriorated ..parents jovarah
refuse blood transfusion what to do.Asks for 2 options?
a. Call senior colleague
b. transfer O negative blood
c. accept parents wish and don't transfer

2. a long case i don't remember.. Mother was drug addict i think.. asks
about oxygen therapy I guess to put the baby on cpap(not sure)

Jan 2016
1. X ray of 11 y boy with cough and fever with O2 sat 92 % show Rt
sided pleural effusion and consolidation doctor decided to give IV
antibiotic but parent refuse and believe on homeopathic ttt and tell
they will take him home
What is your action:
a. Let child consent for ttt
b. Call social worker
c. Inform pediatrician consultant ?

Sept 2014
1. College taking 5 tab of benzodiazepines?
inform your senior

if there is option talk to your college it will be the right answer

426
Ethics

Jan 2013
1. 22-years old mother of Muslim faith with premature baby born at 27
weeks of gestation, doctors advised for porcine surfactant. parents are
reluctant, what will u do?
a. liase with hospital imam
b. explain that the baby’s condition is such that the procaine
surfactant is in his best interest
c. explain that the UK Muslim community has approved its use
d. give another alternative surfactant
e. wait and keep on o2

2. Newborn with hypoplastic left heart, presented with cyanosis, on O2....


parents are thinking to give consent for surgery... in the mean time the
baby condition deteriorates what will u do?
a. transfer the baby to cardiothoracic unit.
b. give prostaglandin no need for parent’s consent
c. wait for parents’ decision
d. keep on o2

3. you went for your weekly shopping to the market and suddenly u hear a
mother shouting for help her 8 weeks old baby turns blue and apnoeic , by
the time u reach there he is well n pink n the paramedics has also arrived.
what will you do ?
a. hand over to the paramedic’s responsibility
b. insist that u will accompany the baby to hospital.
c. give an opd appointment
d. reassures the mother and the paramedic that there is no need for
hospital
4. 11-years old female, come with her foster mother, she has keloid, that
previously made surgery for it, but it returns back, and she insist to do
other surgery, her foster mother refuse, what you do?
a. Don’t do the surgery
b. take consent from the child ??

427
Safeguard

428
Safeguard

Sept 2021
1. 5 yrs. old child, which fracture suggests NAI?
a. Spiral fracture of humerus
b. Spiral fracture of femur
c. supracondylar fracture
d. Buckle fracture

2. When to call police?


a. 13 years female stole things from supermarket
b. Female with alcohol intoxication
c. Female taking cocaine
d. 8 years boy making sexual videos and share it on social media
e. 12 years old girl debulge to have sex relation with 14 y boy

3. 5 months old infant, presented with coraysal symptoms and cough, CXR
done and found to have healed posterior ribs fractures, born as
Premature 36 wks with difficult labor and resuscitation was performed,
what is the most like diagnosis??
a. NAI
b. Vit D deficiency

May 2021
1. A 14-year-old girl came to the outpatient department with her 23-year-
old boyfriend and at the end of the visit she asked you about
contraceptive pills. After arguing with her what will you do?

a. Persuade her to talk to her parents


b. Encourage them to go sex clinic
c. Refer them to social service
d. Give her the contraceptives pill as she will continue her relation
anyway

429
Safeguard

2. A 14-year-old girl came to ER with her father complaining of sore throat.


After examining her the doctor discovered bruises on the right arm and
shoulder. When asking her she states that there was an argument with her
father. She is a smoker, a drug abuser, an alcoholic and is sexually active
with a 16-year-old boyfriend and she is Gillick competent. What is your next
step?

a. After taking her permission, arrange a meeting with her boyfriend as


well as her parents to collect more information
b. Urgent referral to social service
c. Discharge her with a signpost about contraception, alcohol, and
smoking
d. Send her to sexually transmitted infection clinic

3. A 15-year-old Somalian girl whose mother does not speak English well, will
travel with a relative to her country and you have concerns she might be
subjected to FGM, but you feel that there is no immediate risk. What is
your appropriate action?
a. Urgent protection order
b. Urgent refer to social worker
c. Rise your concern to family and parents
d. Discuss with leader safeguarding doctor
e. Urgent refer to FGM team

Jan 2021
1. Case of Paracetamol ingestion with previous history of self-harm
refused treatment and left hospital after abusive manner with the
nurse?
a. call psychiatrist on call
b. Safe guarding
c. call police

430
Safeguard

2. Case of 10 months child lives with grandparent has suspected non


accidental injury with subdural hemorrhage, first step?

a. Skeletal Survey
b. Ophthalmology assessment
c. Call social

3. X ray of child with grandmother with history of trauma and had swelling in
the head

a. Left parietal fracture


b. Widened sutures
c. Frontal bone fracture
d. Right parietal fracture

4. A girl in foster care I think with low IQ with history of escape from home
and school. She was found once near club with alcohol intoxication. She
looks older than her peers, quiet, withdrawn with significant weight loss
and sleep disturbance. Diagnosis?
a. Depression??
b. Sexual abuse
c. Sexual exploitation??
d. drug mis use

431
Safeguard

Sept 2020
1. Picture of skin lesion what is the diagnosis?
Erythematous, healthy skin and bullae
(2 white raw areas and 2 small bullae)
‫الصورة كانت بيضاء وليست بنك كما يف‬
‫الصورة‬

a. Deep dermal burn


b. Pemphigus
c. EB
d. SSSS
e. Vitiligo

2. 1-year came for vaccination, during exposure of the arm you found this
lesion Photo on bite only
‫الصورة كانت باهته مش باين حجمها‬
What to do?
a. Refer social service 100 safe guarding
b. Call police
c. Reassure parents and discharge
d. Coagulation profile
e. Dermatology referral

3. When to call police?


a. 14 years female with genital mutilation done
b. 13 years female stole things from supermarket
c. Female with alcohol intoxication
d. Female taking cocaine
e. 8 years boy making sexual videos and share it on social media

432
Safeguard

4. mom came on Friday evening with her 8 months girl not moving her right
arm on X-ray there was spiral fracture of humorous, orthopaedic doctor
said he will manage fracture next
Monday at the clinic. What to do?
Choose two?

a. Admit
b. CT head
c. Skeletal survey
d. Refer to social service on duty.
e. Ophthalmology referral

5. A mother delivered at 28 weeks then discharged 39 weeks, mother has 2


other well children and she is busy, she then noticed her child crying
during nappy change, On exam; full anterior fontanel, pale, floppy and
Hg 5.5. What is the diagnosis?
a. Abusive head trauma
b. DDH
c. Septic arthritis
d. Meningitis

6. 3-months infant came with cough, runny nose and fever, on X ray: callous
formation on 2nd, 5th and 6th ribs. History of resuscitation and chest
compression at birth because mother had an accident and went on labor
What is the cause of finding on x ray?
a. NAI
b. Chest compression done during resuscitation

433
Safeguard

7. African Girl 18 months play with her brother 3 years old, her brother fell
on her ,mother came to the room found the boy fell down on his sister,
examination: bruises on the chest and x ray given, the girl took her
immunization on time ( rib fractures with callus) vs Rachitic rosary .
What are the next 2 steps?
a. Skeletal survey
b. Referral to social service
c. Ask if the child known to child protection register
d. Coagulation profile
e. Vit D
f. Bone Profile
g. CT brain

Jan 2020
1. Which of these you need to call the police?
a. 14 yr taking cocaine
b. 10 yr come with alcohol intoxication
c. 13 years girl stole the store
d. 8 years makes sexual video in front of others
e. 12 years old girl debulge have sex relation with 14y boy

2. picture of back of knee lesion, elder sister has skin infection previous week

a. cigarette burn
b. Impetigo

434
Safeguard

3. 2-years child teacher noticed he has bruises in both leg has ankle swelling,
informed social service and child can explain live in foster care.
Investigation platelet ls 131,000
a. NAI
b. HSP
c. ITP

4. Picture of child less than one year, rib fracture chest x-ray, recurrent chest
infection 3 options
a. Skeletal survey
b. CT brain
c. Bone profile (ca – mg- vit d)
d. Sweet test

5. Mother get out & left her 9-weeks baby with his father, when she return,
she found him on his cot not respond, HC 42 cm, HB 6 , fibrin 5 ,acidosis
A. What further investigation?
a. CT head
b. Abd U/S
c. skull x ray
d. head uls
e. CR
B. Diagnosis?
a. Meningitis
b. late hemorrhagic disease of new born
c. in born error
d. NAI Head injury

6. 12-year girl has 14 yr old boyfriend want contraceptive, next step?


a. call police
b. call parent
c. call her GP
d. prescribe contraceptive

435
Safeguard

Sept 2019
1. which fracture will indicate NAI in 5yrs old?
a. spiral fracture of femur
b. spiral fracture of humours
c. supracondylar fracture
d. buccal Fracture

2. patient with forceps delivery receive vit k oral according to parent wish
and came with subdural hematomas increase head circumference ,, on
bottel feed at age of 5-weeks ,,lethargy bruises in right upper limp
a. NAI
b. vit k def

3. baby with poor feeding and irritability bilateral subdural


hematoma (may be same question of that with oral wit k????)
a. glutaric acid urea
b. NAI

May 2019
1. 13 years girl from Somalia you have doubt she will go to Somalia with
relative in next holiday and there is no immediate risk no
what is your action?
a. call police
b. court order
c. inform doctor of safe guarding in hospital
d. discuss with parents
e. talk to FGM team
f. discuss with social service in local area

436
Safeguard

2. Safe guarding with hold information from parent /carer safeguarding and
multidiscipling
a. 9 year whose her uncle touch her (abnormal touch)
b. 9-month in care with fracture femur with mother boy friend
c. age 3-weeks with circular bruise in check
d. age one month with circular burn in buttock
e. young girl with haematuria when go to hospital will be negative

Jan 2019
1. CXR, Accident for mother at 36 week gestation, delivered in poor condition
needed bag and mask resuscitation, now 3 months came with crying, fever
CXR: callus at 2nd . 5th & 6th left ribs:
a. NAI
b. Due to resuscitation
c. ostiogenesis imperfecta
d. osteopenia

An oblique chest image in a 4-


month-old demonstrates acute (red
arrows) and healing (black arrow)
rib fractures concerning for abusive
injury. Recent rib fractures do not
show callus formation for almost
two weeks, so the lack of callus on
two fractures suggests that this child
has fractures of two different ages.
Rib fractures in an infant are
uncommon except in cases of abuse.

437
Safeguard

2. A picture with a history of 2 days irritability, no fever, came with


grandmother?
a. Mastoiditis
b. NAI
c. BATTLE SIGN
d. INFECTED LN

3. A teenager took 10 tabs of paracetamol, level at 4 hours below ttt line,


patient have suicide attempt before ,wants to go home , became abusive
to nurses and leaved hospital you informed your consultant appropriate
action?
a. Police
b. call parents
c. inform safeguarding
d. psychiatric on call

4. A child is on child protection plan, presents to GP surgery with mild


URTI:
a. health visitor in 48 h for follow up
b. discharge and no further action
c. admit and inform social services
d. discharge and inform social services

5. (Picture) A human bite on arm,


next step?
a. dermatology referral
b. social services

438
Safeguard

Sept 2018
1. 3-yrs-old unwitnessed fall, buckle fracture radius in x ray, her parents were
in wedding, the girl say clearly (I fall from table on my hand) the fracture
was manged adequate, what to do?
a. Nothing
b. sk survy & vit d & fBC & coagulation
c. Sk survey , x ray other arm to compare

2. 10-months old Baby fall from mother lap , stable ,no vomit , 6 cm
hematoma , next step ?
a. CT brain in 1 hr
b. no thing charge after observation
c. CT with in 4 hrs

3. 14-years-old came with her 23-year-old boyfriend, want to have


contraception?
a. prescribe for her as she will continue her relationship any way
b. make sure to educate them to follow and keep follow with sex health
clinic
c. tells her mom about her
d. tells police
e. urgent referral to social service??

May 2018
1. 3-months-old with head circumference more than 98% and weight below
0.2 centile, has excoriation in the buttock, then the child have 2 other
sibling is ok, chest x ray of healed post rib fractures

a. social service
b. CBC
c. Blood film

439
Safeguard

2. African family, 15-months-old with small bruise on chest and swelling in


leg, his brother fall on her I think they asked 2 best approach
(there is x- ray chest)
a. skeletal survey
b. vitamin D

Jan 2018
1. 4-months-old with subdural haematoma what to do next?
a. MRI
b. skeletal survey
c. ophthalmology
d. clotting screening

2. 15-year-old will travel to Somalia with guy and you had concern she could
be subjected to rape, what to do?
a. discuss with parents your concerns
b. refer to social
c. police order
d. protection order
e. Discuss with colleagues

3. 18-months had swelling over left arm different bruises at different ages
over his body had anterior chest bruises as well x ray confirmed spiral
fracture left arm what should you tell his mom?
a. you are concerned about different bruises and ages in her body
b. you will admit for further x rays and you need to inform social worker
c. mom need to stay until social worker come
d. inform police
e. keep baby until coagulation profile of the baby is known.
f. mother should stay for further labs and x rays as you are suspecting
disease in her baby

440
Safeguard

4. 3 months old boy, NICU admission + ventilation, currently BPD on home


O2 lab show vit d def, xray shows post rib fractures (not ricketic rosaries)
next action?
child protection register ?

5. Mother went out in the evening leaving the baby alone with his father
returned at 2:00 am finding the baby in his cot to be very pale , gasping ,
can't breathe although he was previously very well .. lab findings
didn't direct towards metabolic / IEM cause ... examination I think showed
head circumference 95th centile ... what is best investigations?
CT scan of the head

6. Case presented with subdural hge ... investigation?


a. skeletal survey
b. ophthalmology refer.

7. 4-yrs with increasing thirst admitted for drowsiness and behavioural


changes with foster parents GCS 10 had laboured breathing and
nystagmus BP 120/75, Na 165, K 3.8, Cl 120, bicarb 18, glucose 6.7, urine
osmolality 850 what is your diagnosis?
a. RTA
b. Nephrogenic DI
c. Psychogenic
d. salt poisoning
e. primary hyperaldosteronism
f. DM type 1
g. SIADH

441
Safeguard

Sept 2017
1. 17-year boy fell down and died inspite of resuscitation-full investigation
should be ordered, doc on duty to give death certificate, father can refuse
post mortem.?
No they cannot refuse and full investigation should be ordered

2. post rib fracture and < 2nd centile(neglect)


NAI

3. Human bite photo


Inform social services & give amoxiclav if within 36 hours

4. Hypernatremia with high urine osmolarity 800 with foster parents-?


salt poisoning

5. 1.5-yr girl with grandmother, anal patulous, constipation, faecal


impaction, wart-?
child abuse

6. cigarette burn photo

442
Safeguard

May 2017
1. child with many siblings has bruises in his face (maxillary?) past medical
history of injury admission maybe non accidental injury
skeletal survey and social serves
[ ACCORDING TO THE SCINARIO]

Jan 2017
1. human bite picture, Asks about next action?

a. Admission
b. discuss with social service
c. coagulation

2. 3-year old child with fracture at a wedding . . He admitted falling. . Got


full treatment . . What investigation to do?

None accidental fracture

May 2016
1. Neonate 4 kg referred by social worker on examination normal child
request X-ray and blood profile and mother refuse and say investigation
will be normal what to do?
a. refer to social service
b. call police and get consent by lawyer
c. investigate and no need for consent because it best for child

443
Safeguard

May 2015
1. picture of a child after a seizure. Delayed developmental mile stones.
with skin marks above lip and broken tooth.
a. post seizure trauma
b. NAI ??
c. Scurvy
d. phenytoin toxicity

2. 12-year-old with unprotected sex and vague abdominal tenderness.


what to do next.?
a. Swabs
b. urgent child protection
c. US abdomen
d. obstetric referral
e. levonorgesteril

3. a child with increasing head circumference, pale, at home with apnea.


what investigation to do next.?
a. CT
b. skeletal survey

Sept 2014
1. picture Child abuse mark Cigarrate mark

444
Safeguard

2. picture Child abuse mark Shoe or bite mark.?

3. What make you suspect NAI ?


(2 answer needed).
a. Posterior rib fracture
b. Hb 1 g/l ???
c. Nose bleed
d. Weight less than 2nd centile

4.1.5 yr girl with grandmother, anal patulous instipation, faecal impaction,


wart--‐? 2 answers
Child abuse???
Social worker???

5. 17-year-boy fell down and died inspite of resuscitaion--‐?


a. Full investigation should be ordered
b. Doctor on duty to give death certificate
c. Father can refuse post mortem.

6. Hpernatremia with high urine osmaolarity 800 with foster parents?


Salt poisoning

445
Safeguard

7. Neonate which looks drowsy with Subdural hge?


a. NAI
b. Gulatric aciduria
c. Sepsis

Jan 2013
1. a 10 years old with psychiatric(home) trouble, secondary enuresis
encopresis that is get worse, that he did defecate without knowing he
did,, on examination: lax anus ,, suprapubic mass what’s diagnosis?
a. neuropathic bladder
b. sexual abuse

446
Palliative

447
Palliative

Sept 2021
1. Scenario about child had spinal surgery due to trauma then walked alone
without support or analgesia after few days he had pain and need
support to walk but no pain at rest. Which initial analgesia you will give??
a. Paracetamol + Ibubrofen
b. Oral morphine
c. Fetanyl patch
d. S/c morphine

May 2021
1. A 5-year-old child, a known case of ALL, developed relapse after bone
marrow transplantation. He is hospitalized and on end-of-life plan. He is
pale with an HB of 8. He has bilateral pleural effusion. The mother called
you because he has labored breathing, but is otherwise comfortable and
calm, what is the most appropriate management?

a. Low flow mask O2


b. Blood transfusion
c. Pleurocentesis
d. Furosemide
e. Doxapram

2. A patient with second stage neuro blastoma and bone pain, on regular
Paracetamol with no improvement. What is next step in management?

a. Oral morphine
b. Localized radiotherapy
c. Fentanyl patch
d. Morphine with subcutaneous pump

448
Palliative

Jan 2021
1. 5yrs old girl on morphine end stage abdominal neuroblastoma had
constipation (not passing stool for 2 days) and not pass urine since
1 pm. It’s 10 am now. What to do next? B , c , d ---- zero palliative

a. replace with naloxone ??


b. advice warm bath
c. urethral catheter
d. poly ethylene glycol
e. encourage fluid intake ??

Sept 2020
1. Palliative patient on oral morphine and paracetamol, well controlled,
develops urine retention. What to give for his moderate to severe pain?
a. Intranasal diamorphine
b. Transdermal fentanyl patch
c. SC Morphine infusion
d. Oral codeine
e. Oral methadone

Jan AKP
1. Girl has tumor she take morphine oral 5 mg prn then 4 times on the last
24 hr now she complain of severe pain what will you do ?
a. oral morphine 5 mg
b. 10 mg morphine slow release every 12 hr
c. morphine IV
d. 5mg morphine every 4 hr
e. fentanyl patch

449
Palliative

2. 12y Batten disease, presented with a cute onset of chest infection,


desaturation, spo2 80% not improved by nasal oxygen, respiratory
acidosis, deterioration, bilateral lung collapse, Mechanical ventilation
What to do? Two choice were required
a. Contact the consultant
b. Contact ICU
c. Abx and ivf
d. Check his file regarding active plan management
e. Non-invasive ventilation
f. Contact anesthesia
g. Chest physiotherapy

Sept 2019
1. palliative patient on pain given paracetamol not improve what to do?
a. oral morphine
b. local radiotherapy
c. fentanyl patch
d. codeine
e. IV morphine

May-2019
1. End stage life plan presented with agitation:
a. call the expert nurse
b. consultant
c. call ambulance

May 2018
1. OI multiple fracture asked what to give ?
Iv morphine ---- sever pain

450
Palliative

2. a child 12 years old with batten disease with developmental age of 6


months the child with a cute onset chest infection low saturation and
bilateral lung collapse, they asked what to do, 2 options?
a. Check registration for advance care
b. Call your consultant
c. Check name and intubate
d. Chest physiotherapy

Jan 2018
1. 5-yrs on morphine end stage neuroblastoma had constipation and less
urine what to do?
a. replace with naloxone
b. advice warm bath
c. urethral catheter
d. polynylene glycol

May 2017
1. child end stage cancer, on palliation, presented with increased agitation
and other symptoms, they contacted nurse. Best action?
a. Clinical review by medical doctor
b. To be assessed and managed by nurse
c. change in medications, was morphine and midazolam

May 2015
1. child on end-of-life plan is unconscious what would u do?
a. admit to hospital
b. get assessed by doctor
c. get nurse to check for treatable cause ??!!!
d. reduce morphine
e. increase midazolam

451
Dermatology

452
Dermatology

Sept 2021
1. Newborn 1 hour after delivery. What is the diagnosis?

a. Junctional EB
b. SSSS
c. Cutis aplasia
d. TEN

2. What is the treatment?


a. IV floxacillin
b. Topical fusidic acid

May 2021
1. A Picture of a 3-year-old girl. Her mother has hypothyroidism.
What TWO investigations to could lead to a diagnosis?
(100% derma ,100% endocrine answered a, d, f, g, h)

a. Thyroid function test


b. Thyroid US
c. HLA DQ2 DQ8
d. Skin scrapings for fungal infection
e. Skin biopsy
f. Anti -tissue transglutaminase
g. Serum copper
h. Serum Ca and phosphate
i. Hair microscopy

453
Dermatology

2. A 6-month-old infant, presents with diarrhea, failure to thrive and


recurrent oral and perianal candidiasis. He had been breast-fed until 4
months of age when the mother started formula which she thinks was the
cause of his condition. The weight is in the 0.4thcentile. What is the
diagnosis?

a. SCID
b. HIV
c. Acrodermatitis
enteropathica

Jan 2021
1. Scenario of infant having lesion over the body occasionally will flare up
and cause itchiness?
Mastocytoma

2. a photo of crops of vesicles with bullae over the shoulder and neck?

a. herpes simplex
b. herpes zoster
c. Bullous pemphigoid
d. Scald
e. Impetigo

454
Dermatology

Sept 2020
1. Baby came with this picture. What is dx?
a. Eczema herpeticum
b. HSP
c. Shingles
d. Impetigo
e. Acrodermatitis enteropathica

2. A neonate develops blisters on the foot leaving raw area since birth and
treated with antibiotics, not improved, after 2 days developed oral ulcers
and on buttocks and shoulders dx? (No picture!)
a. SSSS
b. EB
c. Impetigo
d. Incontinent pigmenti

3. Picture of a neonate with skin lesions, he has high eosinophils. Mother and
elder sister also had the same rash which evolved over period of months.
What is the dx? ‫عليه اختالف‬
a. Incontinent pigmenti
b. Herpes simplex
c. Varicella
d. EB
e. Langerhans cell
histiocytosis

455
Dermatology

5. Picture of a baby has 24 hemangiomas. What is your next step in


management?
a. Liver US
b. Oral propranolol
c. Renal U/S
d. Brain U/S
e. Laser therapy
f. Dermatology referral
6. A child came with fever and rhinitis; during examination you notice his
tongue like the picture. What is the dx?
a. Geographic tongue as sample paper
b. Candida
c. Scarlet fever
d. Leukoplakia
e. Lichen planus
7. 15 years old girl with a photo of bilateral erythematous rash on extensor
surfaces of legs, which is tender. What is the dx?
a. Erythema nodosum
b. Erythema multiform
c. HSP
d. ITP
e. Insect bite

Jan 2020
1. Picture of child has Fever for 3 days, was on antiepileptic drugs, has
conjunctivitis, mucositis, rash appear after 3 days then some blister
appeared (picture of rash in lips & conjunctivitis). O/ E ( in shock?)

a. SSSS
b. TEN
c. bullous (EB?)
d. Kawasaki
e. Bullous pemphigoid

456
Dermatology

2. picture of geographical tongue


a. reassure
b. start oral nystatin
c. acyclovir

3. picture of 1-hour neonate


a. ssss
b. EB
c. cutis aplasia
d. colloid baby

Sept 2019
1. Picture of Steven Johnson syndrome ask about the cause, patient was
have balanitis, chest infection
(cough),,need one option
e. mycoplasma
f. Staph
g. Sulphonamide
h. Carbamazepine

457
Dermatology

2. neonate with blistering started at the side of heal brik test ask what
best intial mangment?
a. urgent dermatology refer
b. pain management
c. dressing by silicon gauze
d. skin swab
e. antibiotic

May 2019
1. picture then Q About ttt

floxacillin IV

2. history of eczema not responding

A. asked about diagnosis


e. Chickenpox
f. eczema herpeticum
B. management
systemic antiviral

458
Dermatology

Jan 2019
1. (Picture) An infant with lesion in the eyelid and increasing-in size in the
last few months, ttt?

a. Oral propranolol
b. Dermatology referral

If option ophthalmology referral


will be better choice

2. picture of erythema nodosum

3. A case of 4 days history of low-


grade fever, cough, rhinitis,
erythematous papules,
irritable, not feeding well,
(Pic of scalded skin )
what is the diagnosis?
staph. Infection

459
Dermatology

Sept 2018
1. Picture infant with 24 small haemangioma , what to do ?

a. MRI brain
b. liver us
c. kidney us
d. propranolol
e. steroids

2. Child with large hemangioma face and neck & thrombocytobenia


kaseilbach merit $

May 2018
1. baby boy in incubator with rash and eosinophilia, family history sister &
mom?
a. Langerhans cell
b. inconenteta pigmentosa

460
Dermatology

2. Rash on trunk increasing since birth, itchy


Mastocytoma itchy on rubbing

3. a child with rash on trunk


(in paper 1)
a. capillary hemangioma
b. Mastocytoma

4. picture of the dorsum of hand of a child with rash

a. Acrodermatitis enteropathica
b. Eczema herpaticum
c. Impetigo
d. Chicken box
e. Scabies

Jan 2018
1. Picture of SSSS asking about ttt
Iv floxacillin

461
Dermatology

2. I think it was a picture of child who has eczema and developed rash behind
ear and was febrile what to give?
a. systemic antviral
b. systamic antbiotcs
c. syetemic steroids
d. zinc supplements
e. Topical antviral

3. Father has hereditary angioedema asking you about the risk for his child
what do you need to tell him?
a. Reassure as no risk of transmission
b. C1 esterase level and function need to be tested at neonatal period
c. Genetic testing later
d. C1 esterase later in life

Sept 2017
1. lesion on hand of child?
a. Varicella
b. Impetigo
c. Eczema herpeticum

2. Photo-Erythema nodosum

462
Dermatology

3. abdominal pain, lip swelling


H angioedema

May 2017
1. picture of child legs with fever and irritability within 12 hours
SSSS

2. impetigo

3. picture of erythema nodosum painful rash

463
Dermatology

4. picture of epidermolysis bullosa

Jan 2017
1. skin hemangioma. . History of multiple cutaneous hemangiomas. .
Next to do?
abdominal ultrasound
(similar question in new format

2. picture of a child with eye hemangioma /stye asks about next best action?
a. Ophthalmology referral
b. beta blocker

3. a spot diagnosis of mastocytoma

464
Dermatology

May 2016
1. A case with history of epilepsy before 2 days complain
of fever conjunctivitis oral and lips cracked then deteriorated with
tachycardia tachypnoea and hypotension ask about diagnosis?

a. staphylococcus syndrome
b. bollus impetigo
c. Toxic epidermal necrolysis TEN
d. kawasaki disease

2. Geographic tongue

3. Picture of face
Eczema herpiticum

465
Dermatology

4. picture of molluscum contagenosum , mother ask what to do?


Reassure

Jan 2016
1. ( picture) 9 y girl with tender rash in the lower limb other with normal :
a. Erythema multiform
b. Erythema nodosum
c. Insect bite
d. child abuse

466
Dermatology

May 2015
1. a child with a lesion on chest..in picture.
a. mastocytoma
b. haemangioma

2. a child with rash looked like acrodermatitis but hx was recurrent diarrhea
and thrush.
a. HIV
b. acrodermatitis

Acrodermatitis

467
Dermatology

3. child with SJS rash..had hx of cough recently..possible cause..


a. sulphonamide
b. mycoplasma
c. streptococcal infection.
d. carbamazepine

Sept 2015
1. Picture of Ecezema herpitcum

2. Picture of Shingles

468
Dermatology

3. Picture of Erythema nodosum

4. Picture of Acne

5. abdominal pain, lip swelling. (EMQ)


H. angioedema

469
Ophthalmology

470
Ophthalmology

Sept 2021
1. EMQ
A. SEPTOOPTIC DISPLASIA
B. CONGENITAL CTARACT
C. PRIMARY CONGENITAL GLUCOMA
D. RETINOBLASTOMA
E. CONGENITAL TOXOPLASMOSIS
F. NAI
G. POST HEMORRHAGIC HYDROCEPHALUS
H. PVL
I. RETINOPATHY OF PREMATURITY
J. HYPOXIC ISCHEMIC ENCEPHALOPATHY
A. 6 weeks baby came for routine checkup parent are concerning that baby
has vision loss, mother age was 46 years. At birth baby was born by SVD
was hypotonic and heart murmur.
CONGENITAL CTARACT

B. 3 months old baby parents thinking that baby cannot see, baby born SVD.
At birth was hypotonic and had abnormal movement during 1st week of
life and feeding difficulties.
HYPOXIC ISCHEMIC ENCEPHALOPATHY

C. 3 months old baby parents thinking that baby cannot see. Baby has full
fontanel and once vomiting but the occipito-frontal diameter keep with
other growth parameters.
NAI

2. Picture of 7 days neonate, SVD. What is the causative organism?


a. Gonorrhea
b. Chlamydia
c. GBS
d. Pseudomonas
e. Staph

471
Ophthalmology

3. What is the causative organism?


e. Staph auras
f. Hemophilus influenza
g. Pseudomanas
h. Klebsiella

May 2021
1. A 14-year-old who lives with her grandmother because her mother died
after having breast cancer, presents with headache, weight loss and
difficulty in upward gaze. There was bilateral loss of pupillary reflex and
bilateral nystagmus. What is the diagnosis:
a. Perinaud's syndrome
b. Oculomotor palsy

2. A 3-year-old child. What is the diagnosis?


(No scenario)
a. Central retinal artery occlusion
b. Tay-Sachs disease
c. Toxoplasmosis
d. Toxocariasis
e. ROP

3. A 5-year-old child with orbital cellulitis (and fever???) received Amoxicillin


that was stopped after 5days and
he presented with this picture.
What is the diagnosis?

a. Cavernous sinus thrombosis


b. Ethmoidal sinusitis
c. NAI

472
Ophthalmology

Jan 2021
1. Photo of orbital cellulitis

2. Patient with painful eye movements with CT scan picture, asking for the
diagnosis?

a. orbital cellulitis
b. cavernous sinus thrombosis

3. pic of 6m old with recent history of convergent squint and neonatal


examination was normal?

a. retinoblastoma
b. aniridia
c. congenital glaucoma
d. congenital cataract
e. coloboma

473
Ophthalmology

Sept 2020
1. Child came with diplopia asked to look up and left, what is the muscle
that is not functioning? ‫اإلجابة صح والريكول كامل جدا‬

a. Right inferior oblique


b. Right superior oblique
c. Right inferior rectus
d. Left inferior rectus

2. Picture of the eye of a child with abdominal


distension, What is dx?

a. Aniridia
b. Glaucoma
c. Kaiser Fleischer ring
d. William
e. Lens subluxation

Jan 2020
1. an iridia picture two associated
a-genital (wagar)
b-nephroblastoma (wilmis tumour)
c-retinoblastoma
d-medulloblastoma

474
Ophthalmology

Sept 2019
1. eye picture 3-month of age
a. congenital glaucoma
b. aniridia
c. cataract
d. coloboma

2. picture of child with orbital cellulitis and pneumonia, patient is clinically


unwell, have pets at home, vaccinated child ask about causative organism
f. staph
g. kelebsella
h. pseudomonas
i. cat scratch
j. haemophilus influenza

May 2019
1. PICTURE diagnosis?
a. Tay-sach disease
b. retinal artery occlusion
c. retinitis pigmentosa

475
Ophthalmology

2. picture of 6-month old normal neonatal examination with recent history


of convergent squint

a. reitinoblastoma
b. aniridia
c. congenital glaucoma
d. congenital cataract
e. cloboma

3. history of restricted painful eye movement

e. Orbital cellulites
f. periorbital cellulitis
g. optic glyoma
h. retinal vein thrombosis

Jan 2019
1. A Picture of a child who has cyanotic heart disease, he had unilateral
Ptosis and meiosis what is the cause of facial appearance:

d. Horner syndrome
e. DiGoerge
f. Neuroblastoma

476
Ophthalmology

2. A picture of a swollen red periorbital region, for 24 hours, ttt?

e. IV ceftriaxone+ flucloxacillin
f. oral co-amoxiclav
g. social referral
h. chloramphenicol

3. EMQ:
A. hypopigmented lesion, infantile spasm
angiofibromata on eyelid
B. obese, polydactyly, learning difficulty
pigmentary retinopathy
C. Hypoglycaemia, underdeveloped genitalia, septo-optic dysplasia
Pale optic disc

Sept 2018
1. 5-years-old, diplopia, asked to look up and left lateral, what’s muscles is
affected (not functioning)? (didn’t mention brown syndrome)
a. Rt inf oblique
b. RT sup oblique
c. left sup rectus
d. RT inf rectus
e. RT medial rectus
f. lt inf rectus

477
Ophthalmology

2. 2 yrs old Child picture showing large red reflex, abdominal distension
Aniridia

3. 7-year-old with fever, pain, swelling around eye he take oral antibiotic for
2-days then stop, After 5 days he came back with this pic
a. Cavernous thrombosis
b. ethmoid sinusitis
c. retinoblastoma

May 2018

1. photo fundoscopy cherry red spot what is


the cause?
Tay sach

Jan 2018
1. Picture of both eyes showing red reflex what diagnosis?
a. Cataract
b. Glaucoma
c. aniridia

478
Ophthalmology

Sept 2017
1. left eye swelling and enema photo
iv antibiotic
There were 2 IV

2. eye photo with abdominal mass


a. Aniridia

3. eye photo? ataxia telangiectasia

479
Ophthalmology

4. Girl with nystagmus and loss of upper gaze


a. occulomotor palsy
b. perinaud syndrome

May 2017
1. picture of infant big eyes and black big iris
congenital glaucoma

Jan 2017
1. a case of WAGR syndrome . . Abdominal mass. . Picture

ANIRIDIA

2. a picture of cherry red spot Asks about a possible cause


Tay sach disease

480
Ophthalmology

3. case history of Orbital cellulitis, 3 days later presented with severe


headache . . CT picture
a. Ethmoid sinusitis
b. Cavernous sinus thrombosis

May 2016
1. Picture of Aniridia ask about associated conditions 2 options?

a. neuroblastoma
b. nephroblastoma (wilms tumour)
c. hepatoblastoma
d. Genitourinary malformation

WAGR $

2. Child came with diplopia asked to look up and left, what is the muscle
that is not functioning?

a. Right inferior oblique


b. Right superior oblique
c. Right inferior rectus
d. Left inferior rectus

481
Ophthalmology

Jan 2016
1. Child with orbital cellulitis received ttt come later with the ( in pic) reddish
and mildly swollen left eye with smooth lt nasolabial fold

a. Cavernous sinus thrombosis


b. Sinusitis
c. Neuroblastoma

Sept 2014
1. Picture Rt side Horner Syndrome

2. picture of Periorbital cellulitis --treatment?


Oral antibiotic co-amoxiclav

482
Ophthalmology

3. picture of patient with abdominal mass and abnormal iris?


Aniridia

4. picture of Telangiectasia of eye diagnosis?


Ataxia telangiectasia

5.Girl with nystagmus and loss of upper gaze


Perinaud syndrome

483
Genetics

484
Genetics

Sept 2021
1. Infant presented with feeding difficulties. What is the associated cardiac
anomaly?

a. Supravalvular
aortic stenosis
b. AVSD
c. Pulmonary
stenosis
d. CoA

2. Infant from Pakistan, his parents have concern about abnormal face
shape. What is the Dx?

a. Rt VI cranial n palsy
b. Corouzon syndrome
c. Thalassemia
d. Sickle syndrome
e. Sinusitis

3. Child with cleft palate, systolic murmur at lower sternal edge, facial
dysmorphism, asking about diagnosis
a. Digeroge syndrome
b. William
c. Down syndrome

485
Genetics

4. Pt. presented with behavioural changes and school deterioration.


O/E: head circumference below 2nd centile, heart murmur, short
palpebral fissure and thin upper lip. What is the diagnosis?
a. SSPE
b. Fetal alcohol syndrome
c. William syndrome
May 2021
1. EMQ
A. Rett syndrome (1)
B. Sotos syndrome (3)
C. NF1
D. TS
E. Fragile X syndrome
F. Angelman syndrome
G. 22 q11 deletion (2)
A. A 2-year-old girl presents with speech and language regression,
microcephaly and specific hand movements. She started walking at
15 months.
Rett syndrome
B. A 6-year-old with upward slanting of palpebral fissure, prominent
nasal bridge who underwent TOF operation and cleft palate repair?
22 q11 deletion
C. A 7-year-old presents with macrocephaly, tall stature and mild
learning difficulty. The growth velocity was high before school age.
Sotos syndrome

2. A neonate presents with a large protruded tongue and hemihypertrophy.


He was noticed to be irritable, hypotonic and uneasy to settle before
feeding with the symptoms improving with feeding. All investigations at
8 hours were normal (Glucose at low normal) , What is the diagnosis?
a. Beckwith Wideman's syndrome
b. Congenital hypothyroidism
c. Russell’s Silver’s syndrome
d. Down syndrome

486
Genetics

3. A 4-year-old girl with mild developmental delay and a cardiac murmur.


What is the diagnosis?

a. Fetal alcohol syndrome


b. Cornelia de Lange syndrome
c. 22 q 11 deletion

Jan 2021
1. What is the diagnosis of this child (photo of smiling boy with wide spaced
teeth “peg shaped teeth “and blond hair)? The hair was more dense than
this photo

a. Ectodermal dysplasia
b. Ellis van creveld syndrome
c. William syndrome
d. Angelman syndrome
e. Ehler Danlos
f. Albinism

2. 3 yr old down syndrome, in annual follow up, height and weight on


chart, what are the TWO essential surveillance options?
a. Coeliac
b. RBS
c. cervical spine radio
d. audiology
e. FBC & bl film
f. Echo
g. Thyroid
h. Optometrist

487
Genetics

3. 8 years old child has hearing loss, later on developed loss of night vision,
diagnosis ?
a. Rubella
b. CMV
c. Usher syndrome

4. Child have history of Hypoglycemia and photo of ear,


ask about follow up?

a. abdominal ultrasound
b. Renal function test
c. Brain imaging

5. Child with learning disabilities, thin upper lip, short palpebral fissure-?

a. fetal alcohol syndrome


b. William syndrome

Sept 2020
1. EMQ
Pierre Robben syndrome
Smith Lemli Opitz
22 q deletion
Fetal alcohol syndrome

488
Genetics

All have cleft palate


A. Micrognathia, glossoptosis
Pierre robin syndrome
B. Syndactyly between 2nd and 3rd toes, anteverted nose with cleft palate
smith Lemli Opitz Syndrome
C. VSD, cleft palate, hypocalcemia
22 q deletion

Jan 2020
1. Case of central hypotonia, undescended tests, feeding problem, not
dysmorphic, positive reflexes, ventilated in neonatal periods 3 months

a. Beckwithwidman
b. SMA
c. Prader-willi
d. mytonic dystrophy
e. Zellweger

Sept 2019

489
Genetics

1. picture of child iris ask about associated cardiac lesion


a. AVSD
b. Supra valvar aortic stenosis
c. pulmonary stenosis

2. EMQ
A. cleft lip and palate microcephaly, heart mummer
Digorge syndrome

B. mild mental retardation big head like father


Soto’s syndrome

C. scenario of female 18-months old with developmental regression and


repetitive hand movement, ,fall frequently
Rett syndrome

3. 14yrs old female short stature height in 0.4centile wt on 25centile


breast devolopment stage 2,,pupic hair stage 1 .mother has menarche
at 14yr, concerning about delay menarche

a. turner syndrome
b. hypothyroidism
c. constitutional

490
Genetics

May 2019
1. picture of (girl with blond hair and stellate iris)
history Poor feeding and irritability in neonatal period

a. Praderwili
b. Angelman
c. William syndrome

2. obese child with hemi-hypertrophy, protuberant tongue RBS normal

a. bickwithman syndromes
b. hypothyroidism

3. senario of patient with black perioral pigmentation ,low hb, anaemia ask
about diagnosis?
peutz jeguhre syndrome

4. clear scenario about patient with FRAGILE X SYNDROME has large ears
and large genitalia his cousin had mental retardation

491
Genetics

Jan 2019
1. Picture of new born died soon after birth, parents reluctant to give
permision for post mortem investigation,

A. what is the diagnosis?


a. Thantophoric dwarfism
b. Jeune asphyxiating
c. Hydrops fetalis
B. What investigation can help?
a. skeletal survey
b. skin swab
c. Fibroblast enzymes
d. Chromosomal studies

2. (picture) boy with VSD (with smooth philthrum& thin upper lip) :
fetal alcohol syndrome

492
Genetics

Sept 2018
1. 3-yr old down syndrome, in annual follow up, height and weight on chart,
what’s 2 essential surveillance?
a. Coeliac
b. RBS
c. Cervico-spinal radio
d. audiology
e. FBC & blood film
f. Echo
g. Thyroid
h. optometrist

2. 8-yrs-old, hearing loss , vision loss at night , diagnosis ?


a. rubella
b. cmv
c. usher $

3. Cleft palate, systolic murmur left lower sternal edge, feeding difficulty ,
subtle face ?
a. down $
b. digeorge $
c. pierr robin $

4. Picture girl webbed neck , 10 yr old girl, small for age, with murmur, goes
to normal school need some help in school ,Most common heart defect ?

a. bicuspid aortic valve


b. peripheral pulmonary artery stenosis

493
Genetics

May 2018
1. history about baby with large head and had breathing difficulty, they
give us x-ray of his leg options:
Thanatophoric dwarfism
Telephone handle femur

2. EMQ of syndromes all with cleft palate:


A. cleft palate, micrognathia, post displaced tongue ,feed with orthodontic
teats
Pierre robin
B. colobomata, heart : fallot &vsd ,chonal atresia,growth mental
retardation,genitourinary anomaly:hypoplasia , ear anomaly : deafness,
cleft palate, facial dysmorphism
CHARGE
C. hypertolirsm,low set ears,antimongloid eye slants, micrognathia,cleft
palate , carp shape mouth
DiGeorge
D. microcephaly , cleft palate, heart defects, fused second and third toes,
extra fingers and toes and underdeveloped external genitals in males,
low cholesterol
smith – lemli-optiz

494
Genetics

CHARGE

Digorge
smith – lemli-optiz

3. A newborn with hypotonia, Poor feeding, Mild dysmorphic features,


diagnostic investigation?
FISH ?? methylation test, microarray

Jan 2018
1. picture of William $

495
Genetics

2. Image .. Ectodermal dysplasia (a boy was smiling , with blond hair ,, not so
typical face of an Ectodermal patient ,,, but his teeth were similar to this
photo)

3. X-ray: butterfly vertebrae


Alagille syndrome

Sept 2017
1. stellate iris

496
Genetics

May 2017
1. Picture. Eyes with stellate iris asking for associated cardiac lesion?
supravalvular aortic stenosis

2. scenario of beck-withwedman syndrome


Jan 2017
1. still birth baby, large head, short arms

a. Ostegenesis imperfecta type 2


b. Thantrophic dwarf.

2. a picture of smiling girl, history of poor feeding as an infant

Angelman syndrome

497
Genetics

3. spot diagnosis of foetal alcohol syndrome


Eyes covered in picture.

May 2016
1. Picture of webbing neck girl with learning difficulties ask about cardiac
lesion suspected?
a. supravalvular aortic stenosis
b. bicuspid aortic valve (if turner confirm)
c. VSD
d .peripheral

498
Genetics

2. Picture of child has ear transverse line baby developed hypoglycaemia at


birth How can follow up?
a. Hypothyroidism
b. serial abdominal US

3. Picture of child has ectodermal dysplasia

Ellis van Creveld syndrome


mode of inheritance not fully understood
Skeletal - short stature, post-axial polydactyly,
acromelic dwarfism, short
fingers (brachydactyly) with difficulty making a
fist •
CVS- ASD
• Ectodermal dysplasia : fine diffusely sparse
hair, hypoplastic small nails, teeth
abnormalities (hypoplastic or peg teeth),
congenital teeth may be present.

499
Genetics

May 2015
1.Child with features of William syndrome

2. Child with x-ray showing butterfly vertebrae..

Alagille syndrome

500
Genetics

3. features of Sotos, fragile X, FAS, digeorge, beckwith weidmen

Fragile X Syndrome

Fetal Alcohol Syndrome:

Sotos Syndrome:

501
Genetics

Beckwith-‐Wiedemann Syndrome

4. An x-ray with large head and telephone handle deformity of femur


Thanatophoric dysplasia

502
Genetics

Sept 2014
1. picture of Brush field spot

503
Statistics

504
Statistics

Sept 2021
1. Statistics: study about the effect of mother who take large dose of
vitamin E and incidence of asthma in their children. Odds ratio of
children having asthma for mothers take high dose of Vit. E 0.84 C.I
(0.72 - 0.98) Which one of the following best describes this study?
a. Confidence limit means results are significant
2. Statistics: Study about risk factors of RSV hospitalization.Down $ related
heart disease Odds ratio 3.24 C.I (1.80 -5.80). All children with
cardiomyopathy odds ratio 5.84 C.I (1.26 - 27.16). Haemodynamically
significant heart disease Odd ratio 1.53 C.I (1.4 -2.26). Relative risk and C.I
for need for respiratory support R.R 0.47 C.I (0.32 - 0.67) per each year of
increased age. Which one of the following best describes this study?
a. Severity of RSV decreases with each additional year of age.
b. RSV infection increases the risk for respiratory support.

3. Study: Moderate Hypothermia to Treat Perinatal Asphyxia


Encephalopathy. Background: It is uncertain whether hypothermic
therapy improves neurodevelopmental outcomes in newborn infants
with asphyxial encephalopathy. Methods: A trial was performed in
infants less than 6 hours old and at least 36 weeks gestation with
hypoxic encephalopathy. The trial compared intensive care plus cooling
of the body to 33.5°C for 72 hours and intensive care alone. The primary
outcome was death or severe disability at 18 months of age. Results: Of
325 infants enrolled after obtaining permission from parents, 163
underwent intensive care with cooling, and 162 underwent intensive
care alone. In the cooled group, 42 infants died and 32 survived but had
severe neurodevelopmental disability, whereas in the non-cooled group,
44 infants died and 42 had severe disability relative risk for either
outcome, 0.86; (95% confidence interval [CI] 0.68 to 1.07). Relative risk
(RR) of survival without neurologic abnormality in the cooled group was
1.57 (95% CI, 1.16 to 2.12). Among survivors, cooling resulted in cerebral
palsy with RR of 0.67 (95% CI, 0.47 to 0.96). Which one of the following
best describes this study?
a. CP incidence is reduced by cooling therapy

505
Statistics

May 2021
1.
RCT
AIM: compare the effect of hospital treatment of pneumonia by IV
antibiotics for 2 days then home treatment for 3 days versus
ambulatory treatment by oral antibiotics 5 days at home.
Method: Follow up at day 1,3,7 and14
Result: failure rate in hospitalized group: 88%, in ambulatory group:
78% and the risk difference between both groups was 10.
95% CI (-1.3 – 2.6)
Select one answer:
a. Amulatory treatment improves outcome
b. Hospitalization improves outcome
c. No difference between the two
d. Study design is prone to observer bias

2.
Study: to see the effect of folate supplementation to pregnant
females during the first trimester on wheezes and asthma for
infants at age of 18 months
Method: compare their results with hospital documents (year 2000-
2002) Data collected by asking mothers about symptoms
Study done at year 2009
Result: risk ratio between groups as regard of wheezes symptoms at
age of 18 monthwas 2% 95% CI (-1 – 2.6)
Select 1 answer:
a. Folic acid supplementation reduces the risk of wheezes
b. Folic acid supplementation reduces the risk of asthma
c. Pregnant females should take folate supplementation during first
trimester
d. It is a case control study
e. No difference (or no significance) in supplementation during the first
trimester on wheezing and asthma

506
Statistics

3.
Study: to compare the effect of adjusting O2 saturation in irst day
of life between 85-90 or 90-95 for patients aged between 23-28
weeks
Aim: primary outcome assess degree of neurodisability, CP and death.
Secondary outcome: ROP
Result: All CI of death, neurodisability were insignificant No CI about ROP.
Difference in last time of needing supplemental
O2 in both groups was -.8 weeks CI (-1.5-1)
Select 1 answer :
a. There is no significant difference between both groups as regards
neurodisability and death
b. There is no significant difference between both groups as regards
duration of O2 need
c. There is no significant difference between both groups as regards R

507
508

You might also like